Tuyển tập một số bài toán tổ hợp ôn thi HSG Toán - TOANMATH.com

153 37 0
Tuyển tập một số bài toán tổ hợp ôn thi HSG Toán - TOANMATH.com

Đang tải... (xem toàn văn)

Tài liệu hạn chế xem trước, để xem đầy đủ mời bạn chọn Tải xuống

Thông tin tài liệu

Nếu khả năng 1 xảy ra, Tí sẽ kẻ một đoạn thẳng nối hai điểm không cô lập với nhau; nếu khả năng 2 xảy ra, Tí sẽ kẻ một đoạn thẳng nối một điểm không cô lập với một điểm cô lập. Vì thế, d[r]

(1)

Tuyển tập một số bài toán tổ hợp

Sưu tầm Latex

Hướng tới kỳ thi VMO 2021

Phát hành blog lovetoan.wordpress.com

(2)(3)

Tổ hợp vấn đề khó tốn sơ cấp nói chung kì thi tốn cấp chủ đề ln có chỗ đứng định Các tốn tổ hợp đơi khơng cần biến đổi tốn học phức tạp mà địi hỏi tư nhạy bén người làm bài, việc luyện tập với nhiều toán giúp luyện thêm kiến thức kĩ xử lý toán Với mong muốn tạo tài liệu giúp bạn học sinh ơn luyện chủ đề khó nhằn này, fanpage cố gắng tổng hợp nhiều sưu tầm thành tuyển tập nho nhỏ giúp bạn luyện tập chuẩn bị cho kì thi olympic toán tới mà bạn tham dự Tài liệu kết hợp nhiều nguồn, nhiều tài liệu khác lại nhằm mang tới cho bạn đọc tốn thú vị Trong khơng đề cập tới phương pháp như: đếm hai cách, truy hồi, song ánh, hàm sinh, Các bạn tìm đọc chúng tài liệu khác Hy vọng công cụ đắc lực bạn

Mọi ý kiến đóng góp thắc mắc vui lịng gửi địa

Tạp chí tư liệu toán học

(4)(5)

Lý thuyết tổ hợp.

1.1 Các quy tắc tổ hợp bản.

Định nghĩa Tập không rỗng A tập hữu hạn tồn số nguyên dương n song ánh f : 1,2, , n→A Trong tập tập A bao gồm n phần tử, nói A tập hợp n Số số phần tử tập hợp A đặt |A| Tập rỗng ∅ hữu hạn định nghĩa và|∅|= Một tập hợp gọi vơ hạn khơng hữu hạn Một tập hợp k A tập A bao gồm k phần tử

Quy tắc song ánh.Hai tập hợp khơng rỗng A B có số số phần tử tồn song ánh f :A → B Mặc dù quy tắc song ánh hiển nhiên đề lý sau Đôi người ta nên xác định biến với tính chất đưa nên suy tập A tất biến Nếu B tập hợp với số phần tử k tồn song ánh f :A→B có số phần tử k

Quy tắc nhân Để A B hai tập hợp hữu hạn f :A → B hàm số phần tử b ∈ B tồn xác k phần tử từ tập A mà có ảnh B Sau |A| = k.|B| Chúng ta thường sử dụng quy tắc để phân biệt số phần tử kết xếp không xếp chọn phần tử từ tập cho

Quy tắc cộng.Nếu A tập hữu hạn A=A1∪A2∪ ∪An Ai∩Aj =∅với tất I

khácj,

|A|=|A1|+|A2|+ +|An|

Chúng ta sử dụng quy tắc cộng xét câu hỏi tổ hợp để đếm số phần tử tập A Đơi tự nhiên dễ dàng để phân chia tập hợp A thành tập con.(khối) để xác địn số lượng phần tử khối để tính số phần tử thu

Quy tắc tích số Cho A1, A2, An tập hợp hữu hạn mà chứa k1, k2, , kn phần tử

và tích CartesianA1×A2× ×An tập hợp chứa k1k2 kn phần tử

|A1×A2× ×An|=|A1|.|A2| |An| (1)

Đặc biệt, A tập hợp chứa m phần tử An tập hợp chứa mn phần tử |An|=|A|n

Chứng minh.Ta chứng minh đẳng thức (1) quy nạp Vớin = đẳng thức (1)trở thành

|A1|=k1 Cho đẳng thức (1) với tập hợp chứa n−1 phần tử Bây xét tập hợp chứan phần tử là|Ai|=ki với i∈ {1,2,3, , n}vàAn ={x1, x2, , xkn} Theo

giả thiết quy nạp ta có

|A1.A2 An−1|=k1.k2 kn−1 (2) Với i∈ {k1, k2, , kn}, đặt

(6)

Đó hiển nhiên ánh xạ Si A1 ×A2 × ×An cho (a1, a2, , an−1, xi) → (a1, a2, , an−1) Do

|Si|=k1×k2× ×kn−1, i∈ {1,2, , kn} (4)

Lưu ý tập hợp S1, S2, , Skn tập hợp tách rời nhau, A1 ×A2× ×An=S1∪S2∪ ∪Skn

Sử dụng quy tắc cộng ta |A1×A2× ×An|=|S1|+|S2|+ +|Sn|=k1k2 kn Định lý

được chứng minh

1.2 Chỉnh hợp lặp.

Định nghĩa Cho A={a1, a2, , am} tập hợp thứ tự tuyến tínha1 < a2 < < am,

chok1, k2, , km số nguyên không âm cho n=k1+k2+ +km >0 Mỗi phần tử v ∈An,

sao cho với i ∈ {1,2, , m} phần tử xuất v ki lần, gọi n−

chỉnh hợp phần tử tập A loại (k1, k2, , km)

Định lý Cho điều kiện định nghĩa thỏa mãn,

1 Sốn−chỉnh hợp loại (k1, k2, , km)là

n!

k1!k2! km!

2 Số loạin−chỉnh hợp phần tử m−tập A

n+m−1

n

Chứng minh

1 Cho tập B = a11, a21, , ak1

1 , , a1m, a2m, , amkm Gọi S1 tập tất hoán vị tập B, S2 tập n− chỉnh hợp phần tử A loại (k1, k2, , km) Tập S1 bao gồm

(k1+k2+ +km)! = n! phần tử Ta định nghĩa hàm f : S1 → S2 sau: hoán vị p ∈ S1 tương ứng với mộtn− chỉnh hợpv ∈S2 thu từpbằng cách xóa số TậpS1 bao gồm k1!k2! km! hốn vị củaB, với j ∈ {1,2, , m}, phần tử a1j, a2j, , a

kj

j

chiếm kj vị trí cố định Vì phần tử v ∈ S2 có k1!k2! km! phần tử u ∈ S1 cho f(u) = v

|S2|=

|S1| k1!k2! km!

= n!

k1!k2! km!

2 ChoA={1,2, , m} vàS tập tất (n+m−1)− chỉnh hợp phần tử tập

{0,1,2, , m}có dạng sau

v = 11

| {z }

k1

0 22

| {z }

k2

0 0mm m

| {z }

km

Kí hiệu T tập tất loại n− chỉnh hợp phần tử A Khi đó, chỉnh hợp v ∈ S chứa m −1 số xác định cách vị trí chúng Vì vậy,

|S|=

m+n−1

m−1

Hàm số f :S →T, xác định f(v) = (k1, k2, , km) song ánh Do

ta có

|T|=|S|=

m+n−1

m−1

=

n+m−1

n

Vậy định lý chứng minh

Ví dụ Có số nguyên dương có chữ số, số xuất ba lần, số

(7)

Các số nguyên dương thỏa mãn tính chất là7−chỉnh hợp loại(3,2,2) Như ta có 7!

3!2!2! = 210

số nguyên dương

Ví dụ Có từ khác thu từ việc hoán vị chữ từ COMBINA-TORICS?

Lời giải

Các chữ C, I O xuất hai lần, chữ M, B, N, A, T, R, S xuất lần từ COMBINATORICS Vì vậy, số từ phân biệt thu từ việc hoán vị chữ

13!

2!2!2! = 778377600

1.3 Tổ hợp lặp.

Định nghĩa Định nghĩa Đặt A = {a1;a2; ;am} tập thứ tự nghiêm ngặt a1 < a2 < < am Một tổ hợp gồm n phần tử tập A cho phép lặp lại phần tử tổ hợp lặp

chậpn (f(1), f(2), , f(n)), f :{1,2, , n} →A hàm không giảm tức f(1) 6f(2) 6 6f(n)

Định lý Số tổ hợp lặp chậpn m phần tử tập A

n+m−1

n

Chứng minh Đặt A ={a1;a2; ;am} giả sử rẳng a1 < a2 < < am Đặt K tập tổ hợp

lặp chập n m phần tử A, T tập hợp chỉnh hợp chập n m phần tử A Xét hàm f :T →K, thỏa mãn với (k1;k2; ;km)∈T

f((k1;k2; ;km)) =a1a1 a1

| {z }

k1

a2a2 a2

| {z }

k2

amam am

| {z }

kmlần

∈K

Rõ ràng hàmf toàn ánh Như ta được|K|=|T|=

n+m−1

n

Ví dụ 3.1 Đặt A={a, b, c} a < b < c ta có mười tổ hợp lặp chập 10 phần tử A aaa, bbb, ccc, aab, aac, abb, acc, bbc, bcc, abc

Định lí 3.2 Số tổ hợp lặp chập n m phần tử tập A, thỏa mãn phần tử a ∈ A xuất lần

n−1

m−1

Chứng minh Đặt K tổ hợp lặp chập n phần tử A = {a1, a2, , am} thỏa mãn phần tử a∈ A xuất lần, V tập gồm chỉnh hợp chập n tập A∪ {0} Xét hàm f :K →V xác định bởi, với

u=a1a1 a1

| {z }

k1

a2a2 a2

| {z }

k2

amam am

| {z }

kmlần

∈K,

Đặt f(u) =a1a1 a1

| {z }

k1−1

0a2a2 a2

| {z }

k2−1

0 0am−1am−1 am−1

| {z }

km−1−1

0amam am

| {z }

kmlần Như định lý chứng minh

1.4 Nguyên lý bao hàm - loại trừ.

(8)

Ví dụ Có 10 học sinh đạt điểm cao mơn Tốn, 12 học sinh đạt điểm cao môn Lý học sinh đạt điểm cao hai mơn Tốn Lý Hỏi có học sinh đạt điểm cao trong hai môn này?

Lời giải

Gọi A B tập hợp học sinh đạt điểm cao mơn Tốn Lý u cầu tốn xác định số phần tử tập hợp A∪B Những học sinh đạt điểm cao hai môn Toán Lý đếm hai lần phép tính |A|+|B| Do

|A∪B|=|A|+|B| − |A∩B|

Cuối cùng, ta có|A∪B|= 10 + 12−7 = 15

Ví dụ Có số ngun dương tập hợp S={1,2, ,1000} chia hết cho số2,3 5?

Lời giải

Gọi A, B C tập S chứa phần tử chia hết cho 2,3 Ta cần tìm số phần tử tập hợpA∪B∪C Xét tổng|A|+|B|+|C| Những phần tử thuộc hai ba tập hợp A, B C đếm hai lần, phần tử thuộc ba tập đếm ba lần Vì

|A∪B∪C|=|A|+|B|+|C| − |A∩B| − |B∩C| − |C∩A|+|A∩B ∩C| (*) Chú ý tập hợpA∩B, A∩C, B∩C A∩B∩C chứa số S chia hết cho

6,10,15và 30.Vì vậy,

|A|= 500,|B|= 333,|C|= 200,|A∩B|= 166,|A∩C|= 100,|B∩C|= 66

và |A∩B∩C|= 33 Từ (∗)ta |A∪B ∪C|= 734 Định lý Cho A1, A2, , An tập tập hữu hạn S.Khi

|A1∪A2∪ ∪An|= n

X

i=1

|Ai| −

X

16i<j6n

|Ai ∩Aj|+

+ X

16i<j<k6n

|Ai∩Aj ∩Ak| − + (−1) n−1

|A1∩A2∩ ∩An| (1)

Số |A1∪A2∪ ∪An| vế trái đẳng thức (1) hạng vế phải viết

thành tổng số với dấu+ hay − tương ứng Mỗi phần tửa∈A1∪A2∪ ∪An tương ứng

với hạng tử vế trái của(1) Giả sử có rtập số A1, A2, , An chứaa.Khi phần

tử tương ứng với tổng r−

r

2

+

r

3

− + (−1)r−1

r r

= 1−(1−r)r =

Vậy đẳng thức (1) Công thức (1) thường xem Nguyên lý bao hàm – loại trừ Ta chứng minh thơng qua cơng thức tổ hợp đơn giản Cơng thức cịn chứng minh quy nạp sau

Chứng minh định lý quy nạp toán học Với n = đẳng thức (1) hiển nhiên Với n = cơng thức chứng minh ví dụ Giả sử (∗) với n = m > Xét A1, A2, , Am, Am+1 tập hữu hạn Theo giả thiết quy nạp với tập A1, A2, , Am ta

có đẳng thức

|A1∪A2∪ ∪Am|= m

X

i=1

|Ai| −

X

16i<j6m

|Ai ∩Aj|+

+ X

16i<j<k6m

|Ai∩Aj ∩Ak| − + (−1)m

−1

(9)

Chú ý |(A1∪A2∪ ∪Am)∩Am+1| =

m

[

j=1

(Aj ∩Am+1)

Theo giả thiết quy nạp với tập A1∩Am+1, A2∩Am+1, , Am∩Am+1 ta có

|(A1∪A2∪ ∪Am)∩Am+1|=

m

X

j=1

|Aj∩Am+1| −

X

16i<j6m

|Ai∩Aj ∩Am+1|

+ X

16i<j<k6m

|Ai∩Aj ∩ ∩Ak∩Am+1|

− + (−1)m−1|A1∩A2∩ ∩Am∩Am+1| (3) Từ (2) (3) vàví dụ với A=A1∪A2∪ ∪Am vàB =Am+1,ta

|A1∪A2∪ ∪Am∪Am+1|=|A1∪A2∪ ∪Am|+|Am+1| − |(A1∪A2∪ ∪Am)∩Am+1|

= m+1

X

j=1

|Aj| −

X

16i<i6m+1

|Ai∩Aj|+

X

16i<i<k6m+1

|Ai∩Aj∩Ak|

− .+ (−1)m|A1∩A2∩ .∩Am∩Am+1| tức Nguyên lý bao hàm – loại trừ với n=m+

1.5 Một phương pháp hình học chỉnh hợp.

Những tốn tổ hợp đưa dạng đếm số chỉnh hợp chập n phần tử

−1 với điều kiện kèm theo thỏa mãn Mỗi chỉnh hợp chập n c1c2 cn mà

chứa số−1và 1có thể kết hợp với quỹ đạoZ0Z1 Zn đóZk điểm có tọa độ (xk, yk)

sao cho đẳng thức sau thỏa mãn x0 = 0, y0 = với k ∈ {1,2, , n}, xk =xk−1 + 1, yk =yk−1+ck

1 Nếuck= yk−yk−1 = Zk−1Zk phần lên quỹ đạo

2 Nếuck=−1thì yk−yk−1 =−1và Zk−1Zk phần xuống quỹ đạo.Z0 điểm đầu Zn điểm cuối quỹ đạo Z0Z1 Zn

Định lý

1 GọiM(m, n)là số quỹ đạoZ0Z1 Zncó điểm đầu(0,0), điểm cuối(m, n)vớim >n,m, n∈N

Khi

(a) M(m, n) = m!

m+n

2

!

m−n

2

!

nếu m−n chia hết cho

(b) M(m, n) = m−n không chia hết cho2

2 Giả sửA(a1, a2),B(b1, b2)là điểm mà tọa độ chúng số nguyên chob1 > a1 >0, a2 > 0, b2 > Gọi A1(a1,−a2) điểm đối xứng với A(a1, a2) qua trục hồnh Khi phát biểu sau đúng: Số quỹ đạo với điểm đầu A điểm cuối B cho có điểm chung với trục hoành với số quỹ đạo với điểm đầuA1 điểm cuối B

3 Cho n ∈N gọi T1 tập hợp quỹ đạo với điểm đầu (0; 0) điểm cuối (2n; 0) cho tất điểm quỹ đạo bất kỳ, ngoại trừ điểm đầu điểm cuối, có tung độ dương Khi đó|T1|=

1

n

2n−2

n−1

(10)

4 Cho n ∈N gọi T2 tập hợp quỹ đạo với điểm đầu (0; 0) điểm cuối (2n; 0) cho tất điểm quỹ đạo có tung độ khơng âm Khi đó:|T2|=

1

n+

2n n

5 Cho n, k ∈ N T3 tập hợp quỹ đạo với điểm đầu (0; 0) điểm cuối (2n; 0) khơng có điểm chung với đường thẳngy =−k Khi |T3|=

2n n − 2n n+k

Chứng minh

1 Gọi t quỹ đạo với điểm đầu (0; 0) điểm cuối (m;n), α số phần lên t β số phần xuống t Khi ta cóα+β =m,α−β =n, tức α= m+n

2 ,β =

m−n

2 Vì α

β số nguyên không âm nên điều dẫn đến quỹ đạo với điểm đầu (0,0), điểm cuối (m, n) số nguyên m n số chẵn, số lại lẻ Nếu α β chẵn lẻ, tức m+n m−n chia hết cho2 ta có

M(m, n) =

m α = m m+n

2

= m!

m+n

2

!

m−n

2

!

2 Gọi S1 tập quỹ đạo với điểm đầuA điểm cuối B cho có điểm chung với trục hoành vàS2 tập quỹ đạo với điểm đầu A1 điểm cuối B

y x O A a1 A0 a2 b2 a2 X B b1

Ta xác định hàm f : S1 →S2 với ánh xạ f(t) quỹ đạo t ∈ S1 xác định sau Ký hiệu X điểm chung quỹ đạo t với trục hồnh cho X có hoành độ nhỏ Đặt t1 phần quỹ đạo t với điểm đầuA điểm cuối X t2 phần quỹ đạo t với điểm đầu X điểm cuối B Đặt t01 quỹ đạo thu cách đối xứng t1 qua trục hoành f(t) = t01∪t2 Hàm f song ánh ta có |S1|=|S2|

3 Mỗi quỹ đạo từ T1 chứa điểm(1; 1) (2n−1,1) Bài toán quy đếm số quỹ đạo với điểm đầu (1; 1)và điểm cuối (2n−1,1) khơng có điểm chung với trục hoành Số quỹ đạo với điểm đầu (1; 1)và điểm cuối (2n−1,1)

2n−2

n−1

, quỹ đạo bao gồm n−1phần lên n−1 phần xuống tất phần xếp thành dãy tất cách Số quỹ đạo với điểm đầu (1; 1) điểm cuối (2n−1,1) cho có điểm chung với trục hoành với số quỹ đạo với điểm đầu (1;−1) điểm cuối (2n−1,1) Mỗi quỹ đạo với điểm đầu (1;−1) điểm cuối(2n−1,1) cón phần lên n−2 phần xuống, số quỹ đạo với

2n−2

n

Do ta có

|T1|=

2n−2

n−1

2n−2

n

=

n

2n−2

n−1

(11)

4 Chứng minh tương tự trường hợp

5 Sử dụng lập luận tương tự trường hợp 2) ta thu số quỹ đạo với điểm đầu (0; 0)và điểm cuối (2n; 0) cho chúng có điểm chung với đường thẳng y = −k với số quỹ đạo với điểm đầu (0;−2k) điểm cuối (2n; 0), số M(2n,2k) tất quỹ đạo với điểm đầu (0; 0) điểm cuối (2n; 2k) Sử dụng kết thu trường hợp 1) ta có

|T3|=M(2n,0)−M(2n,2k) =

2n n

2n n+k

Vậy định lý chứng minh

Ví dụ Có 2n người thành hàng trước quầy bán vé Mỗi người họ có ý muốn mua vé, giá vé 5$ Giả sử n người hàng có 10$, n người cịn lại có 5$ Khơng có tiền mặt máy đếm tiền thời điểm bắt đầu Hỏi có cách để xếp lại hàng cho người có10$ hàng phải đợi để đổi?

Chứng minh.Từ định lý trên, ta thu số cách xếp lại hàng cho điều kiện đưa thỏa mãn

n+

2n n

Lưu ý Dãy số nguyên dương (xn)n>1 cho xn =

n+

2n n

, biết đến dãy số Catalan Lưu ý x1 = 1,x2 = 2,x3 = 5,x4 = 14,x5 = 42, Có thể chứng minh xn số nguyên lẻ nếun có dạngn = 2k−1với k số nguyên dương Số Catalan thường

xuất lời giải toán tổ hợp liệt kê

1.6 Phân hoạch nguyên

Ở ta xét biểu diễn số nguyên dương cho trước thành tổng số nguyên dương, biểu diễn tập hợp hữa hạn cho trước thành tập khác rỗng đôi không giao Sự biểu diễn gọi phân hoạch số nguyên dương phân hoạch tập hợp Ở xét số phân hoạch thỏa số điều kiện

Định nghĩa Một phân hoạch số nguyên dươngnthànhksố hạng bộα= (α1, α2, , αk),

thỏa mãn α1;α2; ;αk∈N

α1+α2+ +αk =n, α1 >α2 > >αk (1)

Các số nguyên dương α1;α2; ;αk gọi số hạng phân hoạch α Kí hiệu fi số lần

xuất số thứ itrong dãy α1;α2; ;αk, phân hoạch α cịn kí hiệu α = 1f12f23f3

, ta có

f1+f2+ +fn=k, f1+ 2f2+ +nfn =n (2)

Ví dụ

1 + + + 1,(5,3,3,1),(113251)là kí hiệu khác cách phân hoạch số 12 thành số hạng

2 + + + + + + + + + + + + + = (15335482)là phân hoạch số 50 thành 14 số hạng

3 Có phân hoạch khác số 5, α1 = 51

, α2 = 1141

, α3 = 2131

, α4 = 1231

, α5 = 1122

, α6 = 1321

, α7 = 15

(12)

Gọi p(n) số phân hoạch số nguyên dương n, qui ước p(n) = với n số nguyên âm p(0) = Dễ thấyp(1) = 1, p(2) = 2, p(3) = 3, p(4) = 5, p(5) = 7, p(6) = 11, Cơng thức tínhp(n)

được chứng minh Hardy, Ramanujan, Rademacher Ví dụ

p(10) = 42, p(20) = 627, p(50) = 204226, p(100) = 190569292, p(200) = 3972999029338

Hàmp(n) tăng nhanh theo n

Định lý Cho n1, n2, , nk số nguyên dương khác Kí hiệu F (n1, n2, , nk;n) số

phân hoạch số nguyên dươngn thành số hạng khác mà số hạng thuộc tập hợp

{n1, n2, , nk} Ta có đẳng thức sau

F (n1, n2, , nk;n) =F (n1, n2, , nk−1;n−nk) +F (n1, n2, , nk−1;n) (*)

Trong đó,F (n1, n2, , nj;m) =

0 m <0

1 m=

Chứng minh Gọi S tập hợp tất phân hoạch số nguyên dương n thành số hạng khác mà số hạng thuộc tập hợp{n1, n2, , nk} S1 tập củaS gồm phân hoạch mà chứa số hạng nk S2 tập S gồm phân hoạch mà không chứa số hạng nk.Theo giả thiết ta có|S|=F(n1, n2, , nk;n).Mỗi phân hoạch thuộc S1 tương ứng với phân hoạch n−nk thành số hạng khác thuộc{n1, n2, , nk−1} ngược lại Do

|S1|=F(n1, n2, , nk−1;n−nk)

Tương tự S2 tập hợp tất phân hoạch n thành số hạng khác mà số hạng thuộc tập{n1, n2, , nk−1}.Do đó|S2|=F(n1, n2, , nk−1;n) VìS =S1∪S2, S1∩S2 =∅ nên |S|=|S1|+|S2|

Hệ Đặt n1 = 1, n2 = 2, , nk =k, Fk(n) = F(1,2, , k;n) Fk(0) = 1, Fk(m) = 0,∀m <0

Khi

Fk(n) = Fk−1(n−k) +Fk−1(n) (***) Dễ thấy F1(1) = 1, F1(n) = với n > Fk(n) = Fn(n) với k > n Sử dụng đẳng thức (∗) ta có

thể tính đượcFk(n) với k, n số tự nhiên

Ví dụ Chúng ta tính vài giá trị Fk(n)

1 F2(1) = 1, F2(2) = 1, F2(3) = 1, F2(n) = 0,∀n>4

2 F3(1) = 1, F3(2) = 1, F3(3) = 1, F3(4) = 1, F3(5) = 1, F3(6) = 1, F3(n) = 0,∀n>7

3 F4(1) = 1, F4(2) = 1, F4(3) = 2, F4(4) = 2, F4(5) = 2, F4(6) = 1, F4(7) = 2, F4(8) = 1, F4(9) =

1, F4(10) = 1, F4(n) = 0,∀n >11

4 F5(8) =F4(3) +F4(8) = + =

Định lý Cho n1, n2, , nk số nguyên dương khác Kí hiệu G(n1, n2, , nk;n) số

phân hoạch số nguyên dươngn thành số hạng khác mà số hạng thuộc tập hợp

{n1, n2, , nk} Ta có đẳng thức sau

G(n1, n2, , nk;n) = G(n1, n2, , nk;n−nk) +G(n1, n2, , nk−1;n) (4*)

Trong đó,G(n1, n2, , nj;m) =

(13)

Chứng minh Gọi S tập hợp tất phân hoạch số nguyên dương n thành số hạng khác mà số hạng thuộc tập hợp{n1, n2, , nk} S1 tập củaS gồm phân hoạch mà tồn số hạng nk S2 tập S gồm phân hoạch mà không chứa số hạng Theo giả thiết ta có

|S|=G(n1, n2, , nk;n)

|S1|=G(n1, n2, , nk;n−nk)

|S2|=G(n1, n2, , nk−1;n) Ta có |S|=|S1|+|S2|

Hệ Đặt n1 = 1, n2 = 2, , nk =k, Gk(n) =G(1,2, , k;n) Gk(0) = 1, Gk(m) = 0,∀m <0

Khi

Gk(n) =Gk−1(n) +Gk(n−k)

Gk(n) =Gk−1(n) +Gk−1(n−k) +Gk−1(n−2k) +

Từ định lý ta dễ dàng xác định số phân hoạch số nguyên dương thành số hạng mà số hạng thuộc tập hợp số tự nhiên cho trước

Ví dụ Chúng ta xác định xem có cách có 50 la từ tờ tiền mệnh giá đô la, đô la, đô la, 10 đô 20 đô la Ta có G(1, n) = 1∀n>0 Theo định lý ta có

1 G(1,2; 2k) = +G(1,2; 2k−2) = k+G(1,2; 0) =k+ 1, k ∈N,

2 G(1,2; 2k+ 1) = +G(1,2; 2k−1) =k+G(1,2; 1) =k+ 1, k∈N,

3 G(1,2,5; 5) =G(1,2,5; 0) +G(1,2; 5) = + = 4,

4 G(1,2,5; 10) =G(1,2,5; 5) +G(1,2; 10) = + = 10,

5 G(1,2,5; 15) =G(1,2,5; 10) +G(1,2; 15) = 10 + = 18,

6 G(1,2,5; 20) =G(1,2,5; 15) +G(1,2; 20) = 18 + 11 = 29,

7 G(1,2,5; 25) =G(1,2,5; 20) +G(1,2; 25) = 29 + 13 = 42,

8 G(1,2,5; 30) =G(1,2,5; 25) +G(1,2; 30) = 42 + 16 = 58,

9 G(1,2,5; 35) =G(1,2,5; 30) +G(1,2; 35) = 58 + 18 = 76,

10 G(1,2,5; 40) =G(1,2,5; 35) +G(1,2; 40) = 76 + 21 = 97,

11 G(1,2,5; 45) =G(1,2,5; 40) +G(1,2; 45) = 97 + 23 = 120,

12 G(1,2,5; 50) =G(1,2,5; 45) +G(1,2; 50) = 120 + 26 = 146,

13 G(1,2,5,10; 10) = G(1,2,5,10; 0) +G(1,2,5; 10) = + 10 = 11,

14 G(1,2,5,10; 20) = G(1,2,5,10; 10) +G(1,2,5; 20) = 11 + 29 = 40,

15 G(1,2,5,10; 30) = G(1,2,5; 20) +G(1,2,5; 30) = 40 + 58 = 98,

16 G(1,2,5,10; 40) = G(1,2,5; 30) +G(1,2,5; 40) = 98 + 97 = 195,

17 G(1,2,5,10; 50) = G(1,2,5; 40) +G(1,2,5; 50) = 195 + 146 = 341,

18 G(1,2,5,10,20; 30) =G(1,2,5,10; 10) +G(1,2,5,10; 30) = 11 + 98 = 109,

(14)

1.7 Phân hoạch có thứ tự số nguyên dương

Định nghĩa Định nghĩa Một phân hoạch có thứ tự số nguyên dương n thành k phần nghiệm phương trình x1+x2 + +xk = n tập hợp số nguyên dương, tức k

bộ (k −tuple) (α1, α2, , αk) số nguyên dương có tổng n Với phân hoạch có thứ tự (α1, α2, , αk) ký hiệuα1+α2+ +αk

Ví dụ Ta có

1 Các phân hoạch số 3,2 + 1,1 + + Tất phân hoạch có thứ tự số

3,2 + 1,1 + 2,1 + +

2 Tất phân hoạch ó thứ tự số thành ba phận là5+1+1,1+5+1,1+1+5, ,4+2+ 1,4+1+2,2+4+1,2+1+4,1+4+2,1+2+4,3+3+1,3+1+3,1+3+3,3+2+2,2+3+2,2+2+3

Ví dụ Ký hiệu α = (1f12f2 nfn) là phân hoạch có thứ tự số nguyên dương n Khi đó có (f1+f2+ +fn)!

f1!f2! fn!

phân hoạch có thứ tự củan,sao cho,với mỗii∈ {1,2, , n}có fi thành

phần i

Chứng minh Với phân hoạch có thứ tự số nguyên dương n với fi thành phần i cho

mỗi i∈ {1,2, , n}, hiển nhiên (f1+f2+ +fn) xếp phân tử 1,2,3, ,n kiểu

(f1, f2, , fn)

Ví dụ Ta có

1 Số phân hoạch có thứ tự số nguyên dươngn thành k phần

n−1

k−1

2 Số phân hoạch số nguyên dươngn là2n−1

Chứng minh Gọi Alà tập hợp tất (n+k−1)sự săp xếp số với tính chất sau

Có n phần tử A 1. Khơng có hai số đứng kề Số hạng đứng đầu cuối số 1.

Tập hợp A chứa tất (n+k −1) xếp bao gồm dãy k số tách số Có song anhs từ tập A tất phân hoạch có thứ tự của số nguyên dương n k phần Để tiếp tục chứng minh định lý này, ta xét toán sau

Bài toán.Chom, nlà hai số tự nhiên GọiS0 tập thứ tự củam+nphần tử, gồmmchữ số vànchữ số GọiS1là tập củaS0, mà phần tử gồm chuỗi khơng chứa số liền nhau.S2 tập củaS0, gồm phần tử mà bắt đầu kết thúc số Tính|S0|,|S1|,|S2| Lời giải

GọiK tập gồm tổ hợpm phần tử tập{1,2, , m+n}và f :S0 →K hàm xác định Ánh xạ chỉnh hợp v =c1c2 cm+n tổ hợp trong K chứa số số hạngc1, c2, , cm+n

thì Khi rõ ràng f song ánh Do |K| =|S0| =

m+n m

Trong chỉnh hợp S1, số đặt vị trí đầu tiên, vị trí thứ số thứ hai, ,giữa vị trí thứ n−1và số thứ n, cuối sau số thứ n Do có n+ vị trí cho số Mà chỉnh hợp trongS1 xác định bởim n+ vị trí nên|S1|=

n+

m

(15)

hợp ta có |S2|=

n−1

m

Như áp dụng toán ta có |A|=

n−1

k−1

Sử dụng kết đẳng thức tổ hợp quen thuộc

n

X

i=0

n i

= 2n tất phân hoạch

sốn

n−1

X

i=0

n−1

i

= 2n−1

Định lý Cho n1;n2; ;nk số nguyên dương phân biệt Ký hiệu H(n1, , nk;n) số phân

hoạch có thứ tự số nguyên dương n với (parts) thuộc tập hợp {n1, , nk}

Khi có đẳng thức H(n1, , nk;k) = k

X

j=1

H(n1, nk;n−nj), đâyH(n1, , nj;m) =

(

0, m <0 1, m=

Chứng minh Gọi S tập hợp tất phân hoạch có thứ tự số nguyên dương n cho phần phân họach thuộc tập hợp {n1, n2, , nk} Gọi Sj tập hợp phân hoạch (α1, α2, ) ∈ S thỏa mãn α1 = nj Khi đó, tập hợp S hợp tập hợp đôi rời S1, S2, , Sk có

H(n1, n2, , nk;n) = |S|= k

X

j=1

|Sj|= k

X

j=1

H(n1, n2, , nk;n−nj)

Ví dụ Giả sử tin nhắn ký hiệu có dộ dài 0,1,2,3 hay đơn vị thời gian Hỏi có tin nhắn khác gửi suốt 10 đơn vị thời gian.?

Trong thí dụ tốn xác định H(1,2,3,; 4,10) Ta ký hiệu H(n) = H(1,2,3,4;n) với n∈N Dễ thấy H(1) = 1;H(2) = 2;H(3) = 4;H(4) = Từ định lý có

H(n) =H(n−1) +H(n−2) +H(n−3) +H(n−4)

Từ kết ta có H(5) = 15;H(6) = 29;H(7) = 56;H(8) = 108;H(9) = 208;H(10) = 401

1.8 Biểu đồ phân hoạch

Định nghĩa Định nghĩa Biểu đồ biểu diễn Ferrer’s phân hoạch α= (α1, α2, , αk), tập

hợp điểm có tọa độ nguyên Gα mặt phẳng tọa độ Decartes thỏa mãn Gα={(x;y)| −k+ 6y 60,06x6α−y+1−1}

Ví dụ Biểu đồ biểu diễn Ferrer’s phân hoạch 30 = + + + + + + + hình 6.3.1.Lưu ý dịng thứ i biểu đồ biểu diễn Ferrer’s phân hoạch (α1, α2, α3, , αk)

(16)

Định nghĩa Định nghĩa Gọi α = (α1, α2, , αk) phân hoạch số nguyên dương

đặt m = α1 Sự phân hoạch β = (β1, β2, , βm) gọi liên hợp α, ∀j = 1, m, βj

bằng số giá trị αi phân hoạch α mà αi >j

Ví dụ Cho phân hoạch + + + + + + + 1như ví dụ 1, phân hoạch liên hợp phân hoạch là8 + + + + + + +

Quan sát biểu đồ biểu diễn phân hoạch Ferrer’s phân hoạch β (là phân hoạch liên hợp α), ta nhận thấy có cách lấy đối xứng biểu diễn Ferrer’s phân hoạch α qua đường thẳng y=−x Đường thẳng gọi đường chéo biểu đồ Ferrer’s Từ ta thấy nếuβ liên hợp α α liên hợp β

Định lý Số phân hoạch số nguyên dương n thành k số hạng số phân hoạch n thỏa mãn khơng có số hạng có giá trị lớn k

Chứng minh Gọi S1 tập hợp tất phân hoạch số nguyên dương n thành k số hạng S2 tập hợp tất phân hoạch n thành số hạng không lớn k Với α thuộc S1, liên hợp củaα làβ thuộc S2 Ánh xạ f :S1 →S2 xác định f(α) =β với β liên hợp củaα song ánh Do |S1|=|S2|

Định lý Gọi p0(n) số phân hoạch số nguyên dương n thành chẵn số hạng phân biệt p1(n) số phân hoạch củan thành lẻ số hạng phân biệt Khi ta có

p0(n)−p1(n) =

(

(−1)k, n =

2k(3k±1), k ∈N trường hợp lại

Chứng minh.Gọi α = (α1, α2, , αk)là phân hoạch n thành số hạng phân biệt Ta đặt f1(α) =αk, f2(α) = max{j|αj =α1−j + 1}

Ví dụ, α= (7,6,5,3,2), f1(α) = 2, f2(α) = Xem hình vẽ

Gọi S tập hợp tất phân hoạch củan thành số hạng phân biệt.Ta xây dựng ánh xạ T1 T2 từ tập S đến S

1 Xét phân hoạch α = (α1, α2, αk)∈S, với α1 > α2 > > αk, giả sử f1(α)6 f2(α) Nếu f1(α) =f2(α) =k ta khơng làm Ta xác địnhT1(α)theo qui tắc: Ta bỏ số hạng nhỏ f1(α) =αk phân hoạch α, tăng thêm 1ở số hạng lớn phân hoạchα (tăng

(17)

T1

−→

Nếu f1(α) = f2(α) = k (k số số hạng phân hoạch), phép biến đổi T1 khơng định Thật xóa “dịng” αk = f1(α) = k ta cịn k −1 dịng nên khơng thể đủ k dịng để tăng dòng lên Trong trường hợp ta thấy α = (2k−1,2k−2, , k+ 1, k), ta có đẳng thứck+ (k+ 1) + + (2k−1) =

2k(3k−1)

2 Nếu f1(α) > f2(α) với phân hoạch α = (α1, α2, , αk), với α1 > a2 > > αk Nếu hai

đẳng thức f2(α) =k, f1(α) =k+ không đồng thời xảy ta xác định T2(α) theo quy tắc: giảm số hạng lớn phân hoạch α (giảm f2(α) số hạng) lập số hạng αk+1 =f2(α) Ví dụ α= (8,7,5,3)thì T2(α) = (7,6,5,3,2) Xem hình vẽ

T2

−→

Nếu f2(α) = k, f1(α) = k+ xảy đồng thời T2 khơng xác định, tạo thành “dòng” (2 số hạng nhau) Ví dụ α = (8,7,6,5), f1(α) = 5, f2(α) = 4, T2(α) = (7,6,5,4,4) Xem hình vẽ

T2

−→

Trong trường hợpf2(α) = k, f1(α) =k+ 1ta thấy α = (2k,2k−1, , k+ 2, k+ 1), ta có

(k+ 1) + (k+ 2) + + 2k =

2k(3k+ 1)

Ta thấy sau thực biến đổi T1 số số hạng phân hoạch giảm 1, thực T2 số số hạng tăng thêm Ta xét trường hợp sau

1 Trường hợp Nếu n khơng có dạng n =

2k(3k±1), với k ∈ N Khi với phân

hoạch α thực hai biến đổi T1 T2 Ta xác định ánh xạ T :S1 →S2 theo quy tắc

T(α) =

(

(18)

Ta thấy T song ánh |S1|=|S2| Ta nhận p0(n) = p1(n)

2 Trường hợp 2.Nếu n=

2k(3k±1), với k ∈Nthì

α0 =

    

(2k,2k−1, , k+ 2, k+ 1) n=

2k(3k+ 1) (2k−1,2k−2, , k+ 1, k) n =

2k(3k−1)

Ta gọiS1 (S2) tập phân hoạch n thành lẻ (chẵn) số hạng mà khác phân hoạch α0 Tương tự trường hợp ta suy |S1|=|S2| Ta nhận p0(n) = p1(n) + (−1)

k

Nhận xét Định lí cịn gọi định lí Euler-Legandre Số ωk =

1

2k(3k±1), k ∈ Z gọi

pentagonal numbers Nếu p(n) số phân hoạch củan ta có cơng thức sau

p(n) = X ωk6n

(−1)k−1

p

n− k(3k−1)

2

+p

n− k(3k+ 1)

2

Ở đâyp(0) = 1, tổng công thức chạy với số pentagonal number không lớn n

1.9 Nguyên lý Dirichlet.

Ngun lí Dirichlet - cịn gọi nguyên lí chim bồ câu (The Pigeonhole Principle) nguyên lý lồng nhốt thỏ nguyên lí xếp đồ vật vào ngăn kéo (The Drawer Principle) - đưa nguyên tắc phân chia phần tử lớp

Nguyên lý Dirichlet Nếu nhốt n+ thỏ vào n chuồng có chuồng chứa hai thỏ

Nguyên lý Dirichlet tổng quát Nếu có N đồ vật đặt vào k hộp tồn hộp chứa

N k

đồ vật (Ở [x]là số nguyên nhỏ có giá trị nhỏ x)

Nguyên lí Dirichlet mở rộng Nếu nhốtn thỏ vào m>2 chuồng tồn chuồng có

n+m−1

m

con thỏ

(19)

Tuyển tập toán

Bài Cho2n số thực đôi khác nhaua1, a2, , an;b1, b2, , bn Viết số vào bảngn×n

như sau: Ơ(i;j) (hàngi cột j) số(ai+bj) Chứng minh tích tất số

các cột tích tất số hàng Lời giải

Tích số cột thứ j

πj = (bj+a1)(bj+a2) (bj+an)

Khi đóπi =πj với i, j = 1,2, , n xét đa thức P(x) = (x+a1)(x+a2) (x+an) Ta suy P(b1) = P(b2) = =P(bn) = C

trong C số Ta lại xét đa thức G(x) =P(x)−C

Do G(x) có n nghiệmb1, b2, , bn nên đa thức bậc n với hệ số cao Từ suy G(x) = (x−b1)(x−b2) (x−bn)

Vậy

(x+a1)(x+a2) (x+an)−C = (x−b1)(x−b2) (x−bn)

Thay x=−aj(j = 1,2, , n) vào đẳng thức cuối ta

−C= (−aj −b1)(−aj−b2) (−aj −bn) = (−1)n.(aj +b1) (aj +bn)

hay

(aj+b1) (aj+bn) = (−1)n+1C

Vế trái đẳng thức cuối tích tất số thuộc hàng j Bài toán chứng minh Bài Một bọ vị trí có tọa độx= trục số Ở bước, từ vị trí có tọa độx=a, bọ nhảy đến vị trí có tọa độ x = a+ x = a

2 Chứng minh có tất

Fn+4−(n+ 4) vị trí khác (kể vị trí ban đầu) mà bọ nhảy đến với khơng q n bước nhảy, (Fn) dãy Fibonacci cho F0 = F1 = 1, Fn = Fn−1+Fn−2 với n >2

Lời giải

ĐặtM tập hợp số thực có dạng a

2b với a nguyên dương lẻ, n nguyên không âm Dễ thấy

(20)

để nhảy từ 1đến vị tríx Rõ ràng f(x)xác định hữu hạn nhận thấy với x0 = a

2b ∈M

sau a−1

2 bước

00+ 200, từ 1 con bọ nhảy đến a Tiếp theo, sau b bước nhảy 00/200, bọ có

thể đếnx0 nên số bước không vượt a−1

2 +b Với số nguyên dương n, đặt

Pn ={x|x∈M ∧f(x) = n}, Qn=

n

x

x∈Pn∧

x

2 ∈Pn+1

o

vàRn=Pn\Qn

Dễ thấy Qn tập hợp số mà x

2 phải đến bước thứ n+ thu được; Rn tập hợp

các số mà x

2 ∈ Pn Với cách đặt đó, ta thấy số lượng vị trí khác mà bọ

nhảy đến saun bước |P0|+|P1|+· · ·+|Pn| Ta tìm cơng thức cho Pn

Ta có nhận xét rằng, từ1→xta cần n bước để 1→x+ 2,ta cần n+ 1bước; ngồi ra, từ 1→ x

2 ta cần n+ bước để → x, ta cầnn bước Do đó, với x ∈M mà f(x) = n

f(x+ 2) =n+ Vì nên ta có

|Pn+1|=|Pn|+|Qn|,∀n∈Z+

Tiếp theo, ta chứng minh x∈Pn ⇔x+ 4∈Rn+2 Thật vậy, xét x0 ∈Pn x0+ 4∈Pn+2, nhiên x0+

2 =

x0

2 + số thu n+ bước nên x0+ ∈Rn+2 Ngược lại,

nếu x0+ 4∈Rn+2 tương tự x0

2 + 2∈Pn+2 ⇒

x0

2 ∈Pn+1 ⇒x0 ∈Pn

Từ suy |Pn|=|Rn+2| Do đó, ta có quan hệ truy hồi

|Pn|=|Qn|+|Rn|=|Pn+1| − |Pn|+|Pn−2| ⇒ |Pn+1|= 2|Pn| − |Pn−2|, n >2

Chú ý |P0| = 1,|P1| = 2,|P2| = nên đặt un = |Pn+1| − |Pn| un = un−1 +un−2 u0 = 1, u1 = nên dễ thấy un = Fn+2 với (Fn) dãy Fibonacci Do đó, ta đưa

|Pn|=Fn+2−1 với mọin >0 Cuối cùng, yêu cầu toán tương đương với việc chứng minh

n

X

k=0

(Fk+2−1) =Fn+4−(n+ 4)

Tuy nhiên, điều dễ dàng thực quy nạp với ý

1 Khi thay n→n+ 1, vế trái tăng lên Fn+3−1đơn vị

2 Khi thay n→n+ 1, vế phải thay đổi(Fn+5−(n+ 5))−(Fn+4−(n+ 4)) =Fn+3−1

Vậy tốn giải hồn tồn

Nhận xét Đây toán đẹp ứng dụng phép đếm cơng thức truy hồi Nó gây khơng khó khăn phải kiểm sốt số lần nhảy bọ cần thực để đến số a

2b

Mấu chốt toán nhận xét: “hai bước +2 bước /2 ” quy đổi “một bước /2

(21)

1 (Dãy Farey) Cho số nguyên dương n, xét dãy tăng phân số có dạng p

q ∈[0; 1] với p, q ∈N 0< q 6n Khi đó, dãy có độ dài bằng1 +

n

X

k=1 ϕ(k)

Ví dụ Khi n = 4, ta có dãy

1 → → → → → → 1

2 (APMO 2015) Cho (an) dãy số xác định a0 ∈ Z+ an+1 tính an+1+ =

2(an+ 1) an+1+ =

2(an+ 1) an+

Hỏi tồn k nguyên dương đểak= 2014 giá trị

nhỏ k mấy?

3 (Định lý Lamé – Knuth) Với n nguyên dương, gọi u > v > số nguyên cho thuật toán Euclid áp dụng trên(u, v) kết thúc sau đúngn bước; ra,u số nhỏ thỏa mãn (u, v) = (Fn+2, Fn+1) với (Fn)là dãy Fibonacci

4 (Tài liệu đội tuyển IMO 2011) Có trị chơi truyền hình mà người chơi phải trả lời n câu hỏi Nếu trả lời cộng1điểm, sai bị chia đôi số điểm có Biết ban đầu người chơi có điểm Gọi Sn tập hợp điểm số mà người chơi nhận sau

khi trị chơi kết thúc Chứng minh rằng|Sn|=Fn+3−1 Ví dụ Khi n = S =

0,1,1

2,2

Bài Cho bảng(n2+n+ 1)×(n2+n+ 1) Ta điền vào số bảng số 0hoặc số 1, sao cho khơng có bốn ô ghi số đỉnh hình chữ nhật Chứng minh số số không vượt (n+ 1) (n2+n+ 1)

Lời giải

Gọi xi số số 1ở hàng thứ i Ta cần chứng minh

S =

n2+n+1

X

i=1

xi 6(n+ 1) n2+n+

Gọi tậpM gồm cặp(k;l) mà 16k < l6n2 +n+ 1, ta có |M|=

n2+n+ 1

2

Với mỗii= 1,2, , n2+n+ 1 ta xét tập M

i gồm cặp(k;l) mà hai ô giao cột k cột l với

hàng thứ i chứa số Ta có |Mi| =

xi

= xi(xi−1)

2 Vì khơng có ghi số đỉnh

hình chữ nhật nên Mi∩Mj =∅,∀i6=j Do

n2+n+1

X

i=1

|Mi|6|M|, hay

1

n

X

i=1

x2i −xi

6 (n

2+n) (n2+n+ 1)

2 ⇔

n2+n+1

X

i=1

x2i −

n2+n+1

X

i=1

xi n2+n

n2+n+

Mặt khác

n2+n+1

X

i=1

x2i > n2+n+ 1

n2+n+1

X

i=1 xi

!2

nên ta có

1

n2+n+ 1S 2−

S 6 n2+n n2+n+ ⇔S2 − n2+n+ 1S− n2 +n n2+n+ 12 60

⇒S6(n+ 1) n2+n+

(22)

Bài Cho n >5 điểm nằm mặt phẳng cho khơng có điểm thẳng hàng Tí Tèo chơi trò chơi sau: Hai bạn luân phiên kẻ đoạn thẳng nối hai điểm số điểm cho; lượt mình, bạn kẻ đoạn thẳng khơng có bạn kẻ lại đoạn thẳng kẻ trước Nếu sau lượt kẻ bạn mà điểm, n điểm cho, đầu mút đoạn thẳng (trong số đoạn thẳng kẻ) bạn coi thắng Giả sử Tí người kẻ đoạn thẳng Hãy tìm tất số nguyên n > cho Tí có cách chơi để thắng cuộc, cho dù Tèo có thực lượt kẻ

Lời giải

Ta gọi điểm điểm lập điểm khơng nối với điểm khác số n−1điểm cịn lại Dễ thấy, Tí người thắng trước lượt kẻ cuối Tí, mặt phẳng cịn lại điểm lập Rõ ràng, Tèo buộc phải chấp nhận để Tí có điều trước lượt kẻ cuối Tí trước lượt kẻ Tèo, mặt phẳng cịn đúng3điểm lập n−3điểm cịn lại đơi nối với đoạn thẳng Vì Tí người thực việc kẻ nên điều vừa nêu xảy số đoạn thẳng có hai đầu mút nằm sốn−3điểm số lẻ; nghĩa

(n−3)(n−4)

phải số lẻ Dễ thấy, điều vừa nêu có khincó dạng4k+ 1hoặc4k+ 2, vớiklà số ngun dương Như vậy, Tí có cách chơi đảm bảo chắn thắng khin = 4k+

hoặc n= 4k+ 2, k∈N∗ Ngược lại xét n có dạng vừa nêu trên, ta có:

Trường hợp 1.n = 4k+ 1, k ∈N∗. Với k = 1, hiển nhiên Tí người thắng Xét k

>2 Dễ thấy, sau lượt kẻ Tí Tèo, số điểm lập (trong số n điểm cho) giữ nguyên, giảm 1hoặc giảm Xét cách chơi sau Tí: Ở lượt đầu tiên, Tí kẻ đoạn thẳng Tiếp theo, số điểm cô lập mặt phẳng lớn hơn3 vào thời điểm Tèo thực lượt kẻ mình, sau lượt kẻ Tèo, số điểm cô lập giảm đis (điểm), s∈ {0; 1; 2}, Tí kẻ đoạn thẳng cho số điểm cô lập giảm 2−s, cách: Nếu s = (tức Tèo kẻ đoạn thẳng nối hai điểm khơng lập) Tí kẻ đoạn thẳng nối hai điểm cô lập tùy ý;

Nếus= (tức Tèo kẻ đoạn thẳng nối điểm lập với điểm khơng lập) Tí kẻ đoạn thẳng nối điểm cô lập với điểm không cô lập;

Nếus = 2(tức Tèo kẻ đoạn thẳng nối hai điểm cô lập) Tí kẻ đoạn thẳng nối hai điểm khơng lập

Tí thực việc kẻ số điểm cô lập mặt phẳng lớn hơn3vào thời điểm Tèo thực lượt kẻ nối hai điểm lập với nhau, Tèo tạo điểm không cô lập mới, mà điểm nối với điểm số điểm khơng lập có mặt phẳng sau Tèo thực lượt kẻ Với cách chơi nêu Tí, sau lượt kẻ Tí, sau cặp lượt kẻ Tèo-Tí, số điểm lập giảm

(23)

thời điểm mà sau lượt kẻ Tí, tất cản−3 điểm khơng lập đơi nối với Điều này, theo nhận xét phần đầu lời giải, buộc Tèo phải nhường phần thắng cho Tí Trường hợp n = 4k+ 2, k ∈ N∗ Với k = 1, dễ thấy, Tí người thắng Xét k

> Khi đó, sau lượt kẻ Tí, xảy hai khả sau:

• Khả 1.Tèo kẻ đoạn thẳng nối hai điểm không cô lập với điểm cô lập

• Khả Tèo kẻ đoạn thẳng nối hai điểm cô lập với

Nếu khả xảy ra, Tí kẻ đoạn thẳng nối hai điểm không cô lập với nhau; khả xảy ra, Tí kẻ đoạn thẳng nối điểm không cô lập với điểm cô lập Bằng cách này, Tí làm cho số điểm cô lập bị giảm số lẻ (3 5) điểm, sau lượt kẻ (gồm lượt Tí lượt Tèo) Vì thế, số điểm cô lập ban đầu số chẵn nên cách kẻ nêu trên, Tí buộc Tèo phải thực lượt kẻ trạng thái số điểm cô lập mặt phẳng số lẻ lớn Điều này, với việc số đoạn thẳng có hai đầu mút nằm số n−3 điểm số lẻ ((2k−1)·(4k−1)), hiển nhiên, giúp Tí dùng cách chơi mơ tả trường hợp để trở thành người thắng

Tóm lại, n= 4k+ n = 4k+ 2,k ∈N∗, Tí có cách chơi đảm bảo chắn thắng Vậy, tất giá trịn cần tìm theo yêu cầu đề n = 4k+ n = 4k+ 2,k ∈N∗.

Bài Trong bảng vng kích thước 999×999, tơ hai màu trắng đỏ Gọi T số (C1, C2, C3) ô mà hai ô đầu hàng hai ô cuối cột, với C1 C3 màu trắng, C2 màu đỏ Tìm giá trị lớn củaT Lời giải

Ta chứng minh bảng vng n×n có nhiều 4n

27 Giả sử hàngi cột j

chứa bj ô trắng tương ứng, R tập hợp ô đỏ Với ô đỏ (i, j) có aibj (C1, C2, C3) chấp nhận với C2 = (i, j), vậyT =

X

(i,j)∈R aibj

Sử dụng bất đẳng thức2ab6a2+b2 ta được

T 6

2

X

(i,j)∈R

a2i +b2j=

n

X

i=1

(n−ai)a2i +

n

X

j=1

(n−bj)b2j

Do cón−ai đỏ hàng ivà n−bj ô đỏ cột j Áp dụng bất đẳng thứcAM˘GM ta có

(n−x)x2 =

2(2n−2x).x.x6

2n

3

3

= 4n

3

27

Đẳng thức xảy x= 2n

Do T 6 n

2 4n3

27 +

n

2 4n3

27 = 4n4

27 Nếun = 999 tơ màu tuỳ ý bảng ô vuông vớix= 2n

3 = 666ô

trắng hàng cột Chẳng hạn tô màu ô (i, j) trắng i−j ≡1,2, ,666 (mod999), ô khác màu đỏ Vậy giá trị lớn T đạt T = 4.999

4

27

(24)

số nhiều số số Lời giải

Nhận xét "Số tự nhiênn tổng chữ số củan số dư phép chia cho 9" Thật vậy, giả sử n=akak−1 a1a0 Tức n =a0+a1.10 +a2102+ +ak10k, k∈N Khi

n ≡a0+a1.1 +a212+ +ak1k ≡a0+a1+a2+ +ak(mod 9)

Mặt khác ta có

1 21 ≡2 (mod9);

2 22 ≡4 (mod9);

3 23 ≡8 (mod9);

4 24 ≡7 (mod9);

5 25 ≡5 (mod9);

6 26 ≡1 (mod9);

7 27 ≡2 (mod9).

Như tính bất biến là26k+rlần lượt nhận số dư phép chia cho 2, 4, 8, 7, 5, tương ứng với giá trị củar 1, 2, 3, 4, 5, Dãy cuối nhận gồm 2018 số thuộc tập hợp {2; 4; 8; 7; 5; 1} Ta có 2018 = 336×6 + số cuối dãy sau thực bước biến đổi Như dãy cuối nhận có 337 số (nhiều số số khác số) Vậy số

các số nhiều số số số

Bài (USAMO 2001) Cho S tập số nguyên cho: i) Tồn tạia, b∈S với gcd (a, b) = gcd (a−2, b−2) = 1;

ii) Nếu xvà y hai phần tử S (có thể nhau) x2−y cũng thuộcS. Chứng minh rằngS tập tất số nguyên

Lời giải

Ta nói "S ổn định x7→f(x)” x∈S f(x)∈S.Nếu c, d∈S, theo (ii), tập S ổn định x7→c2−xvà x7→d2−x, ổn định tại x7→c2−(d2−x) = x+ (c2−d2). Tương tự ta cóS ổn định tạix7→x+ (d2−c2) Do đóS ổn định tạix7→x+n vàx7→x−nvới mọin số nguyên có dạng c2−d2 với c, d∈S Khi đó, ta cho n =m với m = gcd (c2−d2 :∀c;d∈S) Thật vậy, ta đặtP ={c2−d2 :∀c;d ∈S}khi đó, theo tính chất số học đơn giản (Định lý Euler), tồn dãy số nguyênxk để

X

ak∈P

xkak =m Từ nhận xét trên, ta thấy cóx7→x±n;x7→x±m

thì cóx7→x±m±n Vì ta có

x7→x±ak;∀ak ∈P ⇒x7→x±

X

ak∈P

xkak ⇒x7→x±m

Vậy nhận xét Ta chứng minh m = Phản chứng, giả sử m 6= Gọi p mộtước nguyên tố m Khi c2 −d2 ≡ (mod p) với c, d∈ S Nói cách khác, với c, d∈ S, ta d ≡ c (mod p) d ≡ −c (mod p) Với c ∈ S, theo (b) ta có c2 −c ∈ S, nên c2 −c ≡ c

(mod p) c2−c≡ −c (mod p) Do với mỗi c∈S, ta có

(25)

Từ (i), tồn a b trongS chogcd (a, b) = 1, từ ta hai số a b chia hết chop.Điều có nghĩa có phần tửα củaS mà α6= (mod p) Tương tự, theo (i),gcd (a−2, b−2) = 1, nên p chia hết a−2 b−2 Do tồn phần tửβ S cho β6= (mod p) Theo(∗), ta suy raα ≡2 (mod p)và β ≡0 (mod p) Theo (ii), β2−α ∈ S Chọn c = β2 −α, ta −2 ≡ (mod p) −2 ≡ (mod p) cho p = Từ (∗) ta thấy tất phần tử S chẵn, trái với (i) Do giả thiết sai,

chứng tỏm = ta S tập tất số nguyên

Bài Cho bảng vng kích thước 111×2017 (bảng gồm 111 hàng 2017 cột) Điền vào ô vuông bảng số cho cột có 85 số Chứng minh ta bỏ 106 hàng cho bảng nhận (gồm5 hàng 2017 cột), có tối đa cột chứa toàn số

Lời giải

Đánh số thứ tự hàng, từ lên trên, 1, 2, , 111 đánh số thứ tự cột, từ trái qua phải, bởi1, 2, , 2017 Với i∈

1; 2; .; 111 , kí hiệu ri số số1 nằm hàng thứ i Với k∈

1; 2; .; 2017 , kí hiệu ck số số 1nằm cột thứ k Hiển nhiên, ta có

111

X

i=1 ri =

2017

X

k=1

ck (*)

Do ck >85với k ∈

1; 2; ,2017 nên 2017

X

k=1

ck>85×2017 = 171.445 Do đó, từ (∗), ta có

111

X

i=1

ri >171.445

Suy ra, theo nguyên lý Dirichlet, tồn hàng mà số số hàng khơng

171.445 111

= 1.545 (số)

Tô màu xanh cho hàng Vì tính chất bảng số cho khơng thay đổi hốn đổi vị trí cột nên khơng tính tổng qt, giả sử tất 1.545 ô (kể từ trái sang phải) hàng tô xanh chứa số Xét bảng A bảng cho, gồm

111 hàng 472 (= 2017−1545) cột cuối bảng cho (như hình bên)

· · · · · · · · · · · ·

2017 cột

472 cột Hiển nhiên, bảng A có tính chất bảng cho, nghĩa là, cột bảng A có 85 số1 Vì thế, lập luận tương tự trên, suy bảng A, tồn hàng mà số số1 hàng khơng

85×472 111

= 362(số)

(26)

đến ô thứ 2017 hàng xanh, màu đỏ Tương tự trên, không tổng quát, giả sử tất 362

ô hàng tô đỏ, xét bảngA, chứa số Xét bảng B bảng cho, gồm 111 hàng 110 (= 472−362) cột cuối bảng cho Cũng với bảng A, suy bảng B, tồn hàng mà số số1 hàng khơng

85×110 111

= 85(số)

(Hàng phần hàng có chứa tơ màu) Tơ hàng vừa nêu bảngB màu hồng; trường hợp hàng phần hàng chứa tô màu, ta tô lại 110 ô, từ ô thứ 1908 đến ô thứ 2017, màu hồng Tương tự trên, khơng tính tổng qt, giả sử tất cả85ô hàng tô hồng, xét bảng B, chứa số1 Xét bảng C bảng cho, gồm 111 hàng và25(= 110−85) cột cuối bảng cho Cũng với bảng A, suy bảng C, tồn hàng mà số số1 hàng khơng

85×25 111

= 20(số)

(Hàng phần hàng có chứa ô tô màu) Tô hàng vừa nêu bảngC màu tím; trường hợp hàng phần hàng tơ màu, ta tô lại25ô, từ ô thứ 1934đến ô thứ 2017, màu tím Tương tự trên, khơng tính tổng quát, giả sử tất

20ô hàng tơ tím, xét bảng C, chứa số

Xét bảng D bảng cho, gồm 111 hàng (= 25−20) cột cuối bảng cho Cũng với bảng A, suy bảng D, tồn hàng mà số số1 hàng khơng

85×5 111

= 4(số)

(Hàng phần hàng có chứa ô tô màu) Tô hàng vừa nêu bảng D màu vàng; trường hợp hàng phần hàng có chứa tô màu, ta tô lại5ô, từ ô thứ2013đến ô thứ2017, màu vàng Như vậy, ta có tối đa5hàng, mà hàng chứa ô tô màu, đó, có tối thiểu 106 hàng, mà hàng không chứa ô tô màu Bỏ 106hàng vừa nêu, ta có bảng gồm5 hàng và2017cột, mà số cột có chứa số khơng tổng số số nằm ô tô màu Theo ,số số1 nằm ô tô màu xanh, đỏ, hồng, tím, vàng tương ứng khơng 1.545,362, 85, 20,4 Vì thế, tổng số số nằm tơ màu khơng hơn:

1.545 = 362 + 85 + 20 + = 2016(số)

Từ suy ra, bảng gồm5hàng 2017cột vừa nêu có tối đa cột khơng chứa số

1, hay nói cách khác, chứa tồn số Vậy toán chứng minh

Lưu ý.Với xlà số thực, dxe kí hiệu số nguyên nhỏ không nhỏ x Bài Chom,nlà số nguyên lớn hơn1,Slà tập gồmnphần tử Giả sửA1, A2, , Am

là tập S cho với x, y ∈ S có tập Ai x ∈ Ai y6∈Ai x6∈Ai y∈Ai Chứng minh n62m

Lời giải

Gọi T tập tất xâu nhị phân có m ký tự Xét tương ứng từ S đến T sau: Với x ∈ S, đặt f(x) = x1x2 xm, xi = x ∈Ai xi = x 6∈ Ai Khi hiển

(27)

Bài 10 Cho đa giác lồi P Bạn An muốn ghi vào đỉnh P số nguyên dương cho điều kiện sau đồng thời thỏa mãn

i) Trong số ghi, có số chẵn;

ii) Tổng ba số ghi ba đỉnh liên tiếp tùy ý số lẻ

Chứng minh bạn An thực cách ghi số đỉnh P chia hết cho3

Lời giải

Điều kiện cần Giả sử bạn An thực cách ghi thỏa mãn đề Xuất phát từ đỉnh đó, lần lượt, theo chiều kim đồng hồ, kí hiệu đỉnh P A1, A2,· · ·, An Với i∈ {1,2, , n}, kí hiệu số bạn An ghi vào đỉnh Ai

Theo i), sốa1, a2,· · · , an có số chẵn Khơng tính tổng qt, giả sử a1 chẵn

Trong phần trình bày sau ta coi sốn+k, 16k 6n, số k (các số xét theo modulon) Nhận xét:Với mọii, j ∈ {1; 2; 3; .;n}, nếu(j−i)chia hết cho3 thìai vàaj có

cùng tính chẵn lẻ Thật vậy, vớii∈ {1; 2; 3; .;n}theo ii) ta cóai+ai+1+ai+2 ai+1+ai+2+ai+3 số lẻ nên(ai+1+ai+2+ai+3)−(ai+ai+1+ai+2) = ai+3−ai số chẵn Vì thếai ai+3 có tính chẵn lẻ Từ đó, i tùy ý nên ta có nhận xét Xét trường hợp sau

1 Nếun= 3k+ 1, k∈N∗ Trong trường hợp vìa1 chẵn nên theo nhận xét a3k+1 số chẵn, lại có a3k+1+a1+a2 số lẻ, suy a2 số lẻ Do theo nhận xét a3k+2 ≡a1 số lẻ, ta nhận mâu thuẫn, chứng tỏ n6= 3k+

2 Nếun= 3k+ 2, k∈N∗ Trong trường hợp vìa

1 chẵn nên theo nhận xét a3k+1 số chẵn, lại cóa3k+1+a3k+2+a1 số lẻ, suy raa3k+2 số lẻ Do theo nhận xét,a2 số lẻ, suy a3k+2+a1+a2 số chẵn, trái với ii), ta nhận mâu thuẫn, chứng tỏ n6= 3k+ Vì trường hợp khơng xảy nên phải có n chia hết cho3

Điều kiện đủ Giả sử P có n = 3k đỉnh (k ∈N∗) Xuất phát từ đỉnh đó, lần lượt, theo chiều kim đồng hồ, kí hiệu đỉnh P làA1, A2, , A3k Xét cách ghi số sau bạn An:

Với i ∈ {1; 2; 3; .; 3k}, ghi vào đỉnh Ai số nguyên dương lẻ i ≡2 (mod3) ghi vào

đỉnh Ai số nguyên dương chẵn trường hợp lại Rõ ràng cách ghi thỏa mãn

điều kiện i) đề Don ≡0 (mod3)nên ba đỉnh liên tiếp tùy ý P ln có đỉnh có số chia hết cho3 dư1, đỉnh có số chia cho 3dư 2và đỉnh có số chia hết cho

3 Vì thế, với cách ghi số nêu ta ln có tổng ba số ghi ở3 đỉnh liên tiếp củaP số lẻ (do tổng số lẻ hai số chẵn) Vì cách ghi số thỏa mãn điều kiện

ii) đề

Bài 11 Từ số 1,2,3,4,5 lập số tự nhiên có n chữ số (n nguyên dương) mà số chứa số lẻ chữ số 1và số chẵn chữ số 2?

Lời giải

Vì tổng số lượng số1 số 2trong số lẻ nên với số k lẻ cố định, ta tính xem có số tự nhiên có n chữ số (n >k) có k chữ số

Chọn k vị trí n vị trí để đặt chữ số và2, có

n k

cách

Chọn ivị trí k vị trí chọn (i lẻ vài6k) để đặt chữ số 1và k−i vị trí để đặt chữ số

2,có số cách làm

X

i6k ilẻ

k i

=

k

1

+

k

3

(28)

Trongn−k vị trí cịn lại, vị trí đặt ba chữ số3,4,5có3n−k cách Theo quy tắc nhân, có n k

·2k−1·3n−k số tự nhiên mà cók chữ số Vậy số số tự nhiên thỏa đề

X

k6n klẻ

n k

·2k−1·3n−k = ·

X

k6n

n k

·2k·3n−k = ·

(3 + 2)n−(3−2)n

= n−1

4

Bài toán giải

Bài 12 Mỗi vng bảng n×n ta ghi số0 số 1sao cho, với ghi số 0thì có n hàng cột với ghi số1 Chứng minh có

n2

2

số

1 ghi

Lời giải

Với n= ta có cách điền sau

0 1 0 1 1

Trong trường hợp tổng quát, ta xây dựng đồ thị hai phía gồm2n đỉnh đỉnh bên trái n hàng n đỉnh bên phải n cột Đỉnh Hi nối với đỉnh Cj ô (i;j) ghi số Theo giả thiết

đề bài, đỉnh Hi không nối với đỉnh Cj d(Hi) +d(Cj)>n Trong đód(Hi)là số số hàng i d(Cj)là số số cộtj Ta chứng minh số cạnh đồ thị làe>

n2

2 Ta có

S = X

(i;j)=0

(d(Hi) +d(Cj))> n2−e

n

Trong tổng với i, số hạng d(Hi) xuất n−d(Hi) lần với j số hạng d(Cj)

xuất hiệnn−d(Cj)lần Mà n

X

i=1

d(Hi) = n

X

j=1

d(Cj) = e nên ta suy

S = n

X

i=1

d(Hi) [n−d(Hi)] + n

X

j=1

d(Cj) [n−d(Cj)] = 2ne− n

X

i=1

d2(Hi)− n

X

j=1

d2(Cj)

Áp dụng bất đẳng thức Cauchy – Schwarz ta có

n

X

i=1

d2(Hi)>

n n

X

i=1

d(Hi)

!2 = ne và n X j=1

d2(Cj)>

n n

X

j=1 d(Cj)

!2

= e

2

(29)

Suy S 62ne− 2e

2

n Từ dẫn đến

n2−en62ne−2e

2

n ⇔2e 2−

3n2e+n4 60⇔ n

2

2 6e 6n

2

Bài toán giải

Bài 13 Tìm số ngun n >3 nhỏ có tính chất: Với cách tô n điểm A1, A2, , An,

đôi phân biệt theo thứ tự nằm cách đường thẳng, hai màu, tồn ba điểm Ai, Aj, A2j−i(16i <2j−i6n) tô màu

Lời giải

Giả sử điểm tô hai màu, đen trắng Xét điểmA1, A2, , A8,được tô màu sau: A1, A2, A5, A−6được tô màu trắng;A3, A4, A7, A8 tô màu đen Dễ thấy, khơng có ba điểm Ai, Aj, A2j−i(16 i < 2j −i 8) tơ màu Do n > Ta chứng minh n =

số có tính chất đề u cầu Thật vậy, giả sử ngược lại, tồn cách tô điểmA1, A2, , A9 hai màu, đen trắng cho khơng có ba điểm Ai, Aj, A2j−i(1 6i < 2j −i 69) tô

cùng màu

Trường hợp Tồn i∈ {3; 4; 5},sao cho điểmAi Ai+2 tô màu, giả sử màu trắng Khi đó, điểm Ai−2, Ai+1, Ai+4 phải tơ màu đen (chú ý i−2>1và i+ 69), vô lý

Trường hợp Với i∈ {3; 4; 5},các điểm Ai Ai+2 tơ khác màu Khơng tính tổng quát, giả sử A5 tô màu trắng Khi đó, A3 A7 tơ màu đen Nhờ tính đối xứng, ta giả sử A4 tơ màu trắng vàA6 tơ màu đen Khi A8 tô màu trắng (do A6, A7 A8 không tô màu); suy ra, A2 tô đen (do A2, A5 A8 không tô màu) A9 tô trắng (do A7, A8 vàA9 khơng tơ màu) Do đó,A1 phải vừa tô màu trắng (do A1, A2 A4 không tô màu),vừa tô đen (do A1, A5 A9 không tô màu), điều vô lý

Những điều vơ lí nhận nhận cho ta điều muốn chứng minh Vậy tóm lại, n= số

ngun nhỏ có tính chất đề yêu cầu

Bài 14 (VN TST 2003) Cho bốn điểm phân biệt A(0,0), B(p,0), C(m, q), D(m, n) với m, n, p, q bốn số nguyên dương thỏa mãn p < m, q < n Xét đường tốt f từ A đến D đường tốt g từ B đến C Gọi S số cặp đường đi(f, g)sao cho chúng khơng có điểm chung Chứng minh

S =

m+n n

m+q−p q

m+q q

m+n−p n

Lời giải

Bổ đề Số đường đif tốt

m+n n

, số đường g tốt

m+q−p q

(30)

x y

A B

C D

Do đó, số cặp đường tốt(f, g)tùy ý

m+n n

m+q−p q

Ta thấy hai đường f, g gặp điểm K thay xét cặp đường gấp khúcA → K → D, B →K →C, ta thay đổi hướng thành A → K → C, B → K → D Từ đó, dễ dàng chứng minh tồn song ánh từ cặp đường đi(f, g)có điểm chung đến cặp đường (f0, g0) từA →C, B →D Số đường

m+q q

m+n−p n

Vậy số cặp đường đi(f, g)thỏa mãn đề

m+n n

m+q−p q

m+q q

m+n−p n

Bài toán giải

Bài 15 (IMO 2005) Trong thi tốn cón thí sinh (n∈N, n >5) 6bài toán đặt Biết rằng, với hai tốn có nhiều

5 tổng số thí sinh giải

khơng có thí sinh giải tốn Chứng minh có hai thí sinh cho người giải đúng5 toán

Lời giải

Với 6 i 6 6,1 6 j 6 n, đặt aij = người j giải toán i aij = người j

khơng giải bàii Khi ta có ma trận A= (aij)cỡ6×nmà cột có nhiều số 1, hai hàng có nhiều

5n cột chung (cột chung cột mà phần tử hai hàng

trên cột số 1) Ta chứng minh toán phản chứng cách đếm số cặp số1

như Giả sử có nhiều thí sinh giải được5 tốn, cịn lại giải khơng Khi đóA có nhiều cột có năm số cột cịn lại cịn lại có nhiều 4số Vậy,

T 6(n−1)

4

+

5

= 6n+ (2.1)

Giả sử n= 5p+k,k = 0,1,2,3,4 Do hàng A có nhiều

5n cặp số chung nên

Vớik = 0thì với hai có khơng hơn2p+ 1thí sinh giải hayT >15(2p+ 1). Điều mâu thuẫn với (2.1)

(31)

NếuT < 6n+ có cột A có3 số Khi đó,

T 6(n−2)

4

+

5

+

3

= 6n+ = 30p+ 13<30p+ 15, (vô lý)

Vậy T = 30p+ 16 Khi có 2bài có 2p+ thí sinh giải được, cịn với hai cịn lại có

2p+ thí sinh giải Như vậy, có người giải được5bài n−1người giải 4bài Không tính tổng qt ta coi hai có 2p+ thí sinh giải 1và

và thí sinh giải thí sinh n Gọi B ma trận có từ A bỏ cột n Ta có cột củaB có 4số Ta tính số số hàng B

Trường hợp Thí sinh hai Giả sử thí sinh n khơng làm bài6 Số cặp số chung hai hàng(1; 2),(1; 6),(2; 6),(3; 6),(4; 6),(5; 6) là2p+ 1, số cặp số chung hai hàng cịn lại 2p Do số hàng B có 3cặp số 1nằm ở3trong số 5cặp hàng(1; 2); (1; 3),(1; 4),(1; 5),(1; 6) nên số số hàng 10p+

3 Tương tự ta có số số hàng2,3,4,5,6 lần

lượt 10p+

3 ;

10p+

3 ;

10p+

3 ;

10p+

3 ;

10p+

3 Từ kết suy

số 10p+ 1; 10p+ 2; 10p+ chia hết cho3 Vô lí

Trường hợp Thí sinh n khơng làm hai Giả sử thí sinh n khơng làm Tương tự trường hợp ta tính số số hàng ma trận B

10p+

3 ;

10p+

3 ;

10p+

3 ;

10p+

3 ;

10p+

3 ;

10p+

3

Suy số 10p+ 1; 10p+ 2; 10p+ chia hết cho Vơ lí

Bài tốn giải

Bài 16 Cho số nguyên dương lẻ n Người ta dùng màu để tô tất đỉnh mộtn-giác cho đỉnh tô màu màu dùng để tô số lẻ đỉnh Chứng minh tồn tam giác cân có đỉnh tơ màu (Một tam giác coi tam giác cân)

Lời giải

(32)

B hai đỉnh tùy ý khác màu n-giác đoạn thẳng AB cho ta cặp dạng (∗) cặp dạng (∗) Do đó, gọi p, q r (p, q, r ∈ N) tương ứng số cặp đỉnh tô hai màu (1,2), số cặp đỉnh tô hai màu (2,3) số cặp đỉnh tô hai màu (3,1), mà cặp đỉnh cho cặp dạng (∗) Khi số cặp dạng (∗) tính

3[(ab−p) + (bc−q) + (ca−r)] +p+q+r= 3(ab+bc+ca)−2(p+q+r), (2) ab cặp đỉnh tô hai màu (1,2), bc cặp đỉnh tô hai màu (2,3), ca cặp đỉnh tô hai màu (3,1) Từ (1) (2), ta được:

3(ab+bc+ca)−2(p+q+r) = 2s

Suy raab+bc+calà số chẵn, trái với giả thiết a,b,c số lẻ Mâu thuẫn cho ta thấy điều giả sử ban đầu sai hay ln tồn tam giác có đỉnh tơ màu Bài tốn

chứng minh xong

Bài 17 Trên bảng có dãy100 số ngun Hai bạn An Bình chơi trò chơi theo luật sau: An chọn dãy dãy bảng (gồm phần tử) sau Bình phép tăng1hay giảm1với tất số dãy mà An chọn Sau đến lượt An chọn dãy, tiếp tục Nếu bảng có từ98số chia hết cho4 An thắng Chứng minh An có chiến thuật để chiến thắng

Lời giải

Bổ đề Với số nguyên a, b, c, d theo thứ tự An ln có thuật tốn để xảy trường hợp sau, a≡0 (mod4)hoặc d ≡0 (mod4) b ≡c(mod4) (∗)

Chứng minh Thật An xử lí đểa, d đều lẻ.

Nếua d đồng dư với {1; 3} (mod4) An chọn dãy a, b, c, dsuy (*) Nếua≡d≡1 (mod4) Biến đổi b, c đến b, ckhác tính chẵn lẻ

Nếub−c≡1 (mod4) ta biến đổi dãy c, dthì

"

d≡0 (mod4) (−1)

b ≡c(mod4) (+1)

Nếub−c≡3 (mod4) lí luận tương tự

Trường hợp a≡d≡3 (mod4) ta lí luận tương tự Vậy bổ đề chứng minh

Ta chứng minh tốn tổng qt Vớin số An ln có thuật tốn để có (n−2)số chia hết cho Với n= Xét dãya, b, c Biến đổi để b chẵn vàa, c lẻ

Nếub chia hết cho4 thì xong.

Nếub chia4 dư2 Nếu a, c chia4 dư{1; 3}thì ta biến đổi dãy a, b, cthì xong Nếua, c chia4 dư1 thì ta biến đổi a, bvà b, c thì xong.

Nếua, c chia4 dư3 ta biến đổi tương tự Vậy n= ln có số chia hết cho4

Khơng tính tổng qt ta giả sử với dãy n−1số ln có (n−3)số chia hết cho Xét trường hợp dãy bảng x1;x2; ;xn Ta coi số x1;x2;x3; ;xn−1;xn (coi x3; ;xn−1 số đại diện x3) thực bổ đề

(33)

Nếu x2 ≡ x3(mod4) thì ta coi số x2;x3 thành số X Thực với dãy x1;X; ;xn ta (n−3) số chia hết cho thực chất (n−2) số chia hết cho Vậy ta có điều phải chứng minh

Bài tốn giải

Bài 18 Cho đa giác lồi, có 20 đỉnh Ghi tất số nguyên từ đến 20 vào đỉnh đa giác cho số ghi đỉnh đỉnh có số ghi

1 Hỏi ghi cho giá trị tuyệt đối hiệu hai số ghi hai đỉnh kề tùy ý lớn hơn4 đồng thời nhỏ 11 hay không?

2 Hỏi ghi cho giá trị tuyệt đối hiệu hai số ghi hai đỉnh kề tùy ý lớn hơn5 đồng thời nhỏ 11 hay không?

Lời giải

1 Câu trả lời “có thể”; chẳng hạn, ta xếp số hình bên

10 20 15 11 16

6

17

12

2

8 18

13

3

19 14

4

2 Giả sử ghi số theo yêu cầu đề Gọi X cách ghi Nếu cách ghiX, hai số a, b ghi hai đỉnh kề ta nói chúng nối với nhau, sợi chẳng hạn Khi đó, sợi nối thể cạnh đa giác cho Chia 20số thành nhóm A, B, C sau:

Nhóm A Nhóm B Nhóm C

1 11 16

2 12 17

3 13 18

4 14 19

5 10 15 20

(34)

giác) nên số nhóm B khơng thể nối với Xem số 6, ta thấy số khơng thể nối sang nhómA, lại khơng phép nối với số nhóm B nên nối với số nhómC; mà nhóm này, nối với số 16 Như vậy, số nối với tối đa số khác, điều vơ lí Điều vơ lí cho thấy giả sử sai

Vậy, không tồn cách ghi số thỏa mãn yêu cầu đề Bài 19 Trong mặt phẳng, cho đa giác lồi 2018 đỉnh, có độ dài cạnh 2018

2019 Chứng minh chu vi đa giác số chẵn ln tồn hai đỉnh chia đôi chu vi đa giác

Lời giải

Ta chứng minh tổng quát với số đỉnh đa giác là2n, vớin số ngun lớn hơn1nào Vì thế, ta đặt tên đỉnh đa giác cho, theo chiều kim đồng hồ, bởiA1,A2, , A2n

Với i = 1, 2,., 2n, kí hiệu xi độ dài cạnh AiAi+1 (quy ước A2n+1 A1) Vì độ dài cạnh

là2018 2019 nên xi ∈ {2018; 2019} với i= 1, 2, , 2n (1)

Vì chu vi đa giác cho số chẵn nên x1+x2 +· · ·+x2n = 2p, với p số nguyên

dương Với i= 1, 2, , n+ 1, đặt Si =xi+xi+1 +· · ·+xi+n−1 Khi đó, điều phải chứng minh theo yêu cầu đề tương đương với việc tồn số16i6n+ cho Si =p (2)

Hiển nhiên, với đa giác cho trước thỏa mãn điều kiện đề bài, xảy hai trường hợp sau

Trường hợp 1.S1 =p Khi đó, hiển nhiên ta có điều phải chứng minh.

Trường hợp 2. S1 6=p Khơng tính tổng qt, giả sử S1 < p (trường hợp S1 > p được xét tương tự)

Khi đó, S1+Sn+1 = (x1+x2+· · ·+xn) + (xn+1+xn+2+· · ·+x2n) = 2p nên Sn+1 > p Vì thế, S1,S2, , Sn+1 dãy số nguyên dương có số hạng đầu nhỏ p, số hạng cuối lớn pvà với

16 i 6 n, |Si+1−Si| =|xi+n−xi| ∈ {0; 1} (do (1)) Mà p số nguyên dương nên dãy

chắc chắn phải có số hạng bằngp Điều lý giải sau Ta coiS1,S2, , Sn+1 bậc cầu thang từ bậcS1 < p đến bậc Sn+1 > p mà |Si+1−Si|=|xi+n−xi| ∈ {0; 1},

với 16i6n, nghĩa lần bước leo lên tối đa bậc chắn ta phải bước chân lên tất bậc, từ bậc thứS1 đến bậc thứ Sn+1, cấm có bỏ qua bước nào, nên chắn có bậc bằngp Điều có nghĩa, tồn i, 1< i < n+ 1, cho Si =p Từ đó, theo (2), kết

luận chứng minh

Bài 20 Vào ngày giỗ Tổ năm Đinh Dậu, Vua Hùng dâng bánh chưng to Pi muốn xi phép Vua Hùng, cắt bánh chia cho đồng bào theo cách: Lần 1, cắt bánh thành

14hoặc20phần; từ lần thứ hai, lần cắt phần bánh tùy ý thành14hoặc20phần Hỏi, cách đó, Pi cắt bánh chưng thành 1! + 2! +· · ·+ 2016! + 2017! phần hay không? Lời giải

Nhận thấy, lần Pi cắt phần bánh thành 14 phần nhỏ tổng số bánh tăng thêm13 phần lần Pi cắt phần bánh thành20phần tổng số phần bánh tăng thêm

19phần Do đó, saum lần cắt bánh thành14phần n lần cắt bánh thành20phần (m,n số tự nhiên), Pi cắt bánh chưng thành1 + 13m+ 19n phần Như vậy, Pi cắt bánh chưng thành 1! + 2! +· · ·2016! + 2017! phần tồn số tự nhiênm,n cho

1 + 13m+ 19n= 1! + 2! +· · ·+ 2016! + 2017!, hay

(35)

Ta có

2! + 3! + 4! + 5! = 152 = 19·8,

6! + 8! = 6!·57 = 19·3·(6!),

7! + 9! + 10! = 7!·793 = 13·61·(7!),

11! + 12! = 13·(11!),

13! + 14! +· · ·+ 2016! + 2017! = 13·M, M số nguyên dương Từ đó, suy

2! +· · ·+ 2016! + 2017! = 13 (61·(7!) + 11! +M) + 19·(8 + 3·(6!))

Chọn m= 61·(7!) + 11! +M n= + 3·(6!), ta số tự nhiên m, n thỏa mãn (*) Vậy câu trả lời tốn “có thể”

Bài 21 Một bảng vng ABCD kích thước2018×2018 gồm 20182 vng đơn vị, vng đơn vị điền ba số−1; 0; 1.Một cách điền số gọi đối xứng có tâm đường chéo AC điền số −1 cặp ô đối xứng qua AC điền số 0hoặc1 Chứng minh với cách điền số đối xứng bất kì, ln tồn hai hàng có số vng đơn vị theo thứ tự từ trái sang phải làa1, a2, , a2018 hàng thứ nhất, b1, b2, , b2018 hàng thứ hai cho S =a1b1+a2b2+ a2018b2018 số chẵn

Lời giải

Bổ đề Trong nhóm2018người X1;X2; .;X2018, ln tồn hai người có số người quen chung nhóm số chẵn

Chứng minh Ta chứng minh bổ đề phản chứng Giả sử hai người nhóm có số người quen chung lẻ

Trường hợp 1: Tồn người có số người quen lẻ; giả sử X1 Khơng tính tổng qt, giả sử X1;X2; .;X1+k với k lẻ Áp dụng bổ đề bắt taym nhóm lẻ người X2;X3; X1+k ln tồn người có số người quen nhóm chẵn, giả sử làX2 Khi đóX1 X2 có số người quen chung chẵn, mẫu thuẫn Ta có điều phải chứng minh

Trường hợp 2: Tất người cso số người quen chẵn Gọi A tập người quen X1; B tập người X1 khơng quen Khi |A|+|B| = 2017 A chẵn, B lẻ Sử dụng giả thiết phản chúng bạn tỏng A có số người quen chung với X1 lẻ, với Xi ∈ A

bất kỳ có lẻ người quen A lẻ người quen B Lập luận tương tự Xj ∈ B

bất kỳ có lẻ người quen A lẻ người quen B.Gọi M số cặp (Xi;Xj) với Xi ∈ A, Xj ∈ B Xi quen Xj.Do Xi ∈ A có lẻ người quen B |A| chẵn,

nên M chẵn.Do Xj ∈B có lẻ người quen trongA |B| lẻ, nên M lẻ Suy điều

mâu thuẫn

Vậy bổ đề chứng minh

Quay toán.Ta gọi nij số điền ô vuông đơn vị hàngi cộtj (tính từ xuống

và trái sang) Từ giả thiết tốn ta có nij =−1,∀i = 1,2, ,2018 nij =nji ∈ {0; 1}∀i 6=j ∈

{1; 2; .; 2018}.Yêu cầu toán chứng minh tồn hai sốk;k0 ∈ {1; 2; .; 2018}.phân biệt cho S =

2018

X

i=1

nkink0i Do nkk =nk0k0 =−1 nk0k =nkk0 nên nkknk0k+nkk0nk0k0 = 2nkknk0k

Khi ta cần chứng minh S0 =

2018

X

i=1

i6=k,k0

nkink0i

Từ 20182 số n

(36)

Nếunij = 0(i6=j)thì Xi khơng quen Xj. Nếunij = 1(i6=j)thì Xi quen Xj

Khi tổng S0 =

2018

X

i=1

i6=k,k0

nkink0i số người quen chung nhóm 2018 người xét

Xk Xk0 Áp dụng bổ đề trên, ta có điều phải chứng minh

Bài 22 ChoA ={1,2,3, , n} Chứng minh tổng số multiset có phần tử lấy từ A có số phần tử khơng vượt n chia hết cho n+

Lời giải

Gọi x1, x2, , xn số lần xuất 1,2, , n multiset nêu x1+x2+· · ·+xn6n

Xét phương trình tương ứng x0 +x1 +x2 +· · ·+xn = n x0 chênh lệch tổng x1+x2 +· · ·+xn n Số nghiệm phương trình

n+ +n−1

n+ 1−1

=

2n n

Ta cần chứng minh n+ |

2n n

Ta có

1

n+

2n n

= (2n)!

(n+ 1)n!n! =

(2n)!(n+ 1−n)! (n+ 1)n!n!

= (2n)!

n!n! −

(2n)! (n+ 1)!(n−1)! =

2n n

2n n+

Đẳng thức cuối cho ta điều phải chứng minh

Bài 23 Cho tập A gồm 20 số nguyên dương Hãy tìm số nguyên dương k nhỏ cho với cách lấy k số phân biệt từ tập A chọn hai số a, b số lấy mà a+b số nguyên tố

Lời giải

Ta có k >2 Với k 6 10ta lấy k số chẵn 2,4,6, ,2k Tổng hai số k số chẵn lớn hơn2, nên số nguyên tố Suy k >11 Ta chứng minh k = 11

là số cần tìm Thật vậy, ta phân hoạch A thành10 cặp (có nhiều cách), ví dụ

{1; 2};{3; 4};{5; 6};

Ta có tổng hai số cặp số nguyên tố Khi lấy 11số tập A, theo nguyên lý Dirichlet, có hai số nằm cặp trên, chúng có tổng số nguyên tố Vậy

sốk nhỏ thỏa mãn yêu cầu đề 11

Bài 24 Với tập hợp X,Y ta định nghĩa phép toán ∆như sau: X∆Y = (X\Y)∪(Y\X)

(37)

i) A∆A =∅, A∆∅=A, ii) A∆B =B∆A,

iii) (A∆B)∆C =A∆(B∆C)

(Khi thay viết (A∆B)∆C ta viếtA∆B∆C)

2 Cho S ={1,2,3, , n}(n >2) Chứng minh n+ tập khác rỗng S, ta ln chọn số tập hợp X1, X2, , Xk(2 k n + 1) cho X1∆X2∆· · ·∆Xk=∅

Lời giải

1 Với A, B, C ba tập hợp

i) Ta có A∆A = (A\A)∪(A\A) = ∅ Ta có A∆∅= (A\∅)∪(∅\A) = A ii) Ta có A∆B = (A\B)∪(B\A) = B∆A

iii) Ta có

(A∆B)∆C = (((A\B)∪(B\A))\C)∪(C\((A\B)∪(B\A))) = (A\((B\C)∪(C\B)))∪(((B\C)∪(C\B))\A) =A∆(B∆C)

2 Ta chứng minh phản chứng, giả sử không tồn

Đặt F ={A1, A2, , An+1}là họ n+ tập S, T ={1,2, , n+ 1} Với J ⊂T,|J|>2, đặt ∆

j∈J

Xj =Xj1∆ .∆Xjh với J ={j1, , jh} Ta có ∆

j∈J

Xj ⊂S,J ⊂T,|J|>2 Ta thấyS có2n−1 tập khác rỗng,T có2n+1−n−2tập

con có từ phần tử trở lên, với n > 2, 2n+1−n−2> 2n−1 Khi theo Định lý Dirichlet,

tồn hai tập J, J0 ⊂T cho ∆ j∈J

Xj = ∆ j0∈J0Xj

0 6=∅ Suy

∆ j∈J

Xj

∆ j0∈J0Xj

0

=∅⇔ ∆

j∈J∆J0Xj =∅

Trường hợp 1.J ⊂J0 Khi ∆

j∈J∆J0Xj =j∈∆J0\JXj =∅ suy |J

0\J|

>2 Ta có điều phải chứng minh

Trường hợp 2.J 6⊂J0, J0 6⊂J, |J∆J0|>2 Ta có điều phải chứng minh

Bài toán giải

Bài 25 Một khu rừng phi lao chắn cát có dạng hình chữ nhật có chiều rộng 222 m, chiều dài là1000 m Trong rừng có4500 phi lao, to có đường kính gốc là0,5m Chứng minh khu rừng có 60 mảnh đất, diện tích mảnh 40m2 mà khơng có phi lao

Lời giải Ta có

(38)

Ta chia chiều rộng khu rừng thành 48 đoạn, đoạn dài m; khoảng cách hai đoạn

0,6 m; hai đầu hai đoạn 0,9 m Chia chiều dài khu rừng thành 95 đoạn, đoạn dài 10m; khoảng cách hai đoạn 0,52m; hai đầu hai đoạn 0,56m

Ta có tất 48·95 = 4560 mảnh đất có diện tích mảnh 40m2 Vì có 4500 phi lao đường kính phi lao khơng vượt q 0,5 m thơng khơng thể chiếm chỗ hai mảnh đất khác Theo ngun lí Dirichlet, cịn nhất60 mảnh đất, mảnh có diện tích40 m2, mà mảnh khơng có phi lao

Bài 26 Cho tập A={1,2, ,2n} Một tập hợp B ⊂A gọi tập cân tập số số chẵn số số lẻ (quy ước tập rỗng tập cân) Hỏi A chứa tập cân

Lời giải

Gọi X tập số chẵn củaA, Y tập số lẻ củaA, C tập tất tập cân chứa trongA vàD tập tập có n phần tử củaA Ta xây dựng ánh xạf giữaC f(B) = B1∪(Y\B2) Khi ta có

|f(B)|=|B1|+|Y\B2|=|B1|+|Y| − |B2|=|Y|=n f(B)∈D Ta chứng minh f song ánh Thật giả sử

f(B) =f(E)⇔B1∪(Y\B2) = E1∪(Y\E2)⇒B1 =E1;Y\B2 =Y\E2

vì B1;E1 tập chứa số chẵn,Y\B2, Y\E2 chứa số lẻ, hay B1 =E1;B2 =E2, suy raf đơn ánh Lấy M ∈Dtùy ý, kí hiệu M1, M2 tương ứng tập tất số chẵn tập tất số lẻ M ĐặtB =M1∪(Y\M2), ta có |Y\M2|=|Y| − |M2|=n− |M2|=|M| − |M2|=|M1| suy raB tập cân f(B) =M Vậy tồn song ánh C D, |C|=|D|=Cn

2n

Bài 27 (P40, Tạp chí Pi, tháng năm 2017) Chuẩn bị cho buổi giao lưu Pi với bạn học sinh Tiểu học u tốn, Lê Hương giao nhiệm vụ chia 500 chocolate (sô-cô-la) thành gói cho gói có sơ lẻ thanh; cô Cẩm Thơ giao nhiệm vụ chia 500 kẹo mút thành gói cho khơng có gói có số kẹo Kí hiệu s t tương ứng số cách chia mà Lê Hương Cẩm Thơ thực Hãy so sánh s t

Lời giải

Nhận thấy

Mỗi cách chia chocolate Lê Hương cách phân tích 500 thành tổng số nguyên dương lẻ Hai cách chia khác cô Lê Hương hai cách phân tích khác theo nghĩa: tồn số ngun dương lẻ có mặt phân tích khơng có mặt phân tích kia, tồn số nguyên dương lẻ mà số lần có mặt phân tích khác số lần có mặt phân tích

Mỗi cách chia kẹo mút cô Cẩm Thơ cách phân tích 500 thành tổng số nguyên dương đôi khác Hai cách chia khác Cẩm Thơ hai cách phân tích khác theo nghĩa: tồn số nguyên dương có mặt phân tích khơng cso mạt phân tích

Như vậy:

s số phần tử tập S gồm tất cách phân tích 500 thành tổng số nguyên dương lẻ;

(39)

Ta so sánhs vàt nhờ việc xây dựng ánh xạ thích hợp từT đếnS Xéta tùy ý thuộcT giả sửalà cách phân tích 500thành tổng củam >1số ngun dương đơi khác nhaua1, a2, , am

Với i= 1, m, có biểu diễn dạng = 2ni ·bi (∗)

trong bi số nguyên dương lẻ vàni số tự nhiên Vì thế, cách phân tích a,

thay mỗiai tổng 2ni số nguyên dương lẻbi, ta có cách phân tích (gọi làb) 500 thành

tổng số nguyên dương lẻ; nói cách khác, ta có cách phân tích b thuộc tập S Hơn nữa, tính biểu diễn (∗) nên cách phân tích b xác định Do đó, phép đặt ứng với cách phân tícha thuộc T với cách phân tích b xác định xác lập ánh xạ, gọi f, từ T đến S Xét b=f(a) với a∈T (nghĩa là, b ảnh a ∈T qua f) Giả sử b cách phân tích500 thành tổng m1 số nguyên dương lẻ b1, m2 số nguyên dương lẻ b2, ., mk số nguyên dương lẻbk Khi đó, từ cách xác định b, dễ thấy, mi, i= 1, k, tổng

các lũy thừa Từ đó, tính biểu diễn dạng (∗) tính biểu diễn hệ nhị phân số nguyên dương, dễ dàng suy ra, a a0 hai phần tử khác thuộc T f(a) f(a0) hai phần tử khác thuộc S Điều cho thấy f đơn ánh (1) Tiếp theo, ta chứng minhf toàn ánh Xét phần tửb tùy ý thuộcS giả sử b cách phân tích

500 thành tổng củam1 số nguyên dương lẻb1,m2 số nguyên dương lẻ b2, , mk số nguyên dương lẻ bk Khi đó, ta có

500 =m1b1+m2b2+· · ·+mkbk (2)

Biểu diễn m1 hệ nhị phân:

m1 = 2s1 + 2s2 +· · ·+ 2sj,

trong đó,s1, s2, , sj số tự nhiên đôi khác Trong tổng (2), thay m1b1 tổng số2s1·b

1,2s2·b1, ,2sj·b1 Làm tương tự đối vớim2b2, , mkbk Khi đó, dễ thấy, ta có

một phân tích 500 thành tổng số nguyên dương đôi khác nhau; nghĩa là, ta có phần tử T Hiển nhiên, b ảnh phần tử qua f Như vậy, với phần tử b ∈S, tồn phần tử a∈T cho f(a) =b Do đó, f tồn ánh (3)

Từ (1) (3), suy f song ánh Vì s=t

Bài 28 Trong trường học, học sinh có khơng q người khác bất đồng quan điểm (tạm gọi "kẻ thù") Để tổ chức ngày hội "Bóng nước" đồn trường dự tính chia học sinh tồn trường thành đội cho đội, học sinh có khơng q "kẻ thù" Hỏi có đồn trường thực kế hoạch hay khơng?

Lời giải

Có nhiều cách giải khác trình bày cách giải sử dụng nguyên lý cực hạn Ý tưởng đơn giản có nhiều ứng dụng (trong nhiều toán phức tạp hơn) Ta chia học sinh trường thành nhóm A, B cách Với nhóm A, B, ta gọi s(A), s(B) tổng tổng số kẻ thù thành viên tính viện Vì số cách chia hữu hạn nên phải tồn cách chia (A0, B0) cho s(A0) +s(B0) nhỏ Ta chứng minh cách chia thỏa mãn yêu cầu toán Giả sử cách chia chưa thoả mãn yêu cầu, tức có học sinh có nhiều kẻ thù nhóm Khơng tính tổng qt, giả sử học sinh x thuộc A0 có kẻ thù A0 Khi ta thực phép biến đổi sau, chuyển x từ A0 sang B0 để cách chia A0 = A0\ {x} B0 =B0∪ {x} Vì x có kẻ thù A0 A0 khơng cịn chứa x nên ta có s(A0) s(A0)−4 (trong tổng s(x) và2 kẻ thù x A0) Vì x có khơng q kẻ thù có kẻ thù trongA0 nên xcó nhiều kẻ thù B0 (hay B0), cho nêns(B0)6s(B0) + Từ

s(A0) +s(B0)6s(A0) +s(B0)−2

Mâu thuẫn với tính nhỏ củas(A0) +s(B0) Vậy điều giả sử sai, tức cách chia(A0, B0)thỏa

(40)

Bài 29 Robinson Crusoe lạc đảo hoang 28 năm Trong chừng quãng thời gian, năm, ơng ta tìm khắc lên thân hình đa giác lồi thật lớn để kỷ niệm Biết tổng tất góc đa giác lồi vừa đúng1687π, năm mà ông cứu thoát, đưa trở giới lồi người Bạn cho biết có cách ông khắc đa giác lên thỏa mãn yêu cầu trên, biết 10 năm đầu tiên, năm, ơng khắc lên đa giác có 100 cạnh

Lời giải

Đa giác cón cạnh có tổng góc (n−2)π Do đó, ta cần điểm số nghiệm phương trình sau      28 X i=1

(xi−2)π= 1687π, xi ∈N

xi >100, i= 1,10, xi >3, i= 11,28

Ta có

28

X

i=1

(xi−2)π = 1687π⇔

28

X

i=1

xi = 1743

Đặt xi =yi+ 100, i= 1,10và xi =yi+ 3, i= 11,28, ta có phương trình

10

X

i=1

yi+ 10·100 +

28

X

i=11

+18·3 = 1743⇔

28

X

i=1

y1 = 689

Vậy số nghiệm nguyên không âm phương trình

689 + 28−1 28−1

= 716 27

Bài 30 Cho p số nguyên tố lẻ Tìm số tập X tập {1,2, ,2p} biết X chứa p phần tử tổng tất phần tử X chia hết chop

Lời giải

Đặt A={X ⊂ {1,2, ,2p}:|X|=p}; Aj =

X ∈A:S(X)≡j(modp), j = 0, p−1 Khi ta có

|A|=

2p p

Vì A=A0∪A1∪ ∪Ap−1 Ai∩Aj 6=∅,∀i6=j nên |A|=|A0|+|A1|+ +|Ap−1| Xét đa thức P(x) =xp−1 +xp−2 + +x+

Đa thức có p−1 nghiệm phức phân biệt Giả sử α nghiệm P(x) tập nghiệm P(x)là {α, α2, , αp−1} vàαp = Do phương trìnhxp = 1có pnghiệm phức phân biệt α, α2, , αp nên ta viết xp−1 =

p

Y

k=1

x−αk Từ đó, ta có 2p

Y

k=1

x−αk= p

Y

k=1

x−αk p

Y

k=1

x−αp+k= p

Y

k=1

x−αk p

Y

k=1

x−αk= (xp−1)2

So sánh hệ số xp hai vế của

2p

Y

k=1

x−αk = (xp−1)2, ta (−1)pX X∈A

αS(X) = −2 với S(X) tổng tất phần tử tậpX Do p số nguyên tố lẻ vàαS(X) =αk nếu X ∈A

k ta p−1

X

k=0

(41)

Do đóα nghiệm đa thức Q(x) = p−1

X

k=0

|Ak|xk+|A0| −2 Vì α nghiệm đa thứcP(x) =xp−1+xp−2+ +x+ nên

|A1|=|A2|= =|Ap−1|=|A0| −2

Suy |A0| −2 =

|A1|+|A2|+ +|Ap−1|+|A0| −2

p =

|A| −2

p , |A0|=

2p p

−2

p + Vậy số tập conX tập {1,2, ,2p} biết rằngX chứa pphần tử tổng tất phần tử X chia hết chop

2p p

−2

p +

Bài 31 Xét S tập hữu hạn n điểm mặt phẳng khơng có ba điểm thẳng hàng Xét đa giác lồi P với đỉnh điểm S Đặt a(P) số đỉnh P b(P) số đỉnh củaS nằm đa giác P Chứng minh với số thực x ta có

X

P

xa(P)(1−x)b(P)=

gọi tổng lấy đa giác lồi với đỉnh thuộc S Lời giải

Với n= 0,1,2 tập rỗng,một điểm, đoạn thẳng Do đó, ta xét n >3

Với đa giác lồi P với đỉnh điểm thuộc S Đặt c(P) số điểm S nằm đa giác P Khi a(P) +b(P) +c(P) =n với n số điểm S Kí hiệu y= 1−x Khi đó, ta có

X

P

xa(P)yb(P) =X P

xa(P)yb(P)(x+y)c(P)=X P

c(P)

X

i=0

c(P)

i

xa(P)+iyb(P)+c(P)−i

Chúng ta xem biểu thức đa thức bậc n hai biến độc lập x, y Biểu thức tổng hạng tử có dạngxryn−r(06r6n)với hệ số ngun khơng

âm Với r cố định hệ số

n r

của xryn−r biểu diễn số cách chọn đa giác lồi P cór−đỉnh và

đó số cách chọn điểm S nằm P số đỉnh P số cách chọn điểm trongP tăng tới r

Điều tương đương với số cách chọn tập r− phần tử S

n r

Tương ứng song ánh với tập hợpT điểm S tách thành hai tập hợp rời có tập tập tất đỉnh đa giác tập lại tập tất điểm nằm đa giác T Khi hệ số củaxryn−r là

n k

Do đó, ta có

X

P

xa(P)yb(P) = n

X

r=0

n r

xryn−r= (x+y)n=

Bài toán giải

(42)

nhất người gọi khơng có loại thức ăn có 15 người gọi Hỏi có tất cách gọi thức ăn

Lời giải

Gọix1, x2, x3, x4 số người gọi cơm, mì, bún phở hi đó, tốn trở thành tìm số nghiệm ngun phương trình

x1+x2+x3+x4 = 40 (1)

với 16xi 615, i= 1,2,3,4 Đặt yi =xi−1 (∀i= 1,2,3,4) Khi đó, phương trình (1) trở thành

y1+y2+y3+y4 = 36 (2)

Gọi X tập hợp nghiệm nguyên, không âm phương trình (2) Áp dụng tốn chia kẹo Euler, ta được|X|=

39 36

Gọi A1, A2, A3, A4 tập hợp nghiệm nguyên phương trình (2) với điều kiệny1 >15, y2 >15, y3 >15, y4 >15

XétA1, đặt a1 =y1−15>0, ta phương trình (2) trở thành a1+y2+y3+y4 = 21

, với a1 > 0, y2 > 0, y3 > 0, y4 > Áp dụng toán chia kẹo Euler, ta |A1| =

24 21

Tương tự, ta có

|A2|=|A3|=|A4|=

24 21

XétA1 ∩A2 Đặta1 =y1−15>0, a2 =y2 −15>0, ta phương trình (2) trở thành a1+a2+y3+y4 =

, vớia1 >0, a2 >0, y3 >0, y4 >0 Khi đó, số nghiệm phương trình là|A1∩A2|=

9

Tương tự, ta có

|A1∩A3|=|A1∩A4|=|A2∩A3|=|A2∩A4|=|A3∩A4|=

9

XétA1 ∩A2∩A3 Đặt a1 =y1−15>0, a2 =y2−15>0, a3 =y3 −15, ta phương trình (2) trở thành

a1+a2+a3+y4 =−11

vớia1 >0, a2 >0, a3 >0, y4 >0 Khi đó, số nghiệm phương trình |A1∩A2∩A3|=

0 Tương tự, ta có

|A1∩A2 ∩A3|=|A2∩A3∩A4|=|A3∩A4∩A1|=|A4∩A1∩A2|= 0;

|A1∩A2∩A3∩A4|= Do đó|A1∪A2 ∪A3∪A4|=

24 21

−6

9

Vậy số cách gọi thức ăn thỏa mãn toán

|X| − |A1∪A2∪A3∪A4|=

39 36

4

24 21

−6

9

= 1547

(43)

Bài 33 (Vietnam TST 2004) Xét tập hợpSgồm2004số nguyên dương phân biệta1, , a2004 có tính chất: Nếu với i= 1,2,3 ,2004, ta ký hiệu f(ai)là số số thực thuộc S nguyên

tố với d(ai)<2003 vàf(ai) = f(aj)với i, j ∈ {1,2,3, ,2004} Hãy tìm

số nguyên dương k nhỏ cho k – tập tập S tuỳ ý có tính chất nêu tồn hai số phân biệt mà ước số chung lớn chúng khác1 (k - tập tập có k phần tử)

Lời giải Xét tập hợp

S0 ={p1, p2, p3, , p1001, p1002, p1p1003, p2p1004, , p1001p2003, p1002p2004}

trong p1, p2, p3, , p2004 số nguyên tố đôi khác Dễ thấy tập hợp thỏa mãn đề Hơn nữa, 1002 tập con{p1, p2, p3, , p1002}của tập hợp cho, khơng có hai phần tử mà ước chung chúng khác Suy rak >1003.Xét tập hợpStùy ý thỏa mãn điều kiện đề Với s ∈ S, ta gọig(s) số số không nguyên tố với s Từ giả thiết cho, ta có g(s)>1,∀s ∈S g(s) =g(t) với mọis, t ∈S hay g(s) =m,∀s ∈S với m số nguyên dương Xét một1003-tập conT tùy ý củaS.Ta chứng minh trongT,luôn tồn hai số mà ước chung lớn chúng lớn hơn1.Thật vậy, giả sử ngược lại trongT,không tồn hai số mà ước chung lớn chúng khác1 Kí hiệuA={a∈S|∃t∈T,(a, t)6= 1}, ta có A∩T =∅ Từ suy |A|6 2004−1003 = 1001 Mặt khác, số số thuộc S (kể lặp) mà số khơng ngun tố với số thuộcT 1003m số có tính lặp tối đam lần nên|A|> 1003m

m = 1003.Mâu thuẫn cho ta điều phải chứng minh Vậy giá trị nhỏ

nhất cần tìm củak là1003

Bài 34 Có25con lừa xếp hàng lừa đứng bìa phải già có tên Eeyore Ông chủ chúng muốn tặng cho bóng với màu cho lừa đứng cạnh có bóng khác màu nhau, đồng thời, màu sử dụng 1lần Bà chủ chúng lại tặng cho vòng với màu cho lừa Eeyore nhận vòng màu đỏ nhất, lại nhận vòng với 6màu (2

con cạnh không thiết có vịng khác nhau) Hỏi cách tặng q ông chủ hay bà chủ nhiều nhiều lần? Chú ý bóng vịng khác màu sắc

Lời giải

Ta đặt (*) điều kiện tặng bóng cho lừa cho2con lừa đứng cạnh có bóng khác màu Đánh số màu bóng từ đến gọi Ai với 16i 67 cách tặng

bóng cho lừa cho bóng màu thứ i khơng dùng thỏa mãn điều kiện (*) Đặt A0 cách tặng bóng cho có điều kiện (*) thỏa Ta có |A0| = 7·624 Ngồi ra, với T ⊂ {1,2,3, ,7}

\

i∈T Ai

= (7− |T|) (6− |T|)24

Do đó, ta tính

[ k=1 Ak = X l=1

(−1)l+1 X

|T|=l

[

k∈T Ak = X m=0

(−1)m

7

m

·m(m−1)24

Suy số cách tặng bóng thỏa mãn đề

|A0| −

[ k=1 Ak

= 7·624−

6

X

m=1

(−1)m−1

m

7

·m(m−1)24 =

7

X

m=1

(−1)m−1

7

m

(44)

Bằng cách tương tự, ta tính số cách tặng vịng với6 màu cho24con cừu khơng có ràng buộc giống (*)

6

X

m=0

(−1)m

6

m

·m24=

7

X

m=1

(−1)m−1

6

m−1

·(m−1)24

Chú ý

7

m

·m(m−1)24

6

m−1

·(m−1)24

= m m

m−1

=

7!m m!(7−m)! :

6!

(m−1)!(7−m)! = nên suy số

cách tặng bóng nhiều gấp 7lần số cách tặng vòng Bài 35 Một số nguyên dương k gọi "đẹp" phân hoạch tập hợp số nguyên dương Z+ thành k tập hợp A1, A2, , Ak cho với số nguyên dương n > 15

với i∈ {1; 2; ;k} tồn hai số thuộc tậpAi có tổng n 1 Chứng minh rằngk = đẹp

2 Chứng minh k>4 không đẹp Lời giải

Với k = ta cách phân hoạch tập N∗ thành tập A1, A2, A3 thỏa mãn sau

A1 ={1; 2; 3}

[

{3m|m>4}

A2 ={4; 5; 6}

[

{3m−1|m >4}

A3 ={7; 8; 9}

[

{3m−2|m >4}

Vậy k = đẹp Xét k > Giả sử tồn cách chia N∗ thành k tập A1, A2, , Ak thỏa mãn đề

bài Khi cách phân hoạch N∗ thành tập A1, A2, A3 (A4SA5S SAk) thỏa mãn đề

bài Do cần xét với k = đủ Đặt Bi =AiT{1; 2; 3; ; 23} với i= 1,2,3,4 Ta thấy

số thuộc tập {15; 16; 17; ; 24} (gồm 10 số) viết thành tổng hai số thuộc tập Bi (với

mọi i = 1,2,3,4) Như |Bi|= m số tổng

m

2

>10⇒ m >5 Vậy tập Bi

có phần tử Mà|B1|+|B2|+|B3|+|B4|= 23 nên khơng thể xảy ra|Bi|>6,∀i= 1,2,3,4

Do phải tồn tập Bi cho |Bi|=

Giả sử Bi ={a1;a2;a3;a4;a5} Khi

{ai+aj; 16i < j 65}={15; 16; 17; ; 24}

Suy X 16i<j65

(ai+aj) = 15 + 16 + + 24, hay4 (a1+a2+a3+a4+a5) = 195, vơ lí Vậy mọik>4

đều khơng đẹp

Bài 36 Với mlà số nguyên dương lớn 1, cô giáo phát 7m viên kẹo cho7học sinh cho em có kẹo khơng xảy trường hợp tất học sinh nhận số kẹo Chứng minh số cách phát kẹo bội 49

Lời giải

Rõ ràng số cách chia kẹo

7m−1

−1 Ta chứng minh số chia hết cho49,

(7m−1)!

6! (7m−7)! −1 =

(7m−6) (7m−5)· · ·(7m−1)−6!

(45)

Do (6!,7) = nên ta cần chứng minh tử số chia hết cho 49

Nếu coi(7m−6) (7m−5)· · ·(7m−1)là đa thức theom có số hạng tự là6!(triệt tiêu với

6!có sẵn), nữa, hệ số bậc khai triển

(7m) 6!

1 +

1

2 +· · ·+

= 7!

7 1·6+

7 2·5 +

7 3·4

m chia hết cho49

Các hệ số bậc cao m có chứa lũy thừa cao tương ứng nên rõ ràng chia hết cho49 Từ suy tử số phân số chia hết cho 49, ta có điều phải chứng minh

Bài 37 (Ấn Độ, 2012) Cho tam giác ABC điểm P gọi tốt tìm 27tia chung gốc P cắt tam giác thành 27tam giác có diện tích Đếm số điểm tốt

Lời giải

Trước hết ta thấy P A, P B, P C phải 27 tia nêu đề (nếu khơng có phần phần tam giácABC) Giả sử SABC = 27thì diện tích tam giác nhỏ

bằng Đặt SP BC =x, SP CA =y,SP AB =z rõ ràng, tam giác P BC, P CA, P AB phải

chứa số nguyên tam giác có diện tích nên ta có x, y, z∈Z+ và

x+y+z = 27 (2.2)

Chú ý với P nằm tam giác

−→

P A·SP BC +

−−→

P B·SP CA+

−→

P C ·SP AB =

− →

0

và ngược lại với (SP BC, SP CA, SP AB) tồn nhấtP thỏa mãn.(Ở ta cịn

có thể chứng minh tính cách kẻ đường thăng song song với cạnh tam giác)

A

B C

P z

x y

Vậy số điểm cần tìm số nghiệm phương trình (2.2)

27−1 3−1

=

26

= 325

Bài 38 Một hoán vị{x1, x2, , x2n}của{1,2, ,2n}được gọi "đẹp" nếu∃i∈ {1,2, ,2n−1}

thỏa mãn|xi−xi+1|=n Chứng minh số hoán vị "đẹp" nhiều số hoán vị không "đẹp" Lời giải

Bài đề thi IMO năm đó, ý tưởng sử dụng tính chất ánh xạ Ta xây dựng đơn ánh khơng song ánh từ số hốn vị khơng đẹp vào số hốn vị đẹp Ta thấy tập hợp 2n số nguyên dương phân hoạch thànhn cặp có khoảng cách n

(46)

Do với hốn vị khơng đẹp (x1, x2, , x2n) tồn số nguyên dương k ∈ {1,2, ,2n−1}

cho(xk, x2n)là cặp số nêu Xét tương ứng

(x1, x2, , xk, , x2n)→(x1, , xk−1, xk, x2n, x2n−1, , xk+1,)

Rõ ràng đơn ánh khơng phải song ánh hốn vị đẹp(1,2, , n−1, n+ 1, ,2n−

1, n,2n) tạo ảnh tương ứng Vậy số hốn vị đẹp nhiều số hốn vị khơng đẹp Bài 39 Cho số nguyên a1, a2, , a2013 với 06 6100, i= 1; 100 Với cặp (ai;ai+1) ta cộng thêm1 vào hai số cặp khơng xuất q k lần Tìm k nhỏ cho hữu hạn lần thực thiện thao tác ta số

Lời giải

Ta xét trường hợp số cạnh cách xa

(

a1 =a3 =· · ·=a2013 = 100 a2 =a4 =· · ·=a2012 =

ĐặtS = (a2−a3) + (a3−a4) +· · ·+ (a2012−a2013) lúc chưa tác động thìS =−100·1006 Sau hữu hạn lần tác động tất số S = Ta nhận xét

Tác động lên cặp (a2, a3),(a2, a3), ,(a2012, a2013) thì S khơng đổi. Tác động lên cặp (a1, a2) thì S tăng lên đơn vị.

Tác động lên cặp (a2013, a1) thì S giảm đơn vị.

Bộ (a1, a2) bị tác động lớn 100.1006 lần k > 100.1006 Ta chứng minh k =

100.1006 giá trị nhỏ thoả mãn

Tác động cặp(ai−1, ai) số lần là ai+1+ai+3+· · ·. Tác động cặp(ai, ai+1) số lần là ai+2+ai+4+· · ·.

Sau lần tác động số bằnga1+a2+· · ·a2012+a2013 nên số lần tác động

6100.1006 suy điều phải chứng minh

Bài 40 (AIME, 1986) Cho xâu nhị phân 001101001111011 có cặp 01, cặp 10, cặp 11

và cặp 00đứng cạnh Hỏi có tất xâu nhị phân tính chất thế?

Lời giải

Ta thấy xâu nhị phân thỏa mãn đề phải có dạng XY XY XY XY,

trong đóX dãy số 0liên tiếp vàY dãy số 1liên tiếp Như thế, có tổng cộng

dãy 0và 4dãy Rõ ràng dãy có độ dài k số lượng cặp giống k−1 Gọix, y, z,t độ dài dãy 0thì theo giả thiết, ta phải có

x−1 +y−1 +z−1 +t−1 = 2⇔x+y+z+t = Số nghiệm nguyên dương phương trình

6−1 4−1

=

5

= 10

Lại có m, n, p, q độ dài dãy

(47)

Số nghiệm nguyên dương phương trình

9−1 4−1

=

8

= 56 Do hai dãy chọn độc

lập với nên số xâu thỏa mãn 10·56 = 560xâu

Nhận xét Tổng quát Hỏi với số tự nhiêna, b, c, d tồn xâu nhị phân có độ dài dương có a cặp 01,b cặp 10, c cặp 11và d cặp 00 đứng cạnh nhau? Ta thấy xếp số lên vòng tròn số0 tạo thành nhóm rời Rõ ràng số lượng nhóm0

bằng số lượng nhóm Do đó, cắt xâu ranh giới hai nhóm số lượng hai bộ01 10sẽ giảm đi1 đơn vị so với Cịn cắt tạ vị trí nhóm (trường hợp số lượng số nhóm lớn 1) khơng làm thay đổi số lượng 01 10 Trước hết, ta ln có|a−b|61 Ngồi ra, dễ thấy c >0, d >0 phải tồn ranh giới hai nên lúc đó, ta phải có a >0hoặc b >0 Từ ta thấy điều kiện số a,b,c, d

(1) Nếu cd= a=b=

(2) Nếu cd6= |a−b|61 a, b khơng đồng thời

Đặt k = max{a, b} rõ ràng có tổng cộng k nhóm theo nhận xét ban đầu Giả sử k > Gọi x1, x2, , xk số lượng số1 có nhóm rõ ràng

x1+x2+x3+· · ·+xk =c+k

số nghiệm dương phương trình

c+k−1

k−1

Tương tự số cách xếp số

d+k−1

k−1

Vậy số xâu nhị phân thỏa mãn

c+k−1

k−1

d+k−1

k−1

với k = max{a, b} với k>1 Trong trường hợpk = dễ thấy có xâu thỏa mãn đề Bài toán giải

Bài 41

1 Một đại hội thể thao phường Nam Cường có n vận động viên tham gia thi đấu

nội dung gồm: chạy 100 mét, đẩy tạ, bắn nỏ, đua xe đạp, bơi tự 100 mét, nhảy cao, nhảy xa Biết vận động viên tham gia thi đấu nội dung thi đấu Theo thống kê ban tổ chức cho thấy nội dung thi đấu có số vận động viên tham dự 40 Ngoài hai nội dung thi đấu có khơng q vận động viên tham gia thi đấu hai nội dung Chứng minh n>120

2 Có 16học sinh tham gia kì thi, đề thi có n câu hỏi câu hỏi có phương án lựa chọn Biết 2học sinh có khơng quá1câu trả lời chung cho tất câu hỏi Tìm giá trị lớn củan Hãy trường hợp đển đạt giá trị lớn

Lời giải

1 Với vận động viên thứj ký hiệu aj môn thi đấu vận động viên tham gia Ta lập bảng có 7hàng n cột Ơ(i;j) dịngi, cột j đánh dấu ×nếu vận động viên j tham gia nội dung i Tổng số dấu × tính theo cột a1+a2+ .+an, tổng 7·40 = 280

nếu tính theo dịng (vì nội dung thi đấu có 40 vận động viên tham gia) Bây ta tính S số cặp (×,×) theo cột Trên cột j có

aj

cặp, số S cặp (×,×)

theo cột

a1

2

+

a2

2

+ .+

an

= a

2

1+a22+ .+a2n

(48)

Thay a1+a2+ .+an= 280 áp dụng bất đẳng thức quen thuộc a21+a22+ .+a2n > (a1+a2+ .+an)

2

n ,

từ (1) ta cóS > 280

2n −140 (2)

Mặt khác, với cặp dòng, nhiều có cột có cặp (×,×) dịng này, tổng số cặp (×,×) cột tất

7

cặp dịng khơng vượt q

7

cặp (×,×) Ta có bất đẳng thức

7

> S Thay vào (2), ta có

> 280

2n −140 có n> 280

2 + 140

>119

Vậy n>120

2 Ta đếm số (A, B, C)mà học sinh A, B có lựa chọn C

Đếm theo học sinh, có S 6

16

Đếm theo lựa chọn.

Ta đặt ai, bi, ci, di số học sinh chọn đáp án 1, 2, 3, câu hỏi i

Ta có S= n X i=1 + bi + ci + di

với ai+bi+ci+di = 16

+ bi + ci + di = a

i +b

2

i +c

2

i +d

2

i

2(ai+bi+ci+di) >

4(ai+bi+ci+di)

2−

2(ai+bi+ci+di)

Suy S >24n Từ ta có n 65

Chứng minhn = thỏa mãn từ suy giá trị lớn n Dưới cách:

Bài Bài Bài Bài Bài

Sinh viên A A A A A

Sinh viên A B B B B

Sinh viên A C C C C

Sinh viên A D D D D

Sinh viên B A C D B

Sinh viên B C A B D

Sinh viên B D B A C

Sinh viên B B D C A

Sinh viên C A D B C

Sinh viên 10 C D A C B

Sinh viên 11 C B C A D

Sinh viên 12 C C B D A

Sinh viên 13 D A B C D

Sinh viên 14 D B A D C

Sinh viên 15 D C D A B

Sinh viên 16 D D C B A

(49)

Bài 42 (Tây Ban Nha, 2012) Cho x n số nguyên dương cho 6 x 6 n Người ta cóx+ 1cái hộp phân biệt vàn−xquả bóng giống Gọi f(n, x)là số cách phân phối n−x bóng vào x+ 1hộp Với plà số nguyên tố cho trước, tìm số nguyên n lớn hơn1 cho số nguyên tố plà ước f(n, x) với mọix∈ {1; 2; .;n−1}

Lời giải

Theo công thức nêu, ta dễ dàng có f(n, x) =

n x

Do đó, ta cần tìm n cho p |

n x

với mọix∈ {1; 2; .;n−1} Theo định lý Lucas

n x

n1 x1

n2 x2

n3 x3

· · ·

nk xk

(mod p)

trong n1, n2, , nk x1, x2, , xk chữ số n, x biểu diễn p phân Nếu tồn

một chữ sốni >1thì tương ứng, ta chọnxi = tất chữ số lại n,x giống

rõ ràng

n x

6≡0 (mod p), không thỏa mãn Do đó, tất chữ số n, trừ chữ số đầu tiên, phải khác Ngoài ra, chữ số n phải khơng, n1 > ta chọn x1 = dẫn đến trường hợp không thỏa mãn Suy ran = 100 .0(p) hay nói cách khác, n lũy thừa

của p

Bài 43 (Trường Đông TPHCM, 2014) Cho n > số nguyên dương Xét tập hợp đường ngắn lưới nguyên từ điểm A(0; 0) đến điểm B(n;n) (độ dài đường số lượng bước đi) Một đường tương ứng với dãy gồm n lệnh T (lên trên) n lệnh P (sang phải) Trong dãy đó, cặp lệnh (T, P) kề gọi bước chuyển (lưu ý, cặp (P, T) không gọi bước chuyển) Ví dụ dãy P T T P T P P T có2

bước chuyển Hãy tìm số đường từ A đến B cho có đúng:

1 bước chuyển

2 bước chuyển

Lời giải

1 Ta phát biểu lại toán là: Cho xâu nhị phân có độ dài 2n với n số n số Xác định số xâu nhị phân thỏa mãn cặp 10 xuất lần.Gọi A xâu nhị phân chứa toàn số 1, B xâu nhị phân chứa tồn số Rõ ràng có xâu dạng sau thỏa mãn đề bài:AB,ABA, BAB, BABA

Dạng 1 có1 xâu.

Dạng 2 có

n−1 2−1

=n−1xâu (ta đếm số nghiệm nguyên dương x+y=n) Dạng 3 cón−1 xâu.

Dạng4có(n−1)2 xâu (ta đếm số nghiệm nguyên dương cặp phương trìnhx+y=n nhân lại BA phía trước xây dựng độc lập với BA phía sau)

(50)

2 Tương tự trên, ta có dạng ABAB, ABABA, BABAB, BABABA Dạng 1 có(n−1)2 xâu.

Dạng có

n−1 3−1

n−1 2−1

= (n−1)

2

(n−2)

2 xâu

Dạng có (n−1)

(n−2)

2 xâu

Dạng 4 có

n−1 3−1

n−1 3−1

= (n−1)

2

(n−2)2

4 xâu

Vậy số đường tổng cộng thỏa mãn

(n−1)2(n−2)2

4 + 2·

(n−1)2(n−2)

2 + (n−1)

2 = (n−1)2

4

(n−2)2+ 4(n−2) + = (n−1)

2 n2

4

Bài toán kết thúc

Bài 44 Cho tậpS ={1,2, ,3n}, phân hoạchS =A∪B∪C thỏa mãn|A|=|B|=|C|=n Chứng minh tồn a∈A, b∈B, c∈C cho (a+b−c) (b+c−a) (c+a−b) = Lời giải

Giả sử không tồn tạia ∈A, b∈B, c∈C cho (a+b−c) (b+c−a) (c+a−b) = Giả sử 1∈A, k ∈N∗ thỏa mãn 1,2, , k−1∈A mà k /∈A, giả sửk ∈B suy ra 2∈/ C Vì k = 2⇒k ∈B Nếu k >2⇒k−1>2⇒2∈A

Nhận xét Nếux∈N∗, x∈C, x−1∈A Thật giả sửx∈C, x−1∈/ A⇒x−1∈B∨x−1∈C. Ta có

(1, x−1, x) : 1∈A, x∈C ⇒x−1∈C

(x−k, k, x) :k ∈B, x∈C ⇒x−k∈B∨x−k ∈C (x−k, k−1, x−1) : k−1∈A, x−1∈C⇒x−k ∈C

(x−k−1, k, x−1) : k∈B, x−1∈C ⇒x−k−1∈B∨x−k−1∈C (x−k−1,1, x−k) : 1∈A, x−k ∈C ⇒x−k−1∈C

Từ suy x∈C, x−1∈C ⇒x−k, x−k−1∈C lập luận tương tự⇒x−ik, x−ik−1∈C Chọni cho16x−ik−1< k hoặc16x−ik < k ⇒ x−ik−1∈A hoặcx−ik∈A vô lý Vậy x∈C ⇒x−1∈A Đặt C ={c1 < c2 <· · ·< cn} ⇒c1−1;c2−1,· · · , cn−1∈A, mà

ci >2⇒ci−1>2⇒ {1, c1−1, c2−1, , cn−1} ∈A⇒ |A|> n

(51)

Bài 45 Cho X tập gồm n số Giả sử a1, a2, , ak số thỏa mãn điều kiện,

∀u, v ∈X, u6=v tồn cho (u−ai)(v−ai)60 Chứng minh n63k

Lời giải

Đặt Y ={(u1, u2, , uk) :ui = 0,1,2} Với u∈X, đặtf(u) ={u1, u2, , uk} ∈Y,

ui =

    

0, u < 1, u=ai 2, u >

với mọii= 1,2, , k Ta chứng minh f đơn ánh Thật vậy, giả sử tồn tạiu, v ∈X,3 u6=v f(u) = (u1, u2, , uk) = (v1, v2, , vk) = f(v)

thì vìu 6=v nên tồn cho (u−ai)(v−ai)6 Để ý ui =vi,∀i= 1,2, , k nên u, v < u, v = u, v > Cả ba trường hợp không xảy nên f đơn ánh

Từ suy n=|X|6|Y|= 3k

Bài 46 Với n ∈ N, ký hiệu Hn tập tất hoán vị (a1, a2, , an) tập hợp [n] ={1,2, , n} Xét tập hợp

Sn ={(a1, a2, , an)∈Hn:ai >i−1,∀i= 1,2, , n}, Tn={(a1, a2, , an)∈Hn :ai 6i+ 1,∀i= 1,2, , n}

Tìm tất số nguyên dương n cho |Tn|

|Sn| >

3

Lời giải

Đặt Pn={(a1, a2, , an)∈Sn:a1 = 1}, Qn ={(a1, a2, , an)∈Sn :an6= 1} Hiển nhiên Pn∩Qn=∅, Pn∪Qn =Sn

1 Xét ánh xạ f từ Pn đến Qn sau: với (a1, a2, , an) ∈ Pn ta cho tương ứng với (a2, a1, , an)∈Qn Dễ thấy f song ánh nên |Pn|=|Qn|=

1 2|Sn|

2 Xét ánh xạ g từ Pn đến Sn−1 sau: với (a1, a2, , an) ∈ Pn ta cho tương ứng với (a2 −1, a3 − 1, , an −1) ∈ Sn−1 Dễ thấy g song ánh nên |Pn| = |Sn−1| Do ta có

|Sn|= 2|Sn−1|, n>2 Hơn nữa|S2|= 2,|S1|= nên |Sn|= 2n−1

3 Xét ánh xạ h từ Tn đến Tn−1 ∪Tn−2 xác định sau: với (a1, a2, , an)∈ Tn ta cho

tương ứng với(a2−1, a3−1, , an−1)nếua1 = tương ứng với(a3−2, a4−2, , an−2)

nếu a1 = Dễ thấyh song ánh nên |Tn|=|Tn−1|+|Tn−2|, mà |T2|= 2,|T1|=

|Tn|=

5

 

1 +√5

!n+1

− 1−

5

!n+1 

Từ giải |Tn|

|Sn| >

3 ⇔n66

(52)

Bài 47 Xét đa giác đỉnh Với đỉnh A, B, C cho trước, tính số ba tam giác

(41;42;43)được lập từ 9đỉnh cho thỏa mãn điều kiện sau

i

  

A ∈∆1 B ∈∆2 C ∈∆3

ii Các tam giác khơng có chung đỉnh

iii Từ tam giác ta chọn cạnh cho cạnh Lời giải

a1

a2

a3

a4

A1 A2

A3

A4

A5 A6

A7 A8

A9

Ký hiệu thành phố a1, a2, , a9 hình vẽ Khơng tổng qt giả sử bán kính đường trịn ngoại tiếp đa giác Khi khoảng cách 2thành phố khoảng cách cạnh đường chéo nhận1 4giá trị sau

a1 = sin π

9;a2 = sin 2π

9 ;a3 = sin 3π

9 ;a4 = sin 4π

9

Khi độ dài cạnh tam giác thuộc trường hợp sau

(a1;a1;a2); (a2;a2;a4); (a1;a3;a4); (a3;a3;a3); (a2;a3;a4); (a4;a4;a1)

Ta chứng minh với gồm tam giác (khơng có chung đỉnh) ln chọn tam giác cạnh cho cạnh có độ dài Ta phân hoạch tập đỉnh thành3 tập

M ={A1;A4;A7}; N ={A2;A5;A8}; P ={A3;A6;A9} Khi

1tam giác phải có đỉnh thuộc1 tập tập 1tam giác cân (không đều) phải có 3 đỉnh thuộc cả 3 tập trên.

1tam giác khơng cân phải có thuộc 1tập đỉnh lại thuộc tập khác

Trường hợp Tồn tam giác (hay có tam giác mà đỉnh thuộc tập M, N, P) Giả sử là(A1;A4;A7) có cạnh làa3 2tam giác cịn lại có đỉnh 1trong đỉnh sau {A2;A3};{A5;A6};{A8;A9}

Khi tam giác chứa đỉnh Ai;Aj thỏa |i−j| = (mod 3) nên chứa

(53)

Trường hợp Cả 3tam giác cân, tam giác (cả3tam giác tam giác có

1đỉnh thuộc 1trong3tập M, N, P) độ dài cạnh chúng nhận giá trị sau (a1;a1;a2); (a2;a2;a4); (a4;a4;a1); nhiên nhận 3bộ giá trị nên suy điều kiện tốn thỏa

Trường hợp Có tam giác cân tam giác không cân (theo nhận xét trường họp không xảy ra)

Trường hợp Có tam giác cân 2tam giác khơng cân tam giác khơng cân có độ dài cạnh chúng nhận giá trị sau (a1;a3;a4); (a2;a3;a4);tuy nhiên nhận 2bộ nên suy điều kiện toán thỏa

Trường hợp Cả tam giác khơng cân độ dài cạnh chúng nhận giá trị sau (a1;a3;a4); (a2;a3;a4); ln chứaa3

Do số cách chọn thỏa điều kiện đề

2

= 90

Bài 48 Một đề thi cón câu hỏi, điểm câu hỏi 1điểm Một nhómn học sinh tham gia giải đề thi này, em làm thi độc lập với số điểm nhóm tổng số điểm em Người ta thấy hai câu có tối đa 1em giải hai câu

1 Hãy tính số điểm lớn có nhómn em học sinh

2 Chỉ trường hợp số điểm lớn n= 6, n=

Lời giải

1 Ta giả sử emA1, A2, , Antham gia giải câua1, a2, , an ĐặtS ={(Ai, aj, ak)}

nghĩa emAi giải hai câuaj, ak Ta kí hiệud(ai) số câu mà Ai giải được, ta

có|S|= n

X

i=1

d(ai)

Mặt khác, hai câu có tối đa em giải nên

|S|6

n ⇒ n X i=1

d(ai) n ⇔ n X i=1

d(ai) (d(ai)−1)6n2−n

hay

n2−n+ n

X

i=1

d(ai)> n

X

i=1

d2(ai)>

n n

X

i=1 d(ai)

!2

Chú ý giá trị E = n

X

i=1

d(ai)chính tổng số điểm nhóm nên lại có

E2 −nE +n2−n3 60⇔E 6

2 n+n

4n−3 Vậy giá trị lớn số điểm nhóm hn

2 +

4n−3i

2 Trong trường hợp n= tổng số điểm lớn nhóm 21 Đặt Ai{ai1, ai2, }là

người thứ igiải câu ai1, ai2, Ta xếp trường hợp4 người với số điểm

(54)

Trường hợp 5người với số điểm 12

A1{a1, a2, a4};A2{a2, a3, a5};A3{a3, a4};A4{a1, a3}, A5{a5, a1} Trường hợp 6người với số điểm 16

A1{a1, a2, a4};A2{a2, a3, a5};A3{a3, a4, a6};A4{a1, a3}, A5{a5, a1, a6}, A6{a6, a2} Trường hợp 7người với số điểm 21

A1{a1, a2, a4};A2{a2, a3, a5};A3{a3, a4, a6};A4{a1, a3, a7}, A5{a5, a1, a6}, A6{a6, a2, a7}, A7{a7, a4, a5}

Bài toán giải

Bài 49 Trong buổi thi chọn đội dự tuyển chuẩn bị cho kỳ thi chọn học sinh giỏi quốc gia Trường THPT chuyên X, giáo viên phát 5đề thi khác cho nhóm gồmn học sinh ngồi quanh bàn trịn cho khơng có hai học sinh ngồi cạnh có đề thi (n ∈ N, n >2) Hỏi có học sinh tham gia dự thi? Biết số cách phát đề 1048580

Lời giải

Gọi Sn số cách phát đề cho học sinh cho khơng có hai học sinh ngồi cạnh có

đề thi Cố định học sinh làm vị trí học sinh bên phải học sinh vị trí thứ 2, thứ 3, , thứ n (học sinh vị trí thứ n ngồi cạnh học sinh vị trí thứ nhất) Ta thấy,

Nếu học sinh vị trí thứ học sinh vị trí thứ n−1 có đề thi khác có

cách phát đề cho học sinh vị trí thứn

Nếu học sinh vị trí thứ học sinh vị trí thứn−1có đề thi giống có4cách phát đề cho học sinh vị trí thứ n Do ta có hệ thức Sn = 3Sn−1+ 4Sn−2 (n >4)

Xét phương trình đặc trưng x2−3x−4 = ⇔

"

x=−1

x= ⇒Sn=a(−1) n

+b4n Do S2 = 5·4 = 20, S3 = 5·4·3 = 60 nên

(

a+ 16b = 20

−a+ 64b = 60 ⇔

(

a =

b = ⇒Sn= (−1) n

+ 4n= + 410⇒n= 10

Vậy có10 học sinh tham gia dự thi

Bài 50 (IMO Shortlisted 2001, problem 12) Với số nguyên dươngn, gọi dãy gồm toàn số cân chứa n số0 n số1 Hai dãy cân a b gọi hàng xóm chuyển vị trí trong2n ký tự củaa đểa chuyển thành b (VD: 01101001

chuyển thành 00110101 cách chuyển số thứ (hoặc thứ 3) sang vị trí thứ 2) Chứng minh có tậpS chứa

n+

2n n

dãy cân cho dãy cân với hàng xóm với dãy trongS

Lời giải

Vì đề có đề cập đến số n+

2n n

nên ta có ý tưởng phân chia

2n n

dãy cân thànhn+ 1lớp, chọn lớp có số phần tử nhỏ (sẽ nhỏ

n+

2n n

(55)

mọi dãy cân thuộc lớp hàng xóm với 1phần tử lớp Quan trọng cách phân lớp cho phù hợp Do có nhận xét

2n n

chia hết cho n+ nên nghĩ

đến cách chia lớp theo kiểu đồng dư modulo n+ 1thì lớp có n+

2n n

phần tử Quan trọng xem xét thay đổi chuyển vị trí để dựa vào làm chia lớp theo modulo n+ Ta bắt đầu với trường hợp nhỏ lẻ Trước hết, nghiên cứu thay đổi di chuyển kí tự dãy cân Xétn = 3, ví dụ với dãy ban đầu 000111

Số từ vị trí thứ sang vị trí thứ k dãy 000111 không đổi k = 2,3, thành 001011 sang vị trí thứ 4, 001101 sang vị trí thứ 5, sang 001110 sang vị trí số Vị trí số mơ tả lại sau

(4,5,6)−→(3,5,6)−→(3,4,6)−→(3,4,5)

Nếu dãy ban đầu 000111 di chuyển số Di chuyển số từ vị trí cuối sang vị trí thứ k, với k= 5,4 khơng có thay đổi Vớik = dãy 001011;k = được010011; k =

được100011 Vị trí số1 sau

(4,5,6)−→(3,5,6)−→(2,5,6)−→(1,5,6)

Di chuyển số từ vị trí thứ sang vị trí thứ 6, khơng có thay đổi, sang vị trí thứ 3, ta được001011, thứ 2được 010011, sang vị trí thứ1 được100011, vị trí thay đổi sau

(4,5,6)−→(3,5,6)−→(2,5,6)−→(1,5,6)

Khi có thay đổi? Chỉ có vị trí số1trong dãy thay đổi Xét theo modulo 4thì có thay đổi theo quy luật? Chỉ có tổng vị trí số thay đổi, thân số xét theo modulo lại thay đổi khơng theo quy luật Xét tổng vị trí số với việc di chuyển số với dãy ban đầu, ta thu số 15,14,13,12là 1hệ thặng dư đầy đủ modulo Với việc di chuyển số1, ta thu số: 15,14,13,12cũng hệ thặng dư đầy đủ modulo Với việc di chuyển số1

theo cách2, ta thu số tương tự Tiếp theo, xem xét kết n= 2, ta có

4

= dãy

sau: 1100; 1010; 1001; 0011; 0101; 0110, ta thấy 1100 nhận 1010; 1001 làm hàng xóm, tổng vị trí số1trong 3dãy là: 3,4,5 0011nhận 0101; 0110làm hàng xóm, tổng vị trí số 1trong3 dãy là:7,6,5 Xét theo modulo số lập thành hệ thặng dư đầy đủ modulo 3, chọn tập S ={1100; 0011} thìS tập thỏa mãn Như vậy, có khả ta chia lớp theo tổng vị trí số1trong dãy với modulon+ 1, tổng quát từ trường hợpn = 2, ta chia thành nhiều hệ thặng dư đầy đủ modulo n+ 1, hệ chọn dãy cân đại diện, với dãy cân đó, hàng xóm quét thành hệ thặng dư đầy đủ Như thế, ta chọn hệ phần tử theo lớp thặng dư để chia thành n+ lớp, nhiều khả cách phân lớp thỏa mãn Suy nghĩ theo hướng vậy, cuối cùng, ta có lời giải sau: Với dãy cân bằnga= (a1, a2, , a2n), xétf(a)

là tổng sốj cho aj = Ví dụ f(10010011) = + + + = 20 Phân chia

2n n

dãy cân thànhn+ lớp

Ai ={a/f(a)≡i (mod n+ 1)}, i= 1,2, , n+

Gọi S lớp có số phần tử

|S|6

n+

2n n

Ta chứng minh dãy cân bằnga thuộc S hàng xóm với phần tử củaS Xét

(56)

i) Nếua1 = Di chuyển số 1về phía bên phải số0thứ k, ta thu dãy cân bằngb hàng xóm a với f(b) = f(a) +k Vậy a /∈S tồn số k ∈ {1,2, , n}mà b∈S

ii) Nếu a1 = Di chuyển số phía bên phải số thứ k ta hàng xóm b a mà f(b) = f(a)−k Vậy tồn tạik ∈ {1,2, , n} mà b ∈S

Ta có điều phải chứng minh

Bài 51 Có n bóng đèn A1, A2, , An (n > 2, n ∈ N) xếp thành hàng ngang,

bóng đèn có hai trạng thái bật tắt Ở thời điểm ban đầu, bóng đèn A1 bật cịn bóng đèn khác tắt Cứ sau giây, bóng đèn thay đổi trạng thái sau: bóng đèn Ai (i = 1,2, , n) có trạng thái với bóng đèn kề với Ai tắt;

trường hợp khácAi bật lên (trong đó, bóng đèn A1 An có bóng đèn kề

với nó) Chứng minh

1 Nếu n= 2m, m∈

N∗ đến thời điểm tất bóng đèn bật

2 Tồn vô hạn giá trịn cho thời điểm, tất bóng đèn khơng thể bật tắt

Lời giải

1 Khơng tính tổng quát, ta coi bóng đèn xếp thành hàng ngang cách Gọi O điểm A1An Ta chứng minh n = 2m, m ∈N∗ sau n−1 = 2m−1

bước bóng đèn bật phương pháp quy nạp theo m Dễ thấy mệnh đề với m= 1, m =

Giả sử mệnh đề đến m, ta chứng minh mệnh đề với m+ 1 Thật vậy, giả sử k = n

2 =

m bóng đèn bên trái A

1, A2, , Ak bật sau k−1 bước Để ý k bóng

đèn bên phảiAk+1, Ak+2, , Ankhông thay đổi trạng thái suốtk−1bước ban đầu

Đến bước thứ k, tất bóng đèn tắt trừ hai bóng Ak, Ak+1 bật Sau bước này, tính chất đối xứng, bóng đèn đối xứng qua tâmO có trạng thái Xétkbóng đèn bên phảiAk+1, Ak+2, , An, sau bước thứk, cóAk+1bật cịn bóng khác tắt Trạng thái trùng với trạng thái ban đầu k bóng đèn bên trái A1, A2, , Ak Như sau bước thứ k+j dãy k bóng đèn bên phảiAk+1, Ak+2, , An

trùng trạng thái với dãykbóng đèn bên tráiA1, A2, , Ak sauj bước Theo giả thiết quy

nạp, sau bước thứ k+ (k−1) =n−1, tất bóng đèn bên phải Ak+1, Ak+2, , An

đều bật Do tính đối xứng qua tâm O, k bóng đèn bên trái A1, A2, , Ak bật

Như vậy, tất n bóng đèn bật sau bước thứ n−1 Phép chứng minh quy nạp hoàn tất

2 Ta chứng minh, n = 2m + 1, m ∈ N∗ thì thời điểm, bóng đèn khơng thể có trạng thái Lập luận tương tự phần a), ta thấy, suốt n−2 = 2m −1 bước

ban đầu, khơng có thời điểm mà bóng đèn A1, A2, , An có trạng thái Sau

bước thứ n−1 = 2m, có hai bóng đèn An−1 An bật, bóng cịn lại tắt Khi đó,

dãy (An, An−1, , A2, A1) trùng trạng thái với dãy(A1, A2, , An−1, An) sau bước thứ

Như vậy, trạng thái dãy(A1, A2, , An−1, An)sau bước thứ j trùng với trạng thái dãy (A1, A2, , An−1, An)sau bước thứn−2+j Suy , trạng thái dãy(An, An−1, , A2, A1) sau bước thứj trùng với trạng thái dãy (A1, A2, , An−1, An) sau bước thứ2(n−2) +j

Vậy trạng thái dãy (A1, A2, , An−1, An) tuần hoàn với chu kỳ nhỏ 2(n−2)

(57)

Bài toán giải Bài 52 Mười hai hiệp sĩ ngồi quanh bàn tròn Mỗi hiệp sĩ ghét hai hiệp sĩ ngồi bên cạnh anh ta, không ghét số hiệp sĩ khác Một nhóm gồm năm hiệp sĩ cử bảo vệ người yêu admin Minh Tuấn Hai hiệp sĩ ghét khơng vào nhóm Hỏi lập nhóm hiệp sỹ bảo vệ girl xinh

Lời giải

Ta chứng minh toán tổng quát

Tổng quát Cho n > người ngồi quanh bàn tròn Mỗi người ghét hai người ngồi hai bên cạnh Hỏi có cách chọn nhóm gồmm

m

hn

2

i

người cho nhóm khơng có hai người ghét Cho tương ứng theo thứ tự từ trái sang phải người ngồi bàn tròn với trongn số nguyên dương Bài toán đặt tìm số tập có m phần tử tập hợp n số nguyên dương cho khơng có hai phần tử có khoảng cách

hoặcn−1 GọiA, B theo thứ tự họ tập cóm phần tử tậpnsố nguyên dương cho khơng có hai phần tử có khoảng cách 1, 1và tồn hai phần tử có khoảng cách bằngn−1 Rõ ràng số tập thỏa mãn bằng|A| − |B| Ta tính lực lương củaA vàB

1 Tính lực lượng củaA Gọi{a1, a2, , am}là phần tử họ A Khơng tính tổng qt

giả sửa1 < a2 < < am, ai+1 > ai+ 1,∀i= 1, n−1nên

1a1 < a2−1< a3−2< < am−m+ 1n−m+

Nếu đặtbi =ai−i+ 1thì dễ thấy {b1, b2, , bm} tập tập n−m+ số nguyên

dương Xét ánh xạ từ tập A đến họ tập có m phần tử tập n−m+

số nguyên dương dương sau

(a1, a2, , am)→(b1, b2, , bm)

Dễ dàng chứng minh đay song ánh nên

|A|=

n−m+

m

2 Tính lực lượng tập B Vì phần tử họB có chứa hai phần tử có khoảng cách n−1 nên {a1, a2, , am} ∈B a1 < a2 < < am a1 = 1, am =n Do

3a2 < a3 < < am−1n−2 Xây dựng song ánh tương tự ta có

|B|=

n−m−2

m−2

Vậy số tập cần xác định là|A| − |B|=

n−m+

m

n−m−2

m−2

Bài 53 Cho tập hợp T gồm n phần tử(n ∈N∗) và F(T) là họ tất tập khác nhau T thỏa mãn tập có3 phần tử khơng có hai tập rời Tìm giá trị lớn |F(T)| (Hai tậpA vàB gọi rời nếuA∩B =∅ và|A| ký hiệu số phần tử tập hợpA)

Lời giải

(58)

Nếu16n62 thì F(T) =∅⇒k(n) = 0.

Nếu36n65thì tất tập gồm 3phần tử củaT đều đôi có phần tử chung, suy rak(n) =

n

3

Nếu n > 6, giả sử T = {1; 2; .;n} Khi xét

n−1

tập gồm phần tử tập

T \ {1}, với tập A gồm 2phần tử T \ {1} A∪ {1} ∈F(T), suy k(n)>

n−1

= (n−1)(n−2)

2

Tiếp theo ta chứng minhk(n)6 (n−1)(n−2)

2 n>6

Giả sử khẳng định k(m) 6

m−1

đúng với 6 m 6 n, xét tập A ∈ F(T), khơng tính tổng qt ta giả sử A={1; 2; 3} Nếu B ∈ F(T) chứa phần tử A

|F(T)| = + 3(n −3) <

n−1

(do n > 7) Nếu có tập B ∈ F(T) chứa phần tử A, ta giả sử B ={1; 4; 5} Nếu tất phần tử F(T) chứa

|F(T)| =

n−1

, ngược lại giả sử tập C ∈ F(T) không chứa 1, A∩C 6=∅, B ∩C 6=∅

nên ta giả sửC ={2; 4; 6} Từ đẳng thức

(n−1)(n−2)

2 −

(n−2)(n−3)

2 =n−2

nên có khơng qn−2tập theo giả thiết quy nạp ta có khẳng định Trái lại, giả sử7 thuộc n−1tập, với tập phải chứa 1,2

• Nếu chứa phải chứa 2,4 6, điều xảy tập hợp {1; 2; 7},

{1; 4; 7} {1; 6; 7}

• Nếu chứa phải chứa 1,4hoặc Như có thêm 2khả tập

{2; 4; 7} {2; 5; 7}

• Nếu chứa phải chứa 1,4,5 có khả băng {3; 4; 7} (vì tập phải giao khác rỗng với tập {1; 4; 5},{2; 4; 6}) Như có khả cho tập chứa Vì n−2>6 ⇔n >8 khơng có q n−2 tập chứa

7 Xét n= T ={1; 2; 3; 4; 5; 6; 7}, ta lý luận tương tự đến kết luận không n−2 tập chứa

Như ta có k(n) =

            

0 16n62

n

3

36n65

n−1

n>6

Bài 54 Chứng minh tồn tập A tập hợp {1; 2; .; 2100−1} thỏa mãn đồng thời điều kiện sau:

(i) 1∈A 2100−1∈A;

(59)

biệt) A;

(iii) Số phần tử A không vượt 109

Lời giải

Với số nguyên dương m, gọi Am tập {1; 2; .;m} có số phần tử nhỏ mà

thỏa (i) (ii) gọi f(m) số phần tử Am Khi đó, ta chứng minh A2100−1 thỏa (iii),

tức f(2100−1)6109 Với mỗi n ∈

N∗\ {1} ta đặt

B =A2n−1∪2n+1−2; 2n+1−1

thì ∈ B 2n+1 − ∈ B tức (i) thỏa mãn Mọi phần tử A2n−1 trừ phần tử thỏa mãn tổng hai phần tử A2n−1 Ngồi 2n+1 −1 = (2n+1−2) +

2n+1−2 = (2n−1) + (2n−1) Cho nên B thỏa (ii) Khi đó, f 2n+1−1

6|B|=f(2n−1) + (1)

Mặt khác, với n ∈N∗\ {1} ta đặt B =A2n−1∪

2(2n−1); 22(2n−1); .; 2n(2n−1); 22n−1

thì tương tự ta có B thỏa (i) Mọi phần tử A2n−1 trừ phần tử tổng hai phần tử A2n−1 Mặt khác,

22n−1 = (2n−1)(2n+ 1) = 2n(2n−1) + (2n−1),

2n(2n−1) = 2n−1(2n−1) + 2n−1(2n−1),

· · · ·

22(2n−1) = 2(2n−1) + 2(2n−1),

2(2n−1) = (2n−1) + (2n−1)

Cho nên B thỏa (ii) Khi đó, f(22n−1)6|B|=f(2n−1) +n+ 1. (2)

Áp dụng(1), (2) ta

f 2100−1 6 f 250−1+ 51 6f 225−1+ 51 + 26 6f 224−1+ 51 + 26 + f 212−1+ 51 + 26 + + 136f(26−1) + 51 + 26 + + 13 + f(23−1) + 51 + 26 + + 13 + +

6 f(22−1) + 105 = + 105 = 108<109

Vậy ta có điều phải chứng minh

Bài 55 Có cách chọn từ2016 số nguyên dương 10sốa1, a2, , a10 cho |ai−aj|>1 với mọii6=j?

Lời giải

Khơng tính tổng qt, giả sử a1 < a2 <· · · < a10 đó, ta đưa giả thiết cho vềai+1−ai >2 vớii= 1,9 Như thế, số không đứng cạnh Gọix1 số số trước a1, x11 số sau a10, lại xk số số nằm ak−1 ak

1 a1 a2 a3 a10 2016

(60)

Theo giả thiết, ta phải có x1 >0, x11 >0và xi >1 với i= 2,10 Ngoài ra, x1+x2+· · ·+x11= 2006

Đặt yi =xi−1>0, i= 2,10 ta đưa phương trình

x1+y2+· · ·+y10+x11= 1997 Số nghiệm phương trình

1997 + 11−1 11−1

=

2007 10

Dễ thấy nghiệm cho ta cách chọn bộ10 số thỏa mãn đề Do đó, số cần tìm

2007 10

Bài 56 Gọianlà số xâu nhị phân độ dài nkhông chứa ba bit 0,1,0liên tiếp Gọibnlà số

các xâu nhị phân độ dài n không chứa bốn bit 0, 0,1, 1, 1, 0, liên tiếp Chứng minh bn+1 = 2an với số nguyên dương n

Lời giải

Gọi An, Bn tập xâu nhị phân độ dài n thỏa mãn điều kiện thứ thứ hai Với

mỗi xâu nhị phân (x1, x2, , xn) ta cho tương ứng với xâu nhị phân (y0, y1, , yn) xác định

bởi y0 =

yi =x1+x2+· · ·+xi mod 2, i= 1,2, , n (2.3)

Khi

xi =yi−yi−1 mod 2, i= 1,2, , n

Dễ thấy (2.3) song ánh tập tất xâu nhị phân độ dài n tập tất xâu nhị phân độ dàin+ có bit Hơn nữa, xâu nhị phân (x1, x2, , xn)có ba bit 0, 1, liên thứ tự xâu nhị phân tương ứng (y0, y1, , yn) có bốn bit liên

tiếp theo thứ tự 0, 0,1,1 hoặc1,1, 0, Nói cách khác xâu nhị phân thuộcAn tương ứng

với xâu nhị phân thuộc Bn+1 bắt đầu bit Vì số xâu nhị phân thuộc vào Bn+1 bắt đầu bit nửa số xâu nhị phân thuộc vào Bn+1 ta có bn+1 = 2an (điều phải

chứng minh)

Bài 57 Gọi M số nguyên dương hệ thập phân có 2n chữ số có n chữ số

và n chữ số2 Gọi N số nguyên dương cón chữ số hệ thập phân có chữ số 1, 2,3, 4và số chữ số số chữ số2 Chứng minh |M|=|N|

Lời giải

Ta chứng minh có song ánh từ N vào M Lấy số thuộc vào N, ta cho tương ứng với số thuộc vào M theo cách: viết hai số liền ta số có2n chữ số, sau chữ số3 ởn chữ số đổi thành chữ số 1, chữ số ởn chữ số đổi thành chữ số2 Tương tự, chữ số4ởn chữ số đổi thành chữ số 2, chữ số n chữ số đổi thành chữ số Với cách làm ta thu số thuộc vàoM hiển nhiên đơn ánh Để chứng minh song ánh ta xây dựng ánh xạ ngược sau: Với số có n chữ số n chữ số ta cắt đôi n chữ số đầu n chữ số cuối sau đặt chúng song song thực phép cộng sau

1 + = 1,2 + = 2,1 + = 3,2 + =

Khi ta thu số thuộcN Như có song ánh hai tậpM vàN nên|M|=|N|

(61)

Bài 58 Cho X ={1,2, , n} Một tập A X gọi tập béo phần tử A không nhỏ số phần tử Tập rỗng tập béo Đặt an số

tập béo X mà tập không chứa hai số liên tiếp, bn số tập X mà

hai phần tử 3đơn vị Chứng minh an =bn

Lời giải

Gọi A học tập béo thỏa mãn yêu cầu đề bài, B họ tập X có tính chất hai phần tử đơn vị Ta thiết lập ánh xạ f từ A đến B sau: giả sử x={a1, a2, , ak} ∈A, ta giả sử k6a1 < a2 <· · ·< ak 6n

Đặt b1 =a1−k+ 1, b2 =a2 −k+ 2, , bk =ak Khi

ai+1 >a1+ 2, i= 1,2, , k−1

Suy ai+1−ai > bi+1 −bi > b1 > 1, bk n Định nghĩa f(x) = y = {b1, b2, , bk},

suy y∈B Vậyf ánh xạ, dễ thấyf song ánh, ta có điều cần chứng

minh

Bài 59 Với n, xét tập hợp Sn = {1,2,3, , n} Tìm số tập hợp khác rỗng Sn

không chứa 2số nguyên liên tiếp tùy theo n

Lời giải

Trước hết, ta bắt đầu với trường hợp nhỏ Với n= 1, ta được1 tập hợp thỏa mãn là {1}. Với n= 2, ta được2 tập hợp thỏa mãn {1};{2}

Với n= 3, ta được4 tập thỏa mãn {1};{2};{3};{1,3}

Với n= 4, ta được7 tập thỏa mãn {1};{2};{3};{4};{1,3};{1,4};{2,4} Với n= 5, ta được12 tập thỏa mãn

{1};{2};{3};{4};{5};{1,3};{1,4};{1,5};{2,4};{2,5};{3,5};{1,3,5}

Bây giờ, quan sát thay đổi tập thu n tăng dần, ta thấy n tăng lên đơn vị, tập thu ứng với giá trị n trước đó, cịn tập tạo cách thêm số n vào, nhiên phép thêm vào tập không chứa sốn−1 Chẳng hạn với n= 4, tập là{1};{2};{3};{4};{1,3};{1,4};{2,4} Thì với n = 5, có tập đó, thêm vào tập

{5}, tạo từ tập ∅ bằng cách thêm số 5.

{1,5};{2,5};{3,5}, tạo từ tập {1};{2};{3} bằng cách thêm vào số 5. {1,3,5}, tạo từ tập{1,3}bằng cách thêm vào số 5.

Những tập chứa số không tạo tập thỏa mãn Như vậy, tổng quát hóa suy nghĩ này, ta thu lời giải nhờ truy hồi (từ trường hợp trước tạo trường hợp sau) sau Gọian số tập

con tậpSn thỏa mãn đề bài, ta có a1 = 1, a2 = 2, a3 = 4, a4 = 7, a5 = 12 Xét với tập Sn+1, tập thỏa mãn thuộc hai loại

(62)

Loại 2. Chứa n+ 1, tập loại tạo nhờ thêm vào n+ 1 từ tập của Sn thỏa mãn đề bài, tập khơng chứa n, tập Sn−1 thỏa mãn đề Ngồi có thêm tập{n+ 1} Vậy có an−1+ tập loại

Từ ta

a1 = 1, a2 =

an+1 =an+an−1+ n>2 Giải phương trình sai phân thu trên, ta được:

an=

5−3 2√5

1−√5

!n

+

5 + 2√5

1 +√5

!n

−1

Với kiểu lập luận tương tự trên, ta có lời giải cho tốn sau Bài 60 Xét hoán vị (x1, x2, , xn) tập Sn = {1,2, , n} Vị trí i,1 i n

gọi vị trí cực đại xi > xi−1, xi > xi+1 (vị trí n khơng phải vị trí cực đại) Gọi p(n, k)là số hốn vị có k vị trí cực đại Chứng minh

p(n+ 1, k+ 1) = (2k+ 4)·p(n, k+ 1) + (n−2k+ 1)·p(n, k) Lời giải

Nhận xét Nếui vị trí cực đại của(x1, x2, , xn)thì i−1, i+ 1khơng phải vị trí cực đại Gọi p(n, k)là số hốn vị củaSk có k vị trí cực đại Xét(x1, x2 , xn) hốn vị củaSn, cók

vị trí cực đạii1 < i2 <· · ·< ik Đưax1, x2 , xn vào ô sau (xi ln đứng hai trống,

cón+ ô trống)

x1 x2 xi1−1 xi1 xi1+1 xn−1 xn

Khi đưa n + vào hoán vị (x1, x2, , xn) để thành hoán vị Sn+1, ta đặt n+ vào n+ ô trống Ta có trường hợp sau

Trường hợp Nếu n+ đặt vào ô trống cuối số vị trí cực đại hốn vị khơng đổi (khơng ảnh hưởng đến vị trí cực đại)

Trường hợp Nếu n+ đặt vào 2k ô trống cạnh i1, i2, , ik tạo hốn

vị có đúngk vị trí cực đại (thêm vị trí cực đại vị trí củan+ 1nhưng bớt vị trí cực đại ik n+ đứng cạnh ik, nghĩa số vị trí cực đại không đổi)

Trường hợp Nếu n+ 1được đặt trống cịn lại (có n−2k−1ơ) làm tăng thêm vị trí cực đại, vị trí n+

Vậy hốn vị Sn có k vị trí cực đại tạo 2k+ hốn vị Sn+1 có k vị trí cực đại n−2k−1hốn vị Sn+1 cók+ 1vị trí cực đại cách thêmn+ 1vào giữa.n!hốn vị Sn

mỗi hoán vị tạo ran+ 1hoán vị Sn+1 cách thêm n+ 1vào giữa, hoán vị Sn khơng

tạo hốn vị Sn+1, n! hoán vị chia làm loại Loại Có k vị trí cực đại: p(n, k) hốn vị

Loại Có k+ vị trí cực đại: p(n, k+ 1) hốn vị

Loại Có khơng q k−1 vị trí cực đại

Mỗi hoán vị loại cho n−2k−1 hoán vị Sn+1 có k+ ví trí cực đại Mỗi hốn vị loại cho

2(k+ 1) + = 2k+ 4hốn vị Sn+1 cók+ vị trí cực đại Mỗi hốn vị loại3 khơng cho hốn vị củaSn+1 cók+ 1vị trí cực đại Vậyp(n+ 1, k+ 1) = (2k+ 4)·p(n, k+ 1) + (n−2k+ 1)·p(n, k)

(63)

Bài 61 (German MO 1996) Cho viên đá vị trí (1; 1)trên trục tọa độDescartes dịch chuyển theo nguyên tắc sau

a) Khi vị trí (a;b) dịch chuyển tới vị trí(2a;b)hoặc (a; 2b)

b) Khi vị trí (a;b) dịch chuyển tới vị trí(a−b;b) a > b dịch chuyển tới vị trí (a;b−a) b > a

Với cặp số ngun(x;y) hịn đá dịch chuyển tới?

Lời giải

Kí hiệugcd (x;y) ước chung lớn x y Ta có nhận xét

Nếu sử dụng nguyên tắc(a) thì ước chung lớn hai số gấp đôi ước chung lớn hai số cũ

Nếu sử dụng quy tắc(b) thì ước chung lớn hai số ước chung lớn nhất hai số cũ

Giả sử vị trí (x;y) tới vị trí (a;b) nhận xét phải tồn số tự nhiên n để gcd (x;y) = 2n.gcd (a;b) Với tốn có a = b = 1 nên gcd (x;y) = 2n Bây ta chứng minh rằng, với cặp số (x;y) có gcd (x;y) = 2n thì viên đá có

thể dịch chuyển tới sau hữu hạn bước Thật vậy, tập hợp vị trí ban đầu tới vị trí

(x;y), xét vị trí (a;b)có a+b nhỏ

Nếuahoặcb chẵn trước vị tría

2;b

hoặc

a;b

cũng thỏa mãn có tổng nhỏ a+b, điều trái với giả sử a+b nhỏ Do đó, a b số lẻ

Nếu a > b a < b trước ta vị trí

a+b

2 ;b

hoặc

a;a+b

(dùng nguyên tắc (a) dùng nguyên tắc (b) thỏa mãn có tổng nhỏ a + b, điều trái với giả sử a +b nhỏ Do đó, ta có a = b Theo nhận xét ta có

∃m∈ N: 2n = gcd (x;y) = 2m.gcd (a;b) a, blẻ a =b nên a =b = gcd (a;b) = Điều đó, chứng tỏ ta vị trí (1; 1)đến vị trí (x;y)

Như vậy, kết luận toán gcd (x;y) = 2n Bài 62 Vé xe buýt có dạng abcdef với a, b, c, d, e, f ∈ {0,1,2, ,9} Một vé thỏa mãn điều kiện a+b+c=d+e+f gọi “vé hạnh phúc”

1 Chứng minh số nghiệm phương trình a+b+c=d+e+f số nghiệm phương trình a+b+c+d+e+f = 27 với 06a, b, c, d, e, f 69

2 Tính số vé hạnh phúc Lời giải

1 Gọi A, B số nghiệm phương trình a+b +c = d +e+f phương trình a+b+c+d+e+f = 27 Ta thấy

a+b+c+d+e+f = 27⇔a+b+c= (9−d) + (9−e) + (9−f)

(64)

2 Đếm số nghiệm phương trình

a+b+c+d+e+f = 27, (2.4)

với 6 a, b, c, d, e, f 6 Gọi A1, A2, A3, A4, A5, A6 tập hợp nghiệm phương trình (2.4) có điều kiệna >10, b>10,c>10, c>10, d>10,e>10, f >10 Số nghiệm (2.4)

27 + 6−1 6−1

= 32

nên ta cần tính

32

− |A1∪A2∪ · · · ∪A6| XétA1: Đặt a0 =a−10>0thì ta cóa0+b+c+d+e+f = 17 dễ thấy |A1|=

22

5 Tương tự,

ta có |A2|=|A3|=|A4|=|A5|=|A6|=

22

XétA1∩A2: Đặta0 =a−10>0,b0 =b−10>0thì ta cóa0+b0+c+d+e+f = dễ thấy

|A1∩A2|=

12

Ngoài ra, ta thấy rằng|Ai∩Aj∩Ak|= với 16i < j < k 66nên

|A1∪A2∪ · · · ∪A6|=

X

i=1

|Ai| −

6

X

16i<j66

|Ai∩Aj|= 6·

22

−15·

12

Vậy số số may mắn

32 − 6· 22

−15·

12

= 55252

Bài toán giải

Bài 63 Có dãy gồm 100 giống đặt bàn Đạt cô giáo thưởng

5 số Đạt ghét số 5nên bạn muốn chọn cho khơng có hai liên tiếp chọn có thứ tự chênh lệch lượng chia hết cho Biết Đạt chọn hai hai đầu dãy Hỏi Đạt có tổng cộng cách chọn thêm3 theo ràng buộc trên? Ví dụ cách chọn Đạt thỏa mãn

1, 10, 19, 28,100 với chênh lệch là9, 9,9, 72

Lời giải

Gọix,y,z,tlà số nằm có thứ tự1−2,2−3,3−4,4−5 Ta cóx+y+z+t= 95

và x+ 1, y+ 1, z + 1, t+ không chia hết cho Đặt a, b, c, d số dư x, y, z, t chia cho a, b, c, d ∈ {0; 1; 2; 3} a+b+c+d chia hết cho Viết x = 5X+a, y = 5Y +b, z = 5Z +c, t= 5T +d X+Y +Z +T = 19− a+b+c+d

5 , với X, Y, Z, T >0 Đến đây, ta có

các trường hợp sau (giả sử a>b>c>d cho gọn nhân với hoán vị lặp)

1 Nếua+b+c+d= có tất cả1

2 Nếua+b+c+d= có40

3 Nếua+b+c+d= 10 có10

Vậy đáp số toán

22 + 40 21 + 10 20

= 66140cách chọn

Bài 64 Trên khoảng sân hình chữ nhật lát viên gạch hình vng, hàng ngang có 40viên gạch, hàng dọc có 30 viên gạch Các hàng ngang đánh số từ đến 30, hàng dọc dược đánh số từ 1đến40 Trên viên gạch (i;j) hàng thứi cột thứj có ếch Một cào cào dự định thực chuyến thám hiểm: Xuất phát từ viên gạch (1; 1)

(65)

viên gạch nhảy qua lần khơng qua viên gạch có ếch

1 Chứng minh ếch viên gạch (19; 19) cào cào khơng thể thực chuyến thám hiểm

2 Tìm vị trí viên gạch có ếch để cào cào thực chuyến thám hiểm

Lời giải

1 Ta tô màu ô vng hình chữ tương tự bàn cờ vua, ô(1; 1) màu đen ô(i;j)

với i+j chẵn màu đen, cịn lại màu trắng Do bảng 30×40nên số ô đen số ô trắng Ban đầu, cào cào ô đen kết thúc ô đen Mỗi bước nhảy luân phiên từ ô đen sang ô trắng từ ô trắng sang ô đen Vì số đen cào cào qua nhiều so với số ô trắng Để cào cào qua ô lần trừ có ếch ếch phải trắng Do ơ(19; 19) có19 + 19 = 38 chẵn nên đen, cào cào khơng thể thực chuyến thám hiểm, điều phải chứng minh

1 2l−1 40

30

2k

2k−1

1

E

C

2 Ta chứng minh ếch ô (i;j) mà i+j lẻ cào cào thực chuyến thám hiểm Khơng tính tổng qt ta xét ếch ô (2k; 2l−1),16 k 615,16 l6 20 Cào cào dích dắc từ ơ(1; 1) đến ơ(2k−1; 1) theo hàng ngang (chiều mũi tên) Trên 2hàng

2k−1 2k, cào cào dích dắc (chiều mũi tên), vịng qua (2k; 2l−1)có ếch để đến

(2k; 40) Từ ô (2k+ 1; 40) đến ô (30; 40) cào cào dích dắc theo hàng ngang hoàn thành chuyến thám hiểm

Vậy cào cào thực chuyến thám hiểm ếch viên gạch (i;j) với

i+j số lẻ

Bài 65 Dãy (an)n∈Z gọi “cấp số cộng hai phía” với số nguyên n

an+1−an =d số (d gọi công sai dãy) Kí hiệuM tập tất cấp số

cộng hai phía với số hạng ngun cơng sai lớn hơn1

(66)

mỗi số nguyên phần tử cấp số cộng

2 Cho m (m ∈ N, m > 2) cấp số cộng thuộc M cho công sai chúng đôi nguyên tố Chứng minh tồn số nguyên phần tử cấp số cộng m cấp số cộng

Lời giải

1 Xét cấp số cộngA1 ={2k}k∈Z,A2 ={3k}k∈ZvàA3 ={6k+ 1}k∈Z, số ngun cịn lại có dạng6k+ phân hoạch theok chẵn k lẻ nằm cấp số cộngA4 ={4k+ 1}k∈Z

và A5 ={12k+ 11}k∈Z Hơn Z=A1∪A2∪A3∪A4∪A5 cấp số cộng A1, A2, A3,

A4, A5 có cơng sai là2, 3,6, 4,12

2 Giả sử tồn m (m∈N∗, m

>2) cấp số cộng Aj ={djk+aj}k∈Z thuộc M (j = 1,2, , m)

trong d1, d2, , dm >1 số đôi nguyên tố Z =A1∪A2· · · ∪Am

Cố địnhj = 1,2, , m ta thấy rằngx∈Aj x≡aj (mod dj) Xét hệ x≡1 +aj (mod dj), j = 1, , m Theo định lí phần dư Trung Hoa, hệ có nghiệm ngun x mà x /∈Aj với j = 1,2, , m Vậy ta có điều cần chứng minh

Bài toán giải

Bài 66 Có số nguyên dương nhỏ 106 mà tổng chữ số bằng 23? Lời giải

Số cần tìm có dạng a1a2a3a4a5a6 với

a1+a2+a3+a4 +a5+a6 = 23, (2.5)

và 6ai 9, i= 1,6 Gọi |Ai|, i= 1,6 tập hợp nghiệm không âm (2.5) > 10

Trước hết, ta thấy số nghiệm (2.5)

23 + 6−1 6−1

= 28

Ta tính

[ i=1 Ai

với ý

|Ai∩Aj ∩Ak|= với 16i < j < k66vì tổng số 23

Tính |A1| Đặt a0

1 =a1−10>0 a01+a2+a3+a4+a5+a6 = 13 Phương trình có số nghiệm

13 + 6−1 6−1

= 18

Tương tự với |Ai|, i= 2,6

Tính|A1∩A2| Đặta0

1 =a1−10>0,a02 =a2−10>0thìa01+a02+a3+a4+a5+a6 = Phương trình có số nghiệm

3 + 6−1 6−1

=

Tương tự với |Ai∩Aj| mà 16i < j66 khác

Do [ i=1 Ai = X i=1

|Ai| −

X

16i<j66

|Ai∩Aj|= 6·

18 −

Vậy số lượng số cần tìm

28 − · − 6· 18 −

= 47712

(67)

Bài 67 Có hai dãy ghế xếp đối diện nhau, dãy có10ghế, ghế dãy đối diện với ghế dãy cịn lại Có19học sinh tham gia trò chơi Ban đầu học sinh ngồi ghế ghế để trống Cứ sau 10giây, học sinh ngồi dãy khơng có ghế trống chuyển sang ngồi ghế trống dãy đối diện Hỏi có tồn hay khơng thời điểm mà toàn học sinh chuyển dãy cặp học sinh đối diện không thay đổi so với ban đầu?

Lời giải

Giả sử có người vơ hình ngồi ghế trống Như lần học sinh đổi chỗ với người vơ hình Đánh số cho 19 học sinh 1,2, ,19 người vô hình 20 cho ban đầu học sinh

1,2, ,10thuộc dãy học sinh11,12, ,20thuộc dãy lại học sinh i đối diện học sinh i+ 10 với mọi16i610

Với hoán vịσ của(1,2, ,20)thì cặp(i, j)được gọi nghịch σnếu σ(i)> σ(j)và i < j Sự xếp học sinh ban đầu cho ta hoán vịσ= (1,2, ,10,11,12, ,20), số nghịch hoán vị là0 Mỗi bước, học sinh 20đổi chỗ với học sinh dãy đối diện ta lại thu hoán vị Giả sử tồn thời điểm mà yêu cầu toán thoả mãn, học sinh 20chuyển sang dãy đối diện nên tổng số bước phải lẻ Ta biết bước, số nghịch hoán vị thay đổi tính chẵn lẻ nên thời điểm số nghịch hoán vị thu phải số lẻ Mặt khác, hoán vị thu đượcσ0 = (α1, α2, , α10, α11, α12, , α20), đó(α11, α12, , α20) hoán vị của(1,2, ,10) (α1, α2, , α10)là hoán vị của(11,12, ,20); cặp học sinhαi, αi+10 cặp ngồi đối diện lúc ban đầu với 16i610 Dễ thấy từσ thu σ0 thơng qua số chẵn lần đổi chỗ hai học sinh sau: i đổi chỗ cho i+ 10 với 16 i 6 10 thêm vào số lần đổi chỗ từ (1,2, ,10) thành (α11, α12, , α20)bằng số lần đổi chỗ từ (11,12, ,20) thành

(α1, α2, , α10) Theo số nghịch σ0 phải số chẵn, mâu thuẫn Vậy không tồn thời điểm mà toàn học sinh chuyển dãy cặp học sinh đối diện không thay

đổi so với ban đầu

Bài 68 Cho n số nguyên dương Người ta muốn tạo sưu tập thẻ mà thẻ có ghi số dạngm!với m số nguyên dương đó, cho với số nguyên dương k 6n!, chọn số thẻ sưu tập mà tổng tất số chi thẻ chọn có giá trị k Tìm giá trị nhỏ số lượng thẻ sưu tập thỏa mãn yêu cầu

Lời giải

Xét sưu tập gồm

2(n

2−n+ 2)tấm thẻ, bao gồmi tấm thẻ mang sối!và thẻ mang số n! Ta chứng minh sưu tập thõa mãn yêu cầu Thật vậy, k = n! = 1.n! Với 6 k < n!, ta chứng minh quy nạp Với k = 1; hiển nhiên Vì 6 k < n! nên chọn t 6 n−1 lớn cho t! 6 k < (t+ 1)!, k = qt! +h với 6 h < t!, 6 q 6 t áp dụng giả thiết quy nạp hta có sưu tập thỏa mãn Xét sưu tập có thẻ thỏa mãn với số

16k 6n!−1, gọi ci số thẻ mang sối!,i= 1,2, , n−1 Nếu ci >i+ 1, ta thay ci thẻ thành

thẻ (i+ 1)! ci−i thẻ i! Khi sưu tập thõa mãn yêu cầu, mâu thuẫn với tính nhỏ

nhất nên ci 6i Hơn

n!−1 = Xdii!6

X

cii!6

X

i(i!) =n!−1

nên ci = i Tức toàn sưu tập có

2n(n−1) thẻ Giả sử có sưu tập gồm

2n(n−1) thẻ

thõa mãn với các16k < n!thì phải chứaithẻ mang sối!,i= 1,2, , n−1khi đók =n!khơng thể tổng số thẻ sưu tập Vậy đáp số

2(n

(68)

Bài 69 Trong tập hợp tập hợp gồm 2016 số ngun dương

{1,2,3, ,2016} có tính chất: hiệu hai phần tử tập hợp ln khác 7, tìm giá trị lớn số lượng phần tử tập hợp

Lời giải

XétT ={1,2, ,11}, gọiS tập T có tính chất yêu cầu Xét xác cặp không đồng thời thuộcS là{1,5},{2,9},{3,7},{4,11},{6,10} NênScó khơng q phần tử Nếu Scó phần tử 8∈S cặp có phần tử thuộcS Khi đó1,46∈S nên 11,5∈S và7,96∈S nên

3,2∈S suy 10,66∈S vô lý Nên tập có số phần tử lớn T làT0 ={1,3,4,6,9} Khi T∗ ={a+ 1, a+ 3, a+ 4, a+ 6, a+ 9} có tính chất T0 Để bổ sung thêm T∗ vào T0 mà thỏa mãn yêu cầu tốn, ta xét tính chất phần tử a Hiệu phần tử T0

1 ±1⇒a6= 3, a6= 5, a6= 6, a6=

2 ±2⇒a6= 2, a6=

3 ±3⇒a6= 4, a6= 7, a6= 10

4 ±5⇒a6=

5 ±6

6 ±8

Nếua= 11 11−6,11−5,11−86∈ {4,7}nê nta bổ sung T10 ={11 + 1,11 + 3,11 + 4,11 + 6,11 + 9} Tiếp tục trình ta thu tập S∗ có số phần tử lớn

S∗ ={k+ 11a|k ∈T0, a= 0,1,2, ,183}

vì 2016 = 183·11 + Số phần tử củaS∗ 183·5 = 927

Bài 70 Trên giá sách có n sách đánh số từ đến n (n ∈N+ cho trước), ban đầu sách xếp theo thứ tự Một thủ thư muốn xếp lại theo thứ tự 1,2, , n từ trái sang phải, quy tắc xếp sau: Chọn sách số số khơng vị trí (tính từ bên phải sang) chuyển sách vị trí nó, ví dụ giá có sách theo thứ tự 3,1,4,2, sau bước chuyển số vị trí ta xếp lại thành 3,2,1,4 Chứng minh người thủ thư hồn thành cơng việc sau 2n lần xếp theo quy tắc.

Lời giải

Nhận xét Cuốn mang số n không chọn, mang số chọn nhiều lần, mang số chọn nhiều lần (sau lần xếp vị trí, mang số vị trí số bị đẩy khỏi vị trí, đến đẩy đến cuối chọn thêm lần nữa) Kí hiệu f(k) số lần chuyển sách tính đến lần cuối để xếp vị trí sách mang số k (tính đến khơng bị dịch chuyển thêm lần nữa) Vì lần sách số 1,2, , k−1 chuyển vị trí số k lại bị đẩy khỏi vị trí nên ta có

f(k)61 +f(k−1) +· · ·+f(1)

Màf(1) = = 20 nên quy nạp ta dễ dàng có f(k)62k−1

hay f(n)62n−1 <2n.

(69)

Lời giải

Rõ ràng n = số tốt Xét n > 2, n chẵn, chọn k = cách biễu diễn n thành tổng số nguyên dương tồn số chẵn, số không nguyên tố cới n nên khơng thỏa mãn Do n lẻ Ngược lại với n lẻ lớn 2, ta chứng minh n tốt Thật vậy, biểu diễn n dạng tổng lũy thừa phân biệt ta có n = 2α1 + 2α2 +· · ·+ 2αj, j < log

2n+ Khi j 6k 6 n, n viết dạng n = 2α1 + 2α2 +· · ·+ 2αk với a

i ∈N, điều suy

ran tổng củak số nguyên dương nguyên tố với n Thật vậy, nếuk < nthì tồn sốai nguyên dương, giả sử a1 Khi đón = 2α1−1+ 2α1−1+ 2α2+· · ·+ 2αk tổng k+ lũy thừa

của

Với k = Chọn2α là lũy thừa 2lớn mà 2α < n, đón = 2α+ (n−2α)là tổng 2số nguyên

tố với n Tiếp tục cách tách trên, 2α tổng k lũy thừa với 1< k < 2α Chú ý 2α > n

2 nên 26 k <

n

2 +

n

2 + >log2n với n >3 nên khẳng định với 26k 6n n lẻ, tức n tốt VậyS = + + +· · ·+ 2015 = 10082+ = 1.016.065 Bài toán

được giải

Bài 72 Chom, n ∈Z+và bảng có kích thướcm×ngồmmnơ vng đơn vị Mỗi vng có khơng q bọ Biết với số nguyên dươngk thuộc tập hợp{1, 2, 3, , 78}, tồn hàng cột bảng có k bọ

a) Tìm giá trị nhỏ m+n

b) Tìm giá trị nhỏ số bọ bảng cho Lời giải

a) Không tổng quát ta giả sử m6n Vì có hàng cột chứa 78 bọ nên n> 78Giả sử m < n

2, có hàng chứa k bọ với k >

n

2 Do số

hàng n

2,hay m>

n

2,mâu thuẫn với điều giả sử Vậym>

n

2, suy ram>39Với

bảng 39×78ta xếp bọ vào (i, j) cho j >i Khi cột k chứa k bọ (k= 1, 39) hàng79−k chứa k bọ (k= 40, 78) Cách xếp thỏa mãn điều kiện tốn, giá trị nhỏ củam+n 39 + 78 = 117

b) Với 52 giá trị k = 27, 78 ta chọn hàng cột chứa k bọ Giả sử ta chọn p hàng q cột, p+q = 52 Gọi T tập bọ p hàng q cột Giả sửt2 số bọ thuộc T mà nằm ô giao củap hàng q cột, đồng thời t1 số bọ cịn lại thuộcT Khi ta cót1+ 2t2 = 27 + 28 +· · ·+ 78 = 2730 Mặt khác, có t2 bọ nằm pq giao p hàng q cột nên t2 6pq

(p+q)2

4 = 676 Vì số

bọ khơng

|T|=t1+t2 = 2730−t2 >2730−676 = 2054

Ta cách xếp có đúng2054 bọ sau: xếp26 +k bọ vào ô hàng k với k= 1, 26; xếp52 +k bọ vào ô k cột vớik = 1, 26; cịn lại khơng chứa bọ Khi đó, cột 26 +k chứa k bọ với k = 1, 26 Khi bọ nằm 26 hàng 26 cột đầu tiên, đồng thời có có 262 = 676 con bọ thuộc ô giao 26 hàng 26 cột Cách xếp bọ thỏa mãn điều kiện toán Vậy giá trị nhỏ số bọ bảng 2054

Bài toán giải

(70)

hoặc cột Biết quân cờ nhìn thấy đúng5 quân cờ khác màu (và số màu) Tìm số lớn qn cờ có hình vuông

Lời giải

Đáp số 1800 quân cờ

Ví dụ đặt 1800 quân cờ xây dựng Ta lấy từ hình vng100×100 hình bao chiều rộng5 Hình bao gồm4hình vng5×5ở4góc và4hình hình chữ nhật5×90 Ta đặt qn cờ vào4 hình chữ nhật Hình bên trái bên ta đặt quân cờ đỏ, hình bên phải bên ta dặt quân cờ xanh Cách rõ ràng thoả điều kiện av2 có90×10 qn đỏ 90×10 quân xanh, tổng cộng q 1800 quân Ta chứng minh số quân cờ

1800 Xét cách dặt quân thoả mãn điều kiện đề Ta gọi dãy (hàng cột) khác màu dãy có đủ hai màu Ta có hai nhận xét bổ ích sau Thứ quân cờ thấy quân cờ đó, dó quân cờ thuộc dãy khác màu Ngồi ra, dãy khác màu chứa quân cờ màu đỏ nên dãy có khơng q qn cờ màu xanh Tương tự dãy khơng có q 5qn cờ màu đỏ, tức dãy dãy khác màu chứa không quá10 quân cờ Giả sử bảng có a cột khác màu b > a dòng khác màu Nếu b 690 tổng số qn cờ khơng vượt q 10a+ 10b 6 1800 Giả sử b >90 Khi b dịng có tối đa dịng 10 qn cờ Mặt khác, quân cờ điều phải thuộc vào dãy khác màu, từclà chúng nằm giao 100−a cột màu 100−b dòng màu Vì 100−b dịng màu có khơng a(6b) quân cờ tổng số quân cờ không

10b+ (100−b)b= (110−b)b 61800

Bất đẳng thức cuối vì110−b+b= 20 + 90 110−b <20<90< b Bài 74 An Bảo chơi trò chơi: họ viết số tùy thích lên bảng thành dịng, người số, An viết trước Sau Bảo “nhường” An điền dấu + – tùy ý vào số viết An thắng kết bảng không chia hết cho số tự nhiên từ 11 đến 18 Bảo thắng xảy trường hợp ngược lại An nói kiểm sốt nhiều hơn, nên chắn chiến thắng bạn có đồng ý khơng? Tại sao?

Lời giải

Bảo người có chiến thuật thắng thật vậy, hai lượt Bảo điền số 11,12,13, ,18 Ta chứng minh Bảo có cách điền lượt cuối để đảm bảo thắng Gọi số Bảo cần điền lượt cuối làx Sau lượt thứ ba An, bỏ qua số Bảo điền (do chúng chia hết cho11,12,13, ,18 Ta có kết thu ( có23 = 8 cách đặt dấu) làa

1, a2, , a8 Do số tính chẵn, lẻ nên có hai loại số dư chia8số cho4 Hơn có 4số đồng dư với theo

mod Có3số đồng dư với theo mod 3theo nguyên lí Dirichlet, giả sử làa1 ≡a2 ≡a3( mod 3) Khơng tính tổng qt giả sử a4 ≡a3(mod4) Đến ta cần chứng minh tồn tạix cho a1 +x, a2 +x, , a8 +x chia hết cho số 11,12,13, ,18 Khi tốn chứng minh với i tồn tạij cho ai−x=−(aj +x)

Theo định lý thặng dư Trung Hoa, ta chọn

a1+x 9;a2 +x 5;a4+x 16;a5+x 11;a6+x 13;a7+x 17;a8+x

(71)

theo quy tắc trên, ta nhận tập X Lời giải

Cách

Do tập X có số phần tử chẵn nên số tập có số phần tử lẻ chẵn Giả sử có 2k, (k ∈ N∗) tập có số phần tử lẻ, chia 2k tập thành k cặp, thực quy tắc chuyển ta đưa tập có số phần tử chẵn Khi ta đưa trường hợp tập có số phần tử chẵn Nếun = 1, toán chứng minh

Xét n > Do 2n 4, suy số tập có số phần tử chẵn khơng chia hết cho phải chẵn Giả sử có2m, (m∈N∗)

tập có số phần tử chẵn khơng chia hết cho Chia chúng thành m cặp thực phép chuyển phần tử theo quy tắc ta thu tập có số phần tử chia hết cho4 Thực tương tự trên, sau hữu hạn bước ta tập có số phần tử chia hết cho2n, ta nhận tậpX.

Cách Gọi M tập hợp tập ban đầu Ta chia tập M thành hai tập TậpP gồm tập có số phần tử chẵn.

TậpQ gồm tập có số phần tử lẻ

Gọi số phần tử tậpQ q Dễ thấy q số không lẻ Ta thấy Nếuq = 0 ta có tập củaM đều có số phần tử bội của 2.

Nếuq = 2k >0 ta chọn hai tập thuộc Q và thực phép chuyển

• Nếu hai tập có số phần tử chúng hợp thành tập có số phần tử bội

• Nếu hai tập có số phần tử 2m+ và2n+ vơi (m > n) sau phép chuyển ta có hai tập hợp có số phần tử 2(m−n) 4n+ bội

Như vậy, sau k phép chuyển(k >0), ta nhận tập hợp có số phần tử bội Gọi M1 tập tập Ta chia tập M1 thành hai tập

TậpP1 gồm tập có số phần tử bội của22

TậpQ1 gồm tập có số phần tử bội 22 cộng thêm

Gọi q1 số phần tử tập Q1 Dễ thấyq1 số chẵn Giả sử q1 = 2k1 Ta thực phép chuyển

Nếu hai tập có số phần tử chúng hợp thành tập có số phần tử bội của

22

Nếi hai tập có số phần tử là 4m1+ 2 và4n1+ 2 với (m1 > n1)thì sau phép chuyển ta được hai tập hợp có số phần tử là4(m1−n1)và 8n1+ bội của22

Như vậy, sau k1 phép chuyển ta nhận tập có số phần tử bội 22 Gọi M2 tập tập Tiếp tục chia tập M2 thành hai tập P2 Q2, thực tương tự Tiếp tục thực tương tự trên, sau hữu hạn bước ta nhận tập có số phần tử bội 2n−1 Dễ thấy rằng, có hai tập hợp có số phần tử là2n−1 Thực hiện

phép chuyển cuối cùng, ta tập hợpX

Bài 76 Xét hai tập hợp A, B ⊂ {1,2, ,22018} thỏa mãn điều kiện ∀a ∈ A, b∈ B, ta có ab+ số phương Chứng minh min{|A|,|B|}62018

(72)

Giả sử A = {a1, a2, , ap}, B = {b1, b2, , bq}, a1 < a2 < < ap, b1 < b2 < < bq, q p Ta so

sánh apbq với ap−1bq−1

Từ (ap−ap−1)(bq−bq−1)>0⇒(1 +apbq)(1 +ap−1bq−1)>(1 +ap−1bq)(1 +apbq−1)

⇒p

(1 +apbq)(1 +ap−1bq−1)>

p

(1 +ap−1bq)(1 +apbq−1)

⇒p(1 +apbq)(1 +ap−1bq−1)>

p

(1 +ap−1bq)(1 +apbq−1) +

⇒(1 +apbq)(1 +ap−1bq−1)>(1 +ap−1bq)(1 +apbq−1) +

p

(1 +ap−1bq)(1 +apbq−1) +

⇒1 +apbq+ap−1bq−1 >1 +apbq−1+ap−1bq+

p

(1 +ap−1bq)(1 +apbq−1) +

⇒1 +apbq+ap−1bq−1 >1 +ap−1bq−1+ap−1bq−1+

p

(apbq−1)(ap−1bq) +

⇒apbq>2

p

(apbq−1)(ap−1bq)⇒apbq>4ap−1bq−1

Từ bất đẳng thức ta

(22018)2 >apb1 >4ap−1bq−1 >42ap−2bq−2 > >4q−2ap−q+2b2 >4q⇒q <2018

Bài toán giải

Bài 77 Cho m số nguyên dương nhỏ 2017, tập A ={x1.x2, , xm} tập hợp

con tập{1,2,3, ,2017} Giả sử với16i6j 6m, ta cóxi+xj 62017thìxi+xj ∈A

Chứng minh x1+x2+ +xm

m >1009 Lời giải

Giả sử x1 > x2 > > xm Nhận xét xi+xm+1−i >2018,∀i= 1,2, , m

Thật vậy, giả sử tồn i∈ {1,2, , m} cho xi+xm+1−i 62017 Suy xi, xi+xm, xi +xm−1, , xi+xm+1−i ∈A

Mặt khác phần tử xm < xm−1 < < xi ∈ A Do |A| > m−i+ +i =m+ 1, điều vơ

lý Do nhận xét chứng minh Sử dụng nhận xét ta

m

X

i=1

(xi+xm+1−i)>2018m ⇔2(x1+x2+ +xm)>2018m⇔

x1+x2 + +xm

m >1009

Bài toán giải

Bài 78 Cho số nguyên dương n S = {1,2, , n} Gọicn số tập S mà chứa

đúng hai số nguyên dương liên tiếp Chứng minh cn =

2nFn+1−(n+ 1)Fn

5 , (Fn)là dãy Fibonacci

Lời giải

Gọi Cn họ tập củaS mà chứa hai số nguyên dương liên tiếp vàAn họ tập

của S (kể tập rỗng) mà không chứa hai số nguyên dương liên tiếp Mỗi tậpC ∈Cn+2 gồm loại Loại Gồm tập chứa n+ 2, n+

Loại Gồm tập không chứa n+

Loại Gồm tập chứa n+ không chứa n+ Ta thấy

1 Nếu C tập loại C khơng chứa n (vì chứa n C chứa hai cặp số nguyên liên tiếp n, n+ 1và n+ 1, n+ 2) Bỏ khỏi C phần tử n+ 1, n+ 2, ta tập {1,2, , n−1} không chứa hai số nguyên dương liên tiếp, suy phần tử củaAn−1 Ngược lại với tập A An−1 tập C = A∪ {n+ 2, n+ 1} tập loại Cn+2

Phép tương ứng song ánh Vậy, số tập loại số phần tử An−1

(73)

2 Mỗi tập loại rõ ràng tập Cn+1 ngược lại Vậy, số tập loại cn+1

3 Nếu C tập loại C khơng chứa n+ Do loại bỏ khỏi C phần tử n+ 2, ta tập củaCn Ngược lại, với tập conB Cn tậpC =B∪ {n+ 2}là tập

loại củaCn+2 Vậy, số tập loại làcn Từ ta có hệ thức sau cn+2 =cn+1+cn+|An−1| (1) Bây ta cần tìm an=|An| Dễ thấy a1 = 2, a2 = 3, a3 = Mỗi tập A∈An+2 gồm loại: Loại gồm tập chứa n+ Loại gồm tập không chứa n+ Nếu A tập loại A khơng chứa n+ 1(vì chứa n+ A chứa hai cặp số nguyên liên tiếp n+ n+ 2) Do bỏ khỏi A phần tử n+ 2, ta tập An Ngược lại với tập B An tập A=B∪ {n+ 2} tập loại An+2 Do số tập loại làan Mỗi tập loại rõ

ràng tập An+1 ngược lại Vì số tập loại làan+1 Do ta có hệ thức an+2 =an+1+an

Mặt khác, với dãy Fibonacci, ta có

Fn+2 =Fn+1+Fn

Vì a1 =F3 = 2, a2 =F4 = 3, a3 =F5 = Ta suy an=Fn+2 (2) Từ (1) (2) ta có hệ thức cn+2 =cn+1+cn+Fn+1 (3) Từ quy nạp sử dụng hệ thức truy hồi dãy Fibonacci cho (3), ta có công thức

cn=

2nFn+1−(n+ 1)Fn

5

Bài toán giải

Bài 79

1 Có số có 2017chữ số lấy từ số 0,1,2,3,4,5,6,7sao cho số chứa số lẻ chữ số số chẵn chữ số2?

2 Cho n điểm mặt phẳng (n>5), khơng có điểm thẳng hàng Chứng minh có khơng

n−3

tứ giác lồi mà tất đỉnh lấy n điểm cho

Lời giải

1 Gọi LCn tập hợp số gồmn chữ số bao gồm số lẻ chữ số1, số chẵn

chữ số2, CLn tập hợp số gồmn chữ số bao gồm số chẵn chữ số 1, số

lẻ chữ số2, LLn tập hợp số gồm n chữ số bao gồm số lẻ chữ số1,

số lẻ chữ số2,CCn tập hợp số gồm n chữ số bao gồm số chẵn chữ số1;

một số chẵn chữ số Ta có |LCn|=|CLn|;|LCn|+|CLn|+|LLn|+|CCn|= 7n Mỗi số

trongLCn+1 có cách

Thêm số3,4,5,6,7vào sau chữ số cuối số LCn

Thêm chữ số vào sau chữ số cuối số LLn

Thêm chữ số 1 vào sau chữ số cuối số trong CCn

Suy |LCn+1|= 5|LCn|+|LLn|+|CCn|= 4|LCn|+|CLn|+|LLn|+|CCn|

⇒ |LCn+1|= 3|LCn|+ 7n

⇒ |LCn+1| −

1 4.7

n+1 = 3

|LCn| − 4.7

n

= 3n

|LC1| −

1 4.7

=−1

4.3 n+1

⇒ |LCn+1|=

1 4(7

n+1−3n+1). Vậy có

2017−32017

(74)

2 Ta chứng minh quy nạp

Với n= 5 cho 5điểm là A, B, C, D, E Ta lấy bao lồi năm điểm A, B, C, D, E nói Có trường hợp sau:

• Nếu bao lồi ngũ giác điểm 4đỉnh tứ giác lồi

• Nếu bao lồi tứ giác đỉnh bao lồi thỏa yêu cầu

• Nếu bao lồi tam giác Chẳng hạn tam giác ABC

Khi D E điểm tam giác (do điểm thẳng hàng) Kẻ tam giác hình vẽ

C A

E M B

P N

Khi E nằm trong tam giác , chẳng hạn tam giác ADM, tứ giác ABDE tứ giác lồi

Giả sử toán với n=k, ta chứng minh toán với n =k+ 1 Lấy k điểm k+ điểm ta có

k−3

tứ giác lồi Cố định 3điểm A, B, C k

điểm nói trên, với điểm thứ k+ làI, ta lấy thêm điểm k−3 điểm lại, J chẳng hạn Khi tồn thêm tứ giác lồi điểmA, B, C, I, J Thật vậy, tồn tứ giác lồi ABCJ

k−3

nói

C B

I

J A

G

I

Khi đỉnhI thuộc trong8miền cạnh hai đường thẳng qua đỉnh đối diện tạo nên Ta ln có tứ giác lồi vớiI, Jlà2trong số đỉnh Có

k−3

cách lấy điểmJ nên có thêm

k−3

tứ giác lồi Vậy có

k−3

+

k−3

=

k−2

tứ giác lồi có đỉnh k+ điểm

(75)

chọn 1đồng, bước chọn2 đồng, , bước thứ 2019 chọn 2019 đồng) Chứng minh sau

2019 bước ln làm cho tất đồng xu cùng1 trạng thái Giả sử ban đầu có

1 đồng sấp, hỏi sau2019 bước đưa tất đồng xu ngửa không?

Lời giải

1 Trước hết chứng minh chuyển tất đồng xu trạng thái Chứng minh quy nạp theo số đồng xu số lẻ Nếu có đồng xu, có điều chứng minh Giả sử với 2k+ đồng xu chuyển tất trạng thái Xét 2k+ đồng xu Nếu có đồng xu C1, C2 không trạng thái, giả sử C1 sấp C2 ngửa Theo giả thiết quy nạp ta chuyển

2k+ đồng xu lại trạng thái sau2k+ bước, giả sử đồng xu sấp Bước thứ 2k + chuyển 2k + đồng xu C1 ngửa, bước thứ 2k + chuyển tất đồng xu sấp, có điều chứng minh Nếu tất đồng xu trạng thái Ta xếp chúng theo thứ tự đường tròn Bước chọn đồng xu thứ 1, bước chọn đồng thứ 3, tiếp tục q trình đến hết2k+ bước Khi tổng số lần thay đổi tất đồng xu là1 + + + (2k+ 3) = (2k+ 3) (k+ 2), điều chứng tỏ đồng xu thay đổi k+ lần, suy chúng trạng thái

2 Chứng minh làm cho đồng xu ngửa Nếu đồng xu ngửa đánh số sấp đánh số −1 Sau 2019 bước, tổng số lần thay đổi tất đồng xu

1 + + + 2019 = 1010.2019 số chẵn, hay số lần thay đổi dấu tất đồng xu chẵn Suy tích tất số sau2019 bước dấu với tích tất số ban đầu Mà ban đầu có đồng sấp nên tích ban đầu âm, suy tích số sau 2019 bước âm, hay thu trạng thái tất ngửa Ta có điều phải chứng minh

Bài toán giải

Bài 81 An Bình luân phiên đánh dấu vng hình vng101×101 An người bắt đầu Một ô đánh dấu hàng với cột với có đánh dấu Ai không thua Hãy xác định người có chiến thuật thắng

Lời giải

Cách Để đơn giản, ta gọi hai người chơi A B thay An Bình Ta chứng minh B có chiến thuật để thắng Điều thay 101 số nguyên dươngn >2 Rõ ràng theo luật chơi có khơng q 2n đánh dấu Vì nên chiến thuật làB tìm cách đánh cuối Chiến thuật là: người trước đánh người sau đánh dịng với cho số cột đánh nhiều Đặc điểm chiến thuật là:

Sau lượt củaA, B thì có thêm hàng có hai đánh; khơng có hàng chứa một ô

Sau lượt A hàng mà A vừa đánh, cịn n−1 chưa đánh B đánh tùy ý vào ô thuộc cột chưa có đánh; tất cột có đánh B chọn cột tùy ý mà có ô đánh

Bằng cách đó, n−1 lượt đầu tiên, A B ln cịn hàng trống cột chưa đầy ô Do đó, saun−1lượt A, B, bảng cịn lại có đặc điểm là:

(76)

Tất cột có đánh (có cột có ơ, có cột có ơ).

Tổng số ô đánh 2n−2 nên gọia, b số cột có ơ, đánh a+b=n a+ 2b= 2n−2

Suy a= 2, b =n−2, nghĩa có hai cột mà có đánh Hai nằm vị trí giao hai cột dịng chưa đánh hai vị trí cuối đánh A, B thay phiên đánh vào hai ô B người đánh cuối nên chiến thắng

Cách Ta có nhận xét rằng:

(1) Nếu lượt đầu tiên, An đánh vào ô (1,51) hàng Bình đánh vào (101,51) hàng cuối Khi đó, hai bạn khơng thể điền vào ô trung tâm Tiếp theo, An đánh dấu Bình đánh dấu vào đối xứng với qua tâm Mỗi An điền Bình điền được, Bình thắng

(2) Nếu lượt đầu tiên, An đánh vào ơ(a, b) ta đưa nhận xét (1) cách xây dựng bảng tương ứng kích thước hàng hàng a, cột 51 cột b đổi chỗ cho Khi đó, An đánh vào Bình đánh vào ô số hàng/cột tương ứng bảng đối chiếu; Bình đánh vào đối xứng chiến lược (1) đánh vào ô có số hàng/cột bảng gốc Dễ thấy đánh dấu hàng cột hai bảng giống

1

2

3

4

5

1

A3

A4 A5

A2 B4

B3 B2

B5 B1

A1

5

2

3

4

1

1

A3 A4

A5

A2 A1

B1 B2

B5 B3

B4

Hình minh họa cho trường hợp5×5 An đánh vào ô (5,5) nước Từ hai nhận xét trên, ta thấy Bình người có chiến lược thắng trò chơi Nhận xét Rõ ràng theo cách thứ tốn khơng với số 101 hay số lẻ mà với bảng vng có kích thước khơng nhỏ Để cảm nhận đốn người có chiến lược thắng cuộc, ta thử với trường hợp n = 2, n = Khi n = 2, ta thấy tất vng đánh dấu nên Bình hiển nhiên thắng Khi n = 3, ta thấy trung tâm có vai trò quan trọng cho chiến lược chơi Chú ý nhiều chiến lược khác thú vị cho Bình để thắng trị chơi

Bài 82 (IMO shortlisted 2005) Cho số tự nhiênn, n >3 Ta đánh số cạnh đường chéo n-giác P1P2· · ·Pn số nguyên dương nhỏ bằngr thỏa mãn:

(i) Mọi số nguyên dương từ đến r đánh số

(ii) Với tam giác PiPjPk, hai cạnh đánh số số cạnh lại đánh

bởi số nhỏ

Hãy thực yêu cầu sau

(77)

2 Với r thu trên, có cách đánh số thỏa mãn Lời giải

Trước tiên, ta kí hiệu |XY|=k đoạnXY đánh số bởik.Ta xét số trường hợp cụ thể sau

Xétn= 3,ta có tam giác Khi đó, ta có hai số nguyên dương phải dùng là

1 2, số dùng cho hai cạnh, số dùng cho cạnh cịn lại nên có 3cách đánh số

Xét n = 4, ta có tứ giác Có tất cạnh đường chéo, nên số nguyên dùng vượt quá6, số 6được đánh đoạn đó, đoạn cạnh hai tam giác khác nhau, nên tồn thêm hai đoạn khác đánh số Như vậy, ba đoạn, buộc phải đánh số từ đến 5, mâu thuẫn Vậyr <6

1 Nếu r= 5,tương tự, phải đánh số 1,2,3,4 cho ba cạnh, mâu thuẫn

2 Nếu r = 4, giả sử |P1P2| = hai tam giác P1P2P3 P1P2P4 cịn có thêm cạnh đánh số 4nữa

• Giả sử |P1P2|=|P1P4|= ba cạnhP2P3, P2P4, P3P4 đánh ba số1,2,3, trái điều kiện (ii)

• Giả sử|P1P3|=|P2P4|= 4thì tam giácP1P3P4, P2P3P4 cạnh đánh số 4, nên có hai cạnh phải đánh 3số Mâu thuẫn

3 Nếu r= 3, dễ dàng đánh số Giả sử P1 đỉnh mà từ có hai cạnh đánh số 3, có trường hợp sau

Trường hợp Có ba đoạn từ P1 đánh số |P1P2|=|P1P3|=|P1P4|= 3.Khi

|P2P3|,|P2P4|,|P3P4| ∈ {1; 2}, có cách đánh số (xem trường hợp n = 3)

Trường hợp Có hai đoạn từ P1 đánh số 3, có cách chọn hai đỉnh tạo hai đoạn Xét cách chọn, giả sử |P1P2| = |P1P3| = 3,|P1P4| < Khi đó, chia tập {P1, P2, P3, P4}thành hai tập A={P2, P3}, B ={P1, P4}thì đoạn nối điểm thuộc tập A với điểm thuộc B phải đánh số 3, {|P2P3|,|P1P4|}={1; 2} nên có cách chọn |P2P3| |P1P4|.Vậy có cách đánh số thỏa mãn

Tổng cộng có cách đánh số thỏa mãn với P1 làm đỉnh trung tâm, có 36 cách đánh số, cách lặp lại2 lần nên cuối có 18cách đánh số thỏa mãn

Dựa vào lời giải trường hợp trên, ta dự đốnr lớn bằngn−1và đồng thời đưa cách đếm trường hợp r=n−1 Từ đó, có lời giải tổng quát sau Xét đỉnhV mà từ cók 62cạnh đánh số r, đỉnh phải tồn tồn tam giác có hai cạnh đánh sốr Gọi Alà tập đỉnh từ V đánh số r thì|A|=k B tập đỉnh cịn lại (gồm cảV) |B|=n−k Khi

(i) Mọi cạnh nối từ điểm thuộc A đến điểm thuộc B đánh số r, giả sử có X thuộc A, Y thuộc B mà |XY|< r Y 6=V |XV| =r,|Y V| < r, điều kiện ii) nên |XY|=r Mâu thuẫn

(ii) Mọi cạnh nối hai điểm A đánh số nhỏ r, X, Y ∈A |XV|=|Y V|=r nên |XY|< r

(iii) Mọi cạnh nối hai điểm trongB đánh số nhỏ hơnr, nếuX, Y ∈B mà |XY|=r

(78)

1 Ta chứng minh quy nạp giá trị lớn củarlàn−1.Giả sử với đa giáck 6n cạnh, số số dùng nhiều k−1 Xét đa giác n+ đỉnh, giả sử V, A, B đỉnh tập nói đến Ta có

r6|A| −1 +|B| −1 + =k−1 + (n+ 1−k−1) + =n Điều phải chứng minh

2 Như trên, đa giác cón cạnh tậpA cók phần tử tậpA có nhiều k−1số dùng tập B có nhiều n−k−1 số dùng Vậy để đánh số từ đến n−1 cho đoạn tạo từ đa giác, ta phải chọn k−1số số {1; 2;· · · ;n−2} để đánh số cho đoạn tạo từ điểm tập A n−k−1 số cịn lại tập để đánh số cho đoạn tạo từ điểm thuộc tậpB Mặt khác, số cách chọn hai tập A, B cho

|A| = k,|B| = n−k

n k

Từ ta có hệ thức truy hồi sau (với f(n) số cách đánh số

trong trường hợp đa giác cón cạnh)

2f(n) = n

X

k=1

n k

·

n−2

k−1

·f(k)·f(n−k)

=n!(n−2)! n−1

X

k=1

f(k)

k!(k−1)!

·

f(n−k) (n−k)!(n−k−1)!

Có 2f(n) theo cách đếm vậy, số

n k

·

n−2

k−1

·f(k)·f(n−k) tính lần Đặtf(x) = x!(x−1)!g(x)ta có2(n−1)g(n) =

n−1

X

k=1

g(k)·g(n−k).Đến đây, số trường hợp nhỏ lẻ, ta đốn đáp sốf(n) = n!(n−1)!

2n−1 chứng minh kết quy nạp Bây giờ, với ý b), ta xem xét theo khía cạnh khác, trường hợpn = 4, đánh số ba số1,2,3nhưng ta thử bắt đầu đánh số từ1 Nhận thấy nếu|P1P2|=

|P1P3|,|P2P3|,|P2P4|,|P1P4| khác 1,giả sử |P3P4|= xét tam giác P1P3P4,ta thấy dùng tối đa hai số, chẳng hạn là1 2, tam giác P2P3P4 dùng tối đa hai số, suy phải dùng Ta có |P1P3| = 2,|P1P2| = 1,|P2P3| = Mâu thuẫn điều kiện (ii) yêu cầu đề Vậy khơng cịn cạnh phép đánh số Từ đây, ta không quan tâm đến việc đánh số mà đánh số cho đoạn Nhận xét |P1P2| = nên |P1P3| =|P2P3|,

|P1P4|=|P2P4|.Như cách đánh số cho tam giác P1P3P4 cho tương ứng cách đánh số tứ giác Vậy coiP1 P2 điểm Bài toán kéo trường hợpn = Công việc rốt chia làm công đoạn

(a) Chọn đoạn để đánh số 1: Có

4

= cách

(b) Đánh số tam giác sau coi điểm đầu mút đoạn vừa chọn trùng nhau, có cách Kết có 18 cách Với cách suy luận trên, ta có lời giải thứ hai Gọi an số cách đánh

số dùng đến n−1 số cho đoạn tạo từ n điểm Ta chứng minh tồn cạnh đánh số Thật vậy, giả sử |P1P2| = 1, |P1Pi| =|P2Pi|,∀i6= 1,2

Vậy cách đánh số thỏa mãn |P1P2| = tương ứng với cách đánh số cho n −1 điểm P1, P3, P4,· · ·, Pn Nếu số dùng lần nữa, thìn−1điểm đánh số n−1số, theo câu a), đánh số tối đa n−2 điểm Mâu thuẫn Vậy, có cạnh đánh số Chọn cạnh này, có

n

2

(79)

có thể coi hai đỉnh cạnh đỉnh (lập luận trên), ta cònan−1 cách đánh số với n−1 đỉnh n−2 số Vậy có

an=

n

2

·an−1, a3 = 3, a4 = 18

Dễ dàng có an=

n!(n−1)! 2n−1

Bài toán giải

Bài 83 Một cô thợ săn thỏ tàng hình chơi trị chơi sau mặt phẳng Điểm xuất phát A0 thỏ điểm xuất phát B0 cô thợ săn trùng Sau n−1 lượt chơi, thỏ điểm An−1 cô thợ săn điểm Bn−1 Ở lượt chơi thứ n, có ba điều xảy theo thứ tự đây:

(i) Con thỏ di chuyển cách không quan sát tới điểm An cho khoảng cách An−1 An đúng1

(ii) Một thiết bị định vị thông báo cho cô thợ săn điểm Pn, đảm bảo khoảng cách

giữa Pn An không lớn hơn1

(iii) Cô thợ săn di chuyển cách quan sát tới điểm Bnsao cho khoảng cách Bn−1 vàBn

Hỏi điều sau sai hay đúng: cho dù thỏ có di chuyển điểm thiết bị định vị thơng báo có điểm nào, thợ săn ln chọn cho cách di chuyển cho sau 109 lượt chơi, cô ta khẳng định chắn khoảng cách giữa thỏ khơng vượt q 100?

Lời giải

Nếu câu trả lời “đúng” thợ săn phải có chiến lược mà thỏ di chuyển nào, dù thiết bị định vị có thơng báo điểm nào, đảm bảo khoảng cách thỏ không vượt 100, sau 109 lượt chơi Dưới ta chứng minh điều ngược lại: khi gặp “vận rủi” với điểm thiết bị định vị thơng báo, thợ săn khơng thể có chiến lược di chuyển đảm bảo chắn khoảng cách cô thỏ không vượt 100 sau 109 lượt chơi Gọi dn khoảng cách cô thợ săn thỏ sau n lượt chơi Hiển nhiên, dn > 100

với n <109 thỏ thắng, lượt chơi, cần di chuyển tức (theo đường thẳng) xa thợ săn sau đó, khoảng cách thợ săn ln lớn 100 Tiếp theo, ta chứng minh dn 100 dù thợ săn có di chuyển theo chiến lược nào, thỏ

vẫn có phương án di chuyển cho d2n tăng thêm

2, sau 200 lượt chơi (một khoảng thời

gian vừa đủ để thỏ tận dụng may mắn mà thiết bị định vị mang đến cho nó) Dễ thấy, với phương án di chuyển thỏ, d2n vượt 104 sau

2·104·200 = 4·106 <109

lượt chơi, đó, thỏ thắng Giả sử sau lượt chơi thứ n, cô thợ săn điểm An thỏ

điểmBn Giả sử thỏ chí cịn tiết lộ cho thợ săn biết vị trí thời điểm

(80)

Gọi ` đường thẳng qua hai điểm An vàBn, gọiY1,Y2 điểm có khoảng cách đến ` 1, đồng thời, cách Bn khoảng 200 (xem

Hình 1) Kế hoạch thỏ giản đơn chọn hai điểm, Y1 Y2, mong muốn thẳng tiến đến điểm sau 200 lượt chơi

` dn 200−dn ε

1

1

y

y

200

200

(Hình 1)

Y1

Y2

An Bn A0 M B0

Vì tất điểm mong muốn di chuyển đến (trong 200 lượt chơi) thỏ có khoảng cách đến ` nhỏ hơn1 nên xảy tình điểm mà thiết bị định vị thông báo (trong200 lượt chơi) thuộc ` Trong trường hợp đó, thợ săn khơng có cách để biết thỏ chọn Y1 hay Y2

Câu hỏi đặt ra: Cô thợ săn làm gì, điểm thiết bị định vị thông báo thuộc `? Nếu chiến lược thợ săn đường thẳng mà tiến sau200 lượt chơi, đến điểmA0 Hình Có thể thấy, thợ săn khơng thể chọn chiến lược khác tốt Thật vậy, suốt200 lượt chơi, điểm đến cô thợ săn nằm phía bên trái điểmA0 Vì thế, cô thợ săn chọn chiến lược di chuyển đến điểm nằm phía trên` sau 200 lượt chơi, đến điểm chí nằm xa điểm Y2 hơn, so với A0; trường hợp cô thợ săn chọn chiến lược di chuyển đến điểm nằm phía ` sau 200 lượt chơi, đến điểm chí cịn nằm xa điểm Y1 hơn, so với A0 Nói cách khác, dù thợ săn có di chuyển theo chiến lược khơng tin khoảng cách thỏ nhỏ

y=A0Y1 =A0Y2

sau 200 lượt chơi

Gọi M trung điểm Y1Y2; gọi B0 điểm thuộc ` cáchBn khoảng 200 (xem Hình

1 trên) Đặt ε=M B0 (lưu ý A0B0 =dn) Ta có

y2 = +A0M2 = + (dn−ε)

2

, (1)

ε= 200−BnM = 200−

2002−1

=

200 +√2002−1 >

1

400· (2)

Để ý ε2+ = 400ε (do ε= 200−√2002−1), từ (1) ta có

y2 =d2n−2εdn+ε2+ =d2n+ε(400−2dn)

Từ đó, vìdn 6100 ε >

400 (theo(2)), suy

y2 > d2n+1 2·

Như vậy, tình tất điểm mà thiết bị định vị thông báo thuộc`, dù thợ săn có di chuyển nào, thỏ đạt

d2n+200 > d2n+1 2·

(81)

Bài 84 Gọi S tập {1,2, ,2017} cho S không chứa hai phần tử mà phần tử chia hết cho phần tử không chứa hai phần tử nguyên tố Hỏi S chứa nhiều phần tử?

Lời giải

Xét tập hợp số S = {1010,1012,1014, ,2016} rõ ràng tất chẵn nên khơng có hai số ngun tố Hơn nữa, số lớn nhỏ lần số nhỏ nên khơng có hai số mà số chia hết cho số Khi đó, S thỏa mãn đề có 504 phần tử Ta chứng minh giá trị lớn cần tìm Giả sử ngược lại tồn tập hợpS0 thỏa mãn tính chất đề có 505 phần tử Với số chẵn i từ 1010 −→2016 với dạng i = 2α·m và (m,2) = 1, xét tập hợp A

i có dạng Ai = {2k·m|0 k α} Rõ ràng 2017 nguyên

tố nên 2017 không thuộcS (vì khơng tất số cịn lại nguyên tố với nó, mâu thuẫn với tính chất củaS) Ta số ngun dươnga khơng vượt q 1008 thuộc tập hợpAi nêu Thật vậy, xét số2a,22a,23a, giả sử

khơng có sốk để101062ka62016 Khi đó, phải có số m để2ma <1010<2016<2m+1a, suy

(

2ma 61008

2m+1a>2018 ⇔

(

2ma61008 2ma>1009.

Điều vô lý cho thấy nhận xét chứng minh Tiếp theo, ta xét 504 tập A1010∪ {1009}, A1012∪ {1011}, , A2016 ∪ {2015}

thì tập hợp chứa tất số từ đến 2016 Vì S0 có 505 phần tử nên phải có hai phần tửa < b thuộc tập 504 tập hợp Ta có hai trường hợp: Nếub =i−1và a ∈Ai rõ ràng (i, i−1) = nên b nguyên tố với tất số

thuộcAi, dẫn đến (a, b) = 1, không thỏa

Nếua, b∈Ai theo cách xây dựng tậpAi, ta có a|b, khơng thỏa

Vậy khơng tồn tập S0 nên giá trị lớn cần tìm 504 Nhận xét Ta biết kết quen thuộc sau

1 Nếu chọn n+ số nguyên dương từ 2n số nguyên dương ln có số chia hết cho số (chứng minh cách xét lũy thừa 2) Giá trị n+ tốt chọnn số, ta chọn số từ n+ 1, n+ 2, ,2n khơng có hai số chia hết cho

2 Nếu chọn n+ số nguyên dương từ 2n số ngun dương ln có hai số nguyên tố (chứng minh cách chian nhóm cặp số lẻ chẵn liên tiếp) Giá trịn+ 1cũng tốt chọnn, ta chọn tất số chẵn khơng có hai số ngun tố

Bài tốn lời giải nói kết hợp thú vị, độc đáo hai toán đơn giản quen thuộc vừa nêu

Bài 85 Chonlà số tự nhiên lớn vàX ={1,2,3, , n} Với song ánhf :X −→X, gọi Af tập hợp tất số (i;j)sao cho i < j f(i)> f(j)

1 Có song ánhf thỏa |Af|= 1?

(82)

g :X −→X cho |Ag|=k−1 n

X

i=1

|f(i)−i|>

n

X

i=1

|g(i)−i| (|X| số phần tử tập hợp X)

Lời giải

1 Với song ánhf, đặtai =f(i)thì rõ ràng (a1, a2, , an)chính hốn vị của(1,2, , n)

Gọi Sn số song ánhf có|Af|= ứng với tập hợp n số nguyên dương Dễ thấy với n = 3, ta có hai hốn vị thỏa mãn 1,3,2 2,1,3 nên S3 = Xét quan hệ Sn+1, Sn

thơng qua hốn vị dãy(1,2, , n, n+ 1)

Nếu an+1 =n+ loại n+ ra, ta có hoán vị thỏa mãn đề n số ngun dương đầu tiên, có tất Sn hốn vị

Nếu an+1 6=n+ 1thì rõ ràng an =n+ 1vì =n+ 1với i < n |Af|>2, khơng

thỏa Khi đó, phải có an+1 =n, an=n+ = với 16i6n−1 Nghĩa có thêm

đúng hốn vị

Từ suy Sn+1 =Sn+ có Sn =n−1với n Số song ánh cần tìm n−1

2 Trong song ánhf xét , gọi j số lớn cho aj =t < j Rõ ràng j ln tồn

|Af|=k >0 Khi đó, ta cóai =ivới j+ 16i6n Xét sốm choam =t+ 16j

thì rõ ràng 6 m 6 j Khi đổi chỗ hai số (aj, am) làm giảm giá trị |Af|

đơn vị Khi đó, giả sử ta có song ánhg tương ứng với việc đổi chỗ mà|Ag|=k−1và

đặt bi =g(i) Ta chứng minh T = n

X

i=1

|bi−i| − n

X

i=1

|ai−i|60 Thật vậy,

T =|bj−m|+|bm−j| − |aj −j| − |am−m| =|t+ 1−j|+|t−m| − |t−j| − |t+ 1−m|

Vì t+ 16j nên ta có

T =j −(t+ 1)−(j−t) +|t−m| − |t+ 1−m|=|t−m| − |t+ 1−m| −1

Vìt−m t+ 1−m hai số nguyên liên tiếp nên chênh lệch chúng 1, suy T 61−1 = Do đó, hốn vị g thỏa mãn điều kiện đề

Ta có điều phải chứng minh

Nhận xét Ta biết số lượng Af nêu số nghịch Nếu gọi Sk "độ

lệch" nhỏ song ánh có k nghịch theo kết câu b), ta có dãy Sk dãy

khơng giảm Từ suy độ lệch lớn số nghịch lớn nhất, điều đạt hai nửa số 1,2, , n đổi chỗ cho Ở câu a), yêu cầu tính số song ánh f để |Af|= tốn thú vị nhiều Khi đó, cách truy hồi tương tự, ta

có cơng thức Sn=Sn−1+n−2 S3 = hay Sn =

(n−1)(n−2)

2

Bài 86 Một câu lạc có 100 thành viên, thành viên quen với số thành viên khác câu lạc Chứng minh tồn hai thành viên câu lạc có số người quen chung số chẵn (hai người khơng có người quen chung coi thỏa mãn toán)

Lời giải

(83)

1 Nếu tồn đỉnhAn bậc lẻ Xét tập X gồm người quen An Trong tậpX có lẻ người

Nếu người tập X có lẻ người quen tập X Suy bậc đỉnh Ai X bậc lẻ Suy tổng bậc đỉnh X lẻ Mà đoạn thẳng tương ứng với hai đỉnh nên có chẵn bậc trongX (mẫu thuẫn) Do đó, tồn đỉnh Ai thuộc X có chẵn người

quen X Vậy An Ai có chẵn người quen chung

2 Nếu khơng có đỉnh bậc lẻ XétAn có chẵn người quen thuộc tập Y Số người không quen Anlà số lẻ thuộc tập Z Trong tậpZ có lẻ người nên tồn Ai trongZ có số người quen Z chẵn Mà bậc Ai chẵn nên số người quen củaAi Y chẵn

Vậy An Ai có số người quen số chẵn

Bài 87 Gọi M tập tất số tự nhiên mà chữ số hốn vị tập

{1,2,3,4,5,6,7} Sắp xếp phần tử M theo thứ tự tăng dần a1 < a2 < < an,

đón số phần tử tập M.Chứng minh ai+an+1−i =ak+an+1−k,∀16i < k6n

Lời giải

Xét tương ứng f biến x∈M thành 8888888−x Với x∈M, xcó dạng x=x1x2x3x4x5x6x7 với xi ∈ {1,2,3,4,5,6,7} Khi

f(x) = (8−x1) (8−x2) (8−x3) (8−x4) (8−x5) (8−x6) (8−x7)

Do xi ∈ {1,2,3,4,5,6,7} nên 8−xi ∈ {1,2,3,4,5,6,7} ngược lại Nên x∈M ⇔f(x)∈M.Hơn

nữa, với x, y ∈M x=y⇔f(x) = f(y) nên f song ánh Suy

{a1, a2, , an}=M ={f(a1), f(a2), , f(an)}

Lại có, từ định nghĩa f suy với x < y f(x) > f(y) với x, y ∈ M Cùng với giả thiết a1 < a2 < < an ⇒ f(an) < f(an−1) < < f(a1) suy f(ai) = f(an+1−i) với i∈ {1,2, , n} Vậy ai+an+1−i = 8888888 với mọii∈ {1,2, , n}

Bài 88

Cho số nguyên dương n Hình vng kích thước n × n tạo thành từ

2n(n + 1) đoạn thẳng đơn vị Hỏi có cách phân hoạch đoạn thẳng đơn vị thành n(n + 1) cặp, cặp gồm đoạn thẳng thỏa mãn đồng thời điều kiện sau:

1) Trong cặp, có1đoạn thẳng nằm dọc có 1đoạn thẳng nằm ngang hai đoạn thẳng có chung đỉnh (tạo thành chữ L)

2) Khơng có 2cặp phân hoạch chứa đoạn thẳng chung đỉnh Nói cách khác, khơng có hai chữ L có chung đỉnh

(Trong hình minh họa cách phân hoạch cho n= 2)

Lời giải

(84)

×

× × ×

×

×

Suy có (n+ 1)2 −n(n+ 1) =n+ 1 đỉnh đỉnh nối Ta gọi đỉnh đỉnh cô lập Xét trên1 hàng, cón đoạn thẳng nằm ngang nên có tối đa n đỉnh hàng đỉnh nối Suy hàng có đỉnh lập Từ suy dễ dàng hàng, cột có đỉnh cô lập Ta gọi tập S gồm n+ đỉnh lưới gọi phép khơng có2 đỉnh chung hàng chung cột

Ta chứng minh với phân hoạch thỏa mãn điều kiện tập đỉnh lập của phân hoạch phép

Ta chứng minh ngược lại, tập phép tương ứng với phân hoạch thỏa mãn điều kiện

Nguyên tắc xây dựng đoạn thẳng hàng có mũi tên quay hướng điểm lập Sẽ khơng có đoạn thẳng dùng chữ V đoạn thẳng có hai chiều, dẫn đến hàng cột chứa đoạn thẳng có điểm, mâu thuẫn với điều kiện tập phép Giả sử có phân hoạch khác với phân hoạch Thế có đoạn thẳng có mũi tên khác chiều với cách đặt mũi tên chuẩn (ta gọi đoạn xấu) Khơng tính tổng qt, giả sử phân hoạch có1 đoạn xấu nằm ngang Cạnh vào điểmQ.Qlà điểm nối, cạnh ngang xuất phát từQ tiếp tục cạnh xấu Cứ thế, ta đến điểmR nằm biên cạnh xấu vào Từ R đỉnh cô lập, mâu thuẫn (cạnh xấu không hướng đỉnh cô lập) Vậy số phân hoạch thỏa mãn điều kiện số tập phép Dễ thấy số tập phép bằng(n+ 1)!,

do đáp số toán (n+ 1)!

Bài 89 Cho M đa giác lồi có n đỉnh (n > 4) Tô đỏ n−3 đường chéo tô xanh n−3

đường chéo khác cho khơng có hai đường chéo màu cắt miền đa giác (giao điểm hai đường chéo tính miền đa giác)

a) Với đường chéo xanhd ta gọiCd số giao điểm đường chéo với đường

chéo đỏ Cho hai đường chéo xanh d1;d2 cho phần đa giácM nằm hai đường chéo đa giác m đỉnh Chứng minh Cd1 +Cd2 62n−m−4

b) Vớin = 2k+ 1, chứng minh số lượng lớn giao điểm đường chéo xanh đường chéo đỏ đa giác M kể 3(k−1)2

Lời giải

(85)

B d2 d1

A

đỏ

Vì tổng số đường chéo xanh n−3, cịn lại tương ứngm−2đường chéo mà cắt d1;d2 điểm Do Cd1 +Cd2 62 (n−m−1) +m−2 = 2n−m−4

b) Với đường chéo d1, d2, , dn−3 Ta xây dựng hai đường chéo d1;d2 để chia đa giác M thành hai tam giác (n−2)-giác Tiếp tục kẻ d3;d4 để chia (n−2)-giác thành hai tam giác

(n−4)-giác Cứ tiếp tục làm khi lại tam giác n lẻ (xem hình vẽ phía dưới) Ta có n−3 = 2k−2 = (k−1) Hơn nữa, hai đường thẳng d2s−1;d2s có n−2s đỉnh với 16s6k−1 Theo ý a) ta có

Cd2s−1 +Cd2s 62n−(n−2s)−4 =n+ 2s−4 = 2k+ 2s−3 Do đó, số giao điểm

C1+C2+C3+C4+ +C2s−1+C2s+ C2k−1+C2k

6 k−1

X

s=1

(2k+ 2s−3) = (2k−3) (k−1) + 2k(k−1)

2 = 3k

2−6k+ = 3(k−1)2

Bây ta giá trị lớn đạt Xét đa giác A1A2 A2k+1 Tô màu đỏ đường chéo A1Ai với i= 3,4, , k + Ak+1Aj với j =k+ 3, k+ 4, ,2k+ màu đỏ Cịn lại ta tơ

màu xanh đường chéo A2Ai với i=k+ 2, k+ 3, ,2k+ Ak+2Aj với j = 3,4, , k

Cụ thể phép tô màu với n = 13 k = sau

B A

C C

d2s−1 d1

d2 d2s

(86)

A1

A2

A3

A4

A5 A6 A7 A8 A9

A10 A11

A12 A13

Minh họa cho đa giác 13 đỉnh max giao điểm

Bài toán giải

Bài 90 Cho tậpM ={1,2, ,40}.Tìm số nguyên dương k nhỏ choM = k

[

i=1

Ai, Ai∩ Aj = φ không tồn i k cho tồn a, b, c ∈ Ai (a, b, c không thiết phân

biệt) mà a+b =c

Lời giải

Giả sử M =X∪Y ∪Z thỏa mãn toán |X|>|Y|>|Z| ⇒ |X|>14 Đặt X =x1, x2, , x|X| , giả sử x1 < x2 < < x|X| suy dãy x1, x2, , x|X|, x2 − x1, x3 −1, , x|X| − x1 chứa số nguyên dương thuộc M Từ ta có 2|x| −1 6 |M| = 40 ⇒ |X| 6 20 Số cặp (x, y) ∈ X×Y

|X| |Y|>

2|X|(40− |X|), mà

146|X|620⇒ |X| |Y|>min

1

2 ·14 (40−14) ;

2·20 (40−20)

= 182

Mà tổng hai số thuộc đoạn[2; 80] ⇒x+y nhận 79giá trị 182>2.79nên tồn cặp sau

(x1, y1),(x2, y2),(x3, y3),(x4, y4)

sao cho x1+y1 =x2+y2 6=x3+y3 Giả sử x1 6x2 < x3 ⇒y1 > y2 > y3 ⇒ ∀16j 6k xk−xj ∈M, xk−xj ∈/ X, yk−yj ∈M, yk−yj ∈/ Y

Đặt a =x2−x1 =y1−y2;b =x3 −x2 = y2 −y3;c= x3−x1 =y1−y3, từ ta có a, b, c∈ Z a+b=c⇒ vô lý Vậy k >4 Ta chứng minh kmin =

Tổng quát Tập S =

1,2, ,1

2 k−1

có thể phân hoạch thành k tập rời thỏa mãn a, b, c∈Zcùng thuộc tập a+b6=c

Chứng minh Ta có ∈ {0,1,2},∀i∈N, = 0∀i>k (aNaN−1 a1a0) =

N

X

i=0

ai3i biểu diễn

tam phân m= (aNaN−1 a1a0) Ta phân hoạchS sau

1 Nếum = ( a2a1a0), a0 = m ∈A0

2 Nếum = ( a2a1a0), a0 6= tồn l >1 :al−1 6= 0, al = ta chọn l nhỏ có tính chất

đó xếp m∈Al

(87)

3 Nếum1, m2 ∈A0 ⇒m1 +m2 ∈/ Ai m1+m2 = ( a2a12)

4 Nếum1, m2 ∈Al(l>1)

0 11 | {z }

lsố

< m1, m2 <

1 00 | {z }

lsố

, suy 

0 22 | {z }

lsố

< m1+m2 <

2 00 | {z }

lsố

⇒m1+m2 = ( a2a1a0) màal= ⇒m1+m2 ∈/ Al Nhận thấy rằngm=

1

k−1

∈A0,∀m: 16m <

1

k−1

thì chắn biểu diễn củam có chữ số vị tríat,(t6k−1)⇒m ∈A1, A2, , , Ak−1 ⇒

∀m∈S thuộc Al Vậy phân hoạch thỏa mãn

Bài 91 Đặt S={(a, b, c, d)|a, b, c, d∈N} Ánh xạ f từ S vàoS định nghĩa

∀X = (a, b, c, d)∈S :f(X) = (|a−b|,|b−c|,|c−d|,|d−a|) Chứng minh với X ∈S thì:

1 f4(X)là gồm toàn số chẵn,

2 tồn số tự nhiên n cho fn(X) = (0,0,0,0)

(Trong đóf1(X) = f(X),fn(X) = f(fn−1(X)),∀n>2.) Lời giải

1 Để ý |a−b| ≡a+b (mod 2).Suy

f(X)≡(a+b, b+c, c+d, d+a) (mod 2), f2(X)≡(a+c, b+d, c+a, d+b) (mod 2),

f3(X)≡(a+b+c+d, a+b+c+d, a+b+c+d, a+b+c+d) (mod 2), f4(X)≡(0,0,0,0) (mod 2)

2 Nhận xét f(2X) = 2f(X) Ta kí hiệu mk số lớn thành phần fk(X)

Rõ ràngmk>mk+1 Từ hai ý trên, kết hợp với câu 1), ta suy dãy m0,

m4

2 ,

m8

4 , ,

m4k 2k ,

là dãy gồm số tự nhiên, không tăng Nên m4k > 0,∀k dãy dãy giảm thực

sự, vơ lí Vậy tồn m4k = với k Ta thu điều phải chứng minh

Bài toán giải

Bài 92 Cho tậpS ={1; 2; ; 2017} A1, A2, , Ak tập S cho với 16 i < j 6k có tập Ai ∩Aj, A0i∩Aj, Ai∩A0j, A

0

j ∩A

0

j tập rỗng Tìm

giá trị lớn có k (Với A⊂S, A0 kí hiệu phần bù tập A S) Lời giải

Gọi k giá trị lớn có Khi đó, phương pháp quy nạp, ta chứng tỏ S ={1; 2; ;n} k= 2n−3 Trước hết ta thấy tập

(88)

gồm 2n−3 tập S thỏa mãn điều kiện toán Vớin = 2,rõ ràng k = Với n = dễ dàng kiểm tra k63 tập{1},{2},{3} thỏa điều kiện đề bài, nên suy rak = Giả thiết quy nạp kết vớin−1>3(thìk = 2n−5) Ta cần chứng minh kết với tậpS ={1; 2; ;n}.Theo nhận xét ban đầu suy k >2n−3.Gọi M ={A1;A2; .;Ak}

là tập có số phần tử nhiều thỏa mãn điều kiện toán Ta nhận xét rằng: Tập∅ và S đều không thuộc M.

Nếu tồn i cho1>i>n mà {i}hay {i}0 không thuộc M thì ta thêm trong số vàoM để mở rộng tập M

Cả hai tập {i}và {i}0 khơng thể thuộc tậpM,do có đúng {i}hoặc {i}0 thuộcM. Vì ta giả sử|Ai|6

n

2 với 16i6n

Do 2n−3> n, nên M có tập có số phần tử lớn 2, ta chọn tập A ∈ M cho |A| > |A| 6 |X| với X ∈ M mà |X| > Không tính tổng qt ta giả sử 1,2∈A Khi xét tập B ∈M khác{1},{2} A

NếuA∩B =∅ thì 1,2∈B. NếuA∩B0 =

∅ A ⊂B

NếuA0∩B =∅thì B ⊂Avà |B|= 1,theo cách chọnA lấyB ∈M,suy ra1,2∈/ B NếuA0∩B0 =

∅thìA∪B =S.Khi đó, nếunlà số lẻ do|A|,|B|6 n−1

2 ta có|A∪B|6n−1

(vơ lí); n số chẵn có trường hợp xảy |A| = |B| = n

2,

khi B =A0 A∩B =∅,suy 1,2∈/ B

Do ta kết luận {1; 2} ⊂ B {1; 2} ∩B = ∅ với B ∈ M, khác {1},{2} A Khi đó, cách loại bỏ{1}và{2}từM loại bỏ 1từ phần tử củaM ta nhận tập M ứng với n−1 phần tử tập hợp S ={2; 3; .;n} Theo giả thiết quy nạp ta có k−26 2n−5

mà k >2n−5nên ta k = 2n−3.Vậy n = 2017thì giá trị lớn cần tìm có

k k = 2·2017−3 = 4031

Bài 93 (Ả Rập, 2015) Mười đỉnh đa giác 20 cạnh A1A2 A20 tơ đen đỉnh cịn lại tô trắng Xét tập hợp S bao gồm đường chéo A1A4 tất đường chéo có độ dài với Chứng minh S, số đường chéo có hai đầu mút màu đen với số đường chéo có hai đầu mút màu trắng Từ tìm tất giá trị có hai đường chéo có hai đầu mút màu đen S

Lời giải

A1 A2 A3

A4 A5

A6 A7 A8 A9 A10 A11

A12 A13 A14 A15 A16

A17 A18

(89)

Kí hiệu hình vẽ, vìgcd(20,3) = 1nên đường chéo thuộc S tạo thành dãy qua hết tất đỉnh đa giác cho Vì đỉnh tơ đen trắng nên ta chia chúng thành nhóm gồm đỉnh đen liên tiếp đỉnh trắng liên tiếp Rõ ràng nhóm đen nằm hai nhóm trắng nhóm trắng nằm hai nhóm đen Suy số lượng nhóm trắng nhóm đen nhau, đặt làk Trong nhóm có kích thước làa số đường chéo làa−1 Từ đó, tính tổng đường chéo nhóm trắng đen, ta có tổng số đường chéo có hai đầu mút đen trắng bằng10−k Từ ta thấy số đường chéo nhận giá trị1,2,3, ,10

Bài 94

1 Cho đa giác đềuA1A2 A2017 Có tam giác nhọn có đỉnh đỉnh đa giác trên? Cho 2n+ điểm phân biệt mặt phẳng cho điểm khơng thẳng hàng

điểm khơng nằm đường trịn

(a) Chứng minh tồn đường tròn (C) qua số điểm cho điểm cịn lại có n điểm nằm n điểm nằm ngồi đường trịn

(b) Xét 2n điểm cho khơng thuộc đường trịn (C), nối tất đoạn thẳng có đầu mút 2trong số điểm Các đoạn thẳng đường trịn(C)có thể có nhiều điểm chung?

Lời giải

1 Ta có tam giácAiA1Aj tam giác tù A1 cung AiAj khơng chứa A1 chứa 1008 điểm cịn lại

Gọi x1, x2, x3 số đỉnh thuộc cung

_ AiA1,

_ A1Aj,

_

AjAi (theo chiều kim đồng hồ) khơng tính

điểm mút Do số tam giác tù có đỉnh A1 số nghiệm hệ

    

x1+x2+x3 = 2014 x1, x2, x3 ∈N∗

x3 >1008

Theo toán chia kẹo Euler ta có số tam giác tù có đỉnh A1

1008

Suy số tam giác tù là:2017

1008

Vì2017 số lẻ nên khơng có tam giác vng Do số tam giác nhộn cần tính

2017

−2017

1008

2 (a) Nhận thấy tồn đường thẳng qua số điểm cho cho tất điểm lại nằm phía so với đường thẳng này, giả sử đường quaA,B Tính tất góc mà điểm cịn lại nhìnA,B Do khơng có điểm thuộc đường trịn nên khơng có góc nhau, ta giả sử điểm lại X1, X2, · · · , Xn+1,· · · , X2n+1 thỏa mãn

\

AX1B <AX\2B <· · ·<AX\2n+1B

Xét đường tròn qua ba điểmA, B, Xn+1 Do điêmX1, X2,· · · , Xn nhìnAB

góc nhỏ góc nội tiếp chắn cung AB nên điểm nằm ngồi đường trịn Tương tự điểm Xn+1,· · · , X2n+1 nằm đường trịn Từ ta có điều phải chứng minh (b) Nối đoạn thẳng, có loại

(90)

Đoạn có đầu mút nằm đầu mút nằm ngồi đoạn có điểm chung với đường trịn Có tất cản2 đoạn nên cón2 điểm chung

Đoạn có đầu mút nằm ngồi đường trịn Mỗi đoạn có nhiều điểm chung với đường trịn Có tất

n

2

đoạn nên có

n

2

điểm chung Vậy số điểm chung n2 + 2.

n

2

Ta cách dựng để có dấu "=" Dựng đa giác lồi có n đỉnh mà cạnh cắt đường trịn điểm Dễ thấy đường chéo đa giác cắt đường tròn điểm Xét thêm n điểm đường tròn cho n điểm với đỉnh đa giác thỏa mãn điều kiện toán

Bài 95 ChoP ={P1, P2, , P2017}là tập hợp gồm2017điểm phân biệt nằm hình trịn tâm P1 bán kính 1.Với k = 1,2, ,2017 đặt xk khoảng cách nhỏ từ Pk đến

một điểm P (khác Pk) Chứng minh x21+x22+· · ·+x22017 69 Lời giải

Với mỗiPj trongP,vẽ đường tròn chứa điểm Pj có bán kính xj

2 Ta chứng minh khơng có

hai đường trịn chúng cắt hai điểm phân biệt Thật vậy, giả sử có hai đường trịn tâm Pj1 Pj2 cắt hai điểm phân biệt Pj1Pj2 <

xj1

2 +

xj2

2 điều khơng xảy

ra Pj1Pj2 khơng nhỏ xj1 xj2 Vì P1 tâm đường trịn cho trước có bán kính

nên xj khơng vượt q đường trịn vẽ thêm có bán kính khơng vượt Giả

sử Aj điểm nằm đường trịn với tâm Pj Ta có AjPj

2;P1Pj 61 nên theo bất đẳng

thức tam giác AjP1

3

2 Điều chứng tỏ điểm P nằm đường trịn

do nằm đường trịn tâm P1 bán kính

3

2 đường trịn khơng giao nên

cũng nằm đường trịn tâmP1 bán kính

3

2 Do

πx1

2

2

+πx2

2

2

+· · ·+πx2017

2

2

3

2

⇒x21+x22+· · ·+x22017 69

Bài toán giải

Bài 96 Số20162017 viết hệ nhị phân có dạng 1001100111010010111100001 Đây dãy số nhị phân có tính chất sau

Có 6 cặp 00. Có cặp 11 Có cặp 10 Có 6 cặp 01.

Hỏi có tất số tự nhiên viết hệ nhị phân có tính chất trên? Lời giải

(91)

ta không cần xét trường hợpB đứng đầu) Rõ ràng dãy có độ dài k tạo thành k−1

cặp (00 11) nên số lượng số + = 13 số lượng số + = 12 Gọi x1, x2, , x7 độ dài dãy A x1+x2 +· · ·+x7 = 13 Số nghiệm dương phương trình

12

cũng cách tạo thành dãyA Tương tự ta tìm

11

cách tạo thành dãy B Mỗi dãy A,B cho ta dãy nhị phân, dãy nhị phân cho ta số tự nhiên thỏa mãn đề Vậy có tất

12

11

số nguyên dương thỏa mãn đề

Bài 97

1 Cho S tập gồm 2017 số nguyên tố phân biệt M tập gồm 2018 số tự nhiên phân biệt cho số trongM không số phương có ước ngun tố thuộc S Chứng minh chọn trongM số số có tích số phương

2 Có 32 học sinh tham gia 33 câu lạc bộ, học sinh tham gia nhiều câu lạc câu lạc có học sinh tham gia Biết khơng có câu lạc có học sinh giống Chứng minh có câu lạc chung học sinh Lời giải

1 Số tập phân biệt khác tập rỗng củaMlà22018−1Gọi tập làM

1, M2, , M22018−1và

ai tích phần tử Mi Giả sử phần tử S p1 < P2 < < p2017 Ta viết tích dạng =b2ip

ki1 p

ki2 p

ki2017

2017 , đókij ∈ {0; 1}.Ta có

2018−1bộ (k

i1, ki2, , ki2017),

và kij ∈ {0; 1}, nên có tối đa

2017 bộ phân biệt Do tồn trùng nhau, giả sử 2 ứng với hai tícham, an Khi tích am.an số phương Bây ta cần bỏ

phần tử thuộc giao Mn Mn ta lại phần tử khác mà tích số

phương Do tốn chứng minh

2 Giả sử khơng có câu lạc chung học sinh Nếu học sinh tham gia câu lạc có tất 32 câu lạc bộ, mâu thuẫn Suy có học sinh tham gia nhiều câu lạc bộ, giả sử A tham gia câu lạc thứ 1, 2, Xét câu lạc có A, B C Câu lạc thứ có B C, giả sử A, B D Nếu câu lạc thứ B phải có C D, nghĩa có A, C D Khi khơng tồn cách chọn câu lạc thứ Suy câu lạc thứ cóB, có A, B vàE Lập luận tương tự ta suy câu lạc có A cóB ngược lại cóB có A Giả sử A tham gia k câu lạc B tham giak câu lạc Mỗi học sinh lại tham gia nhiều trongk câu lạc học sinh câu lạc vớiA, B khơng tham gia câu lạc (nếu C tham gia câu lạc khác câu lạc chung với A, B, C học sinh C, trái giả sử) Lúc 30−k học sinh tham gia 33−k câu lạc Lập luận lại từ đầu (do 30−k nhỏ 33−k), tồn học sinh tham gia nhiều câu lạc Quá trình diễn vơ hạn, điều vơ lí ta có hữu hạn học sinh hữu hạn câu lạc Bài tốn tổng qt: Có n học sinh tham gia n+ câu lạc bộ, học sinh tham gia nhiều câu lạc câu lạc có học sinh tham gia Biết khơng có câu lạc có học sinh giống Chứng minh có câu lạc chung học sinh

Bài toán giải

(92)

1 Tìm tất giá trị nhận cặp (A, B)

2 Xác định số cách tô màu đỉnh đa giác để B = 14 Biết hai cách tô màu xem chúng nhận qua phép quay quanh tâm đường tròn ngoại tiếp đa giác

Lời giải

1 Gọik số dãy gồm đỉnh liên tiếp tô xanh bị chặn hai đầu đỉnh màu đỏ vàhlà số dãy gồm đỉnh liên tiếp tô đỏ bị chặn hai đầu đỉnh màu xanh Dễ thấy

2dãy đỏ là1dãy xanh 2dãy xanh là1 dãy đỏ nênh=k Gọia1, a2, , ak số lượng

các đỉnh dãy xanh; b1, b2, , bk số lượng đỉnh dãy đỏ k

X

i=1

ai = 24, k

X

i=1

bi = 79 Ngoài ra, rõ ràng dãy màu có số lượng t có t−1 cặp đỉnh liên

tiếp màu Suy raA = k

X

i=1

(ai−1) = 79−kvà tương tự B = 24−k Do đó, cặp(A, B)

sẽ có dạng (79−k,24−k)với 06k 624 Có tổng cộng 25cặp

2 DoB = 14 nên theo đẳng thức xây dựng thì14 = 24−k ⇔k = 10 Như thế, có tổng cộng10dãy xanh và10dãy đỏ Số nghiệm nguyên dương phương trình sau

10

X

i=1

ai = 24,

10

X

i=1

bi = 79

23

,

78

Do xếp vòng tròn nên xét vị trí tương đối nhóm với khơng xét vị trí cố định nhóm

Vậy số cách tơ cần tìm

10

23

78

Bài 99 Cho tam giác (T) Trên cạnh (T) lấy n điểm (n > 1, n ∈ N) cho n điểm chia cạnh thành n+ đoạn thẳng có độ dài Nối điểm lấy đoạn thẳng song song với cạnh (T) (chẳng hạn với n = ta vẽ nhình bên dưới) Gọi Sn số tam giác hình vẽ thu Tính Sn theo n

Lời giải

Với n >1ta có Sn+1 gồm: Sn.

Số tam giác có đỉnh hướng "lên"(n+ 2) + (n+ 1) + .+ = (n+ 2)(n+ 3)

(93)

Số tam giác có đỉnh hướng "xuống"

(n+ 1) + (n−1) + (n−3) + .+ + (−1) n+1

2 =

(n+ 1)(n+ 3) + + (−1) n

4

Suy Sn=Sn−1+

(n+ 2)(3n+ 2)

4 +

1 + (−1)n−1

8 , ∀n>2 với S1 = Ta có

Sn = n

X

k=2

(Sk−Sk−1) +S1

= n

X

k=2

(k+ 2)(3k+ 2)

4 +

1 + (−1)k−1

8 + = n X k=2

k2+ n X k=2 k+ n X k=2 ! +1 n X k=2 +1 n X k=2

(−1)k−1+

=

n(n+ 1)(2n+ 1)

6 −1

+

n(n+ 1)

2 −1

+9(n−1)

8 −

1 + (−1)n

16 +

=

2n3+ 11n2+ 18n+ + + (−1) n+1

2

8 ∀n>1

Ngoài cách giải trên, ta tách riêng trường hợp chẵn lẻ sau: S2n=S2n−1+

2n+ 2

+ + + + .+n+n+ (n−1) + .+ =S2n−1+

2n+ 2

+n(n+ 1),∀n >1

S2n+1 =S2n+

2n+

+ + + + .+n+ (n+ 1) +n+ (n−1) + .+ =S2n+

2n+

+ (n+ 1)2,∀n >1 Suy S2n+1 =S2n−1+ 6n2+ 11n+ 5,∀n >1

Do S2n+1 = 2n3+

17 n

2+23

2 n+ 5,∀n >1

Từ thay ngược lại ta có S2n= 2n3+ 11

2 n

2+9

2n+ 1,∀n >1

Chú ýS1 = 5,S2 = 13,S3 = 27, S4 = 48, S5 = 78, S6 = 118 Bài 100 Một câu lạc toán học tổ chức giải đấu cầu lông giao hữu đơn, vận động viên thi đấu trận đấu đơn với vận động viên khác (không quan tâm đến kết thắng thua) Ta gọi số vận động viên số trận đấu mà vận động viên tham gia thi đấu Trước giải đấu ban tổ chức đưa dãy số nguyên dương tăng ngặt (a1,;a2; .;ak)

và lịch thi đấu theo mã số để vận động viên bốc thăm mã số thi đấu Cuối giải ban tổ chức trao kỉ niệm chương cho vận động viên có số thuộc dãy số cho Một số hạng dãy số ban đầu gọi “số hạng đẹp” có nhất1 vận động viên có số số hạng Một giải đấu “đẹp” số hạng dãy số ban đầu “số hạng đẹp”

1 Tạo lịch thi đấu cho giải đấu“đẹp” cho giải đấu có 6vận động viên tham gia (đánh số mã A, B, C, D, E, F) dãy số ban đầu (1; 2; 3; 4; 5) theo mẫu

(94)

- Nếu vận động viênA thi đấu với vận động viênB giao hàng A, cột B hàng B, cột A đánh dấu X

- Nếu vận động viên A khơng thi đấu với vận động viên B ô giao hàng A, cột B hàng B, cột A bỏ trống

2 Chứng minh với dãy số cho trước ln xếp lịch thi đấu cho giải đấu “đẹp” cóak+ vận động viên

Lời giải

1 Tạo lịch thi đấu cho giải đấu“đẹp” cho giải đấu có 6vận động viên tham gia (đánh số mã A, B, C, D, E, F) dãy số ban đầu (1; 2; 3; 4; 5) theo mẫu Học sinh cần điền lịch thi đấu

A X X X X X

B X X X X

C X X X

D X X X

E X X

F X

A B C D E F

Bây giờ, ta chứng minh với dãy số (a1;a2; .;an) cho trước ln xếp lịch thi đấu cho

giải đấu“đẹp” cóak+ 1vận động viên Mơ hình hố dạng đồ thị: vận động viên tham gia

là đỉnh, hai vận động viên thi đấu với ta có cạnh Ta chứng minh dãy số

(a1;a2; .;ak)cho trước xây dựng đồ thị gồmak+1sao cho với mỗii∈Z,16i6k

luôn tồn đỉnh đồ thị màdegA=ai phương pháp quy nạp theok Vớik = 1:

đánh số đỉnh đồ thị làB1;B2; .;Ba1+1 nối đỉnh Ba1+1với tất đỉnh lại ta

thu đồ thị thoả điều kiện đề Vớik = 2: đánh số đỉnh đồ thị làB1;B2; .;Ba2+1

và nối đỉnhBa1+1 với tất đỉnh; đỉnh Ba1 với đỉnh B1;B2; .;Ba2−1 ta thu đồ

thị thoả điều kiện đề

2 Giả sử với giá trị nhỏ k,Ta chứng minh k > : đánh số đỉnh đồ thị làB1;B2; .;Bak+1ta chia tập đỉnh làm3 tập hợp

    

S1 ={Bi|i∈Z,16i6a1}

S2 ={Bi|i∈Z, a1+ 16i6ak−1+ 1} S3 ={Bi|i∈Z, ak−1+ 26i6ak+ 1}

Ta có

    

|S1|=a1

|S2|=ak−1−a1+

|S3|=ak−(ak−1+ 1)

Theo giả thiết quy nạp ta xây dựng đồ thị đỉnh thuộcS2 cho nhận tất số hạng dãy (a2−a1;a3−a2; .;ak−1−a2) làm bậc

(95)

(b) Nối đỉnh B1 với đỉnh lại S1 (c) Nối đỉnh Bak+1 với tất đỉnh khác

Khi ta có

(

degB1 =a1

degBak+1 =ak

và đỉnh thuộcS2nhận tất số hạng dãy(a2;a3; .;ak−1) làm bậc

Bài toán giải

Bài 101 (Vietnam TST 2014) Trong mặt phẳng tọa độ vng gócOxy,xét điểm ngun có tọa độ thuộc tập hợp sau

T ={(x;y) :−206x, y 620,(x;y)6= (0; 0)}

Tô màu điểm thuộcT cho với điểm có tọa độ (x, y)∈T có hai điểm(x;y)và(−x;−y)được tơ màu Với cách tô thế, gọiN số bộ(x1;y1),(x2;y2) mà hai điểm tô màu x1 ≡2x2, y1 ≡2y2 (mod 41) Tìm tất giá trị có N

Lời giải

Trước hết, ta chuyển toán từ 2chiều thành chiều Ta có 210≡ −1(mod 41) nên 40 số ngun khác0có giá trị tuyệt đối khơng vượt q 20 chia thành2dãy, dãy có độ dài 20sao cho số hạng đầu chia41 dưx số hạng sau chia 41dư2x Tô màu số dãy cho cứ2số đối có đúng1 số tơ màu Ta quan tâm đến số lượng cặp số liên tiếp tô màu dãy Dễ thấy 220 ≡1(mod 41) nên thêm số hạng đầu dãy vào cuối dãy gồm 21 số thỏa mãn tính chất nên ta chuyển thành vịng trịn phát biểu lại tốn sau: Cho đa giác đều20đỉnh nội tiếp đường trịn cho trong2điểm đối xứng qua tâm có đúng1 đỉnh tơ màu Tính số cặp đỉnh liên tiếp tơ màu có Với nlà số chẵn, gọiSn tập hợp số cặp kề tơ màu có đa giác cón đỉnh

Ta chứng minh quy nạp rằngS4n={2k+ 1|06k 6n−1} S4n−2 ={2k|06k 6n−1} Điều chứng minh quy nạp sau Với n = dễ thấy nhận xét Giả sử nhận xét đếnn >2 Xét đa giác có4n+ 2đỉnh Đa giác tạo thành cách thêm đỉnh A vào hai đỉnh thứ 2n,2n+ thêm đỉnh B vào hai đỉnh thứ 4n,1 Ta xét trường hợp

1 Nếu đỉnh2n,2n+ tơ vàA khơng tơ tương ứng: đỉnh 4n,1 không tô B tô Lúc này, số cặp kề tô giảm

2 Nếu đỉnh 2n,2n+ tơ A tơ tương ứng: đỉnh 4n,1 không tô B không tô Số cặp kề tăng lên

3 Nếu hai đỉnh 2n,2n+ có đỉnh tơ A tơ tương ứng: hai đỉnh 4n,1có1 đỉnh tơ vàB khơng tơ Số cặp kề tăng lên

4 Nếu hai đỉnh 2n,2n+ 1, có 1đỉnh tơ A khơng tơ tương ứng: hai đỉnh 4n,1có 1đỉnh tô B tô Số cặp kề tăng lên

Do đó, S4n+2 = {x± 1|x ∈ S4n} hay S4n+2 = {2k|0 k n} Tương tự, ta có S4n+4 =

(96)

1 thành phần x thành thành phần (x, y), ta thực sau: Ứng với vòng tròn chứa số thuộc dãy A, ta lấy vòng tròn gồm số thuộc dãy A đặt lên cho số thuộc đường tròn cũ khớp với số thuộc đường tròn Viết cặp số khớp thành dãy, dãy dãy tọa độ điểm mà liền sau của(x1, y1)là(2x1,2y1) theo mod 41 Dễ thấy có tất cả20cách ghép (cố định vòng tròn cũ xoay vòng tròn mới) Tương tự với việc ghép dãyA−B, B−A, B−B nên có tổng cộng là80cách ghép tạo thành 80

dãy Tuy nhiên, ta xét thêm4dãy đặc biệt, tương ứng với điểm nằm trục tung trục hoành Cụ thể xét thêm dãyC gồm 20số0 xét4 cách ghép: A−C, C−A, B−C, C−B Do đó, tổng cộng có 84 dãy tọa độ Theo chứng minh dãy, số cặp có S20 nên đáp số toán 84S20, số chẵn từ 1·84 = 84 đến 9·84 = 756 Một cách tự nhiên ta lấy x, y nghĩ đến việc tìm 2x,2ythay ngược lại xét điểm tô màu) Từ đó, biết số xoay thành vịng trịn nhận thấy miền giá trị số cạnh việc đưa thành mơ nêu dễ trình bày nhất, thứ sáng sủa Sẽ có ích ta phát biểu tốn tổng qt sau làm việc với tham số nhỏ để tìm quy luật.Bài chất khơng khó, dễ sai kết luận vội vàng thông qua số nhận xét nhỏ Điểm mấu chốt việc tách riêng hồnh độ, tung độ, mơ hình hóa dạng đa giác ý đến đồng dư thức 210 = 1024≡ −1 (mod 41). Bài mẻ dạng tổ hợp đếm có kết hợp số học, phát biểu có phần gượng ép việc đặt vào trục tọa độ lại giúp học sinh dễ tưởng tượng Việc giải toán trường hợp chiều mang tính định để xử lý toán với nhiều chiều Chẳng hạn, khơng gianOxyz xét điểm ngun có tọa độ thuộc tập hợp

T ={(x, y, z) :−206x, y, z 620,(x;y;z)6= (0; 0; 0)}

Tô màu điểm thuộc T cho với điểm có tọa độ (x;y;z) ∈ T có điểm

(x;y;z)và(−x;−y;−z)được tô màu Với cách tô thế, gọi N số bộ(x1;y1;z1),(x2;y2;z2) mà điểm tô màu tọa độ chúng thỏa mãn điều kiện x1 ≡ 2x2, y1 ≡

2y2, z1 ≡2z2 (mod 41) Tìm tất giá trị có N

Bài 102 Xét tập hợpA ={1,2,3,4, ,2n}, n >2 Tìm số tập B A cho tổng hai phần tử A luỹ thừa 2thì hai số thuộc B

Lời giải

Giả sử B tập A có tính chất cho Do + = 22, ta có số 1 3thuộc B

1 Nếu1∈B 3∈/ B Ta chứng minh quy nạp với số nguyênt bất kỳ,06t <2n−2, số dạng 4t + thuộc B số dạng 4t + không thuộc B Khẳng định cho t = giả sử khẳng định cho t 6 s Do (s+ 1) + số lẻ nên tồn l cho

2l <4 (s+ 1) + 1 <2l+1 Vì vậy 2 (4s+ 5) > 2.2l = 2l+1 suy ra 0< 2l+1−(4s+ 5) <4s+ 5. Đặt x = 4s+ y = 2l+1−(4s+ 5) thì x+y = 2l+1 và do y có dạng 4m+ 3 ta kết luận y /∈B (s+ 1) + 1∈B Tương tự (s+ 1) + 3∈/B

2 Nếu ∈/ B ∈ B ta chứng minh số nguyên có dạng 4t+ khơng thuộcB số ngun có dạng4t+ thuộc B

Như số lẻ B tất số nguyên có dạng 4t+ tất số nguyên dạng 4t+ Đặt x= 2px0 y = 2qy0, với x0, y0 lẻ p, q ∈Z+ Nếu2px0+ 2qy0 = 2k p6=q, giả sửp < q, x0+ 2q−py0 = 2k−p, điều khơng thể Vì p=q suy tổng phần tử tập phân biệt Ai =2ia :a số nguyên lẻ, i= 1,2, , n không luỹ thừa Với Ai bất

kỳ, sau chia cho 2i áp dụng lý luận ta tất số nguyên dạng 2i(4t+ 1)

thuộc B tất số nguyên dạng 2i(4t+ 3) thuộc B Vậy tồn tại 2n+1 tập B với tính chất

(97)

Bài 103 Cho m, n (m > n >4) số nguyên dương A tập hợp có n phần tử tập hợp S ={1,2,3, , m} Chứng minh

m >(n−1)

+ n + n + n

thì ta ln chọn n + phần tử đôi phân biệt x1, x2, , xn+1 ∈ S cho tập hợp Ai = {x+y+xi|x∈A, y∈A}, i = 1, n+ thỏa mãn Aj

T

Ak = ∅ với j 6= k j, k = 1, n+

Lời giải

Xét tập hợp B ={ |x+y−z−t||x, y, z, t∈A} Ta bất đẳng thức sau

|B|61 +

n + n + n (1) Thật vậy, ta xét trường hợp sau

1 Nếu sốx, y, z, t số số dạng |x+y−z−t|

2 Nếu sốx, y, z, tcó số nhau, giả sửx=y=z 6=t Khi |x+y−z−t|=

|x−t| suy có tối đa

n

2

số|x+y−z−t|

3 Nếu số x, y, z, t có số Khi x= y có tối đa

n

3

số dạng này, cịn x=z |x+y−z−t|=|y−t| có tối đa

n

2

số dạng xét

4 Nếu sốx, y, z, t đôi khác có tối đa

n

4

số |x+y−z−t|

Do có nhiều nhất1 +

n + n + n

số dạng|x+y−z−t|, từ suy bất đẳng thức (1) Kiểm tra A2TA3 = ∅, A1TA3 = ∅ Cứ tiếp tục đến bước thứ n, ta đặt

Cn =Cn−1\ {x+xn−1|x∈B}

|Cn|>|Cn−1| − |B|>(n−n)|B|= 0⇒ ∃xn+1 ∈Cn ⇒xn+1 > xn

Khi ta kiểm tra đượcAi

T

Aj =∅với mọii6=j Vậy tồn phần tửx1, x2, , xn+1 ∈S

thỏa mãn yêu cầu toán

Bài 104 Cho số nguyên dương n > mặt phẳng, xét 2n điểm phân biệt cho khơng có 3điểm thẳng hàng Nối n2+ cặp điểm số Chứng minh

a) Có tam giác tạo thành

b) Có n tam giác tạo thành Lời giải

a) Bài tốn giải dễ dàng quy nạp Với n = 2, ta có điểm nối

(98)

Nếu trong 2k điểm lại, có điểm nối với A hoặc B thì rõ ràng điểm cùng với A, B tạo thành tam giác

Ngược lại, với điểm C điểm cịn lại khơng có đoạn CA CB nên với

2k điểm có khơng q 2k đoạn nối với A B từ 2k điểm lại Bỏ điểm A, B số cạnh giảm khơng q2k+ 1và số cịn lại k2+ 1, Theo giả thiết quy nạp tồn tam giác số điểm

Ta có điều phải chứng minh

b) Với n = 2, dễ thấy có tam giác tạo thành Giả sử khẳng định với n =k >2 Xét 2k+ điểm có (k + 1)2 + đoạn thẳng nối Khi đó, theo câu a, tồn tam giác, đặt ABC Gọi Sa, Sb, Sc tập hợp điểm 2k−1 điểm lại có nối

với A, B, C Khi đó, dễ thấy có t =|Sa∩Sb|+|Sb∩Sc|+|Sc∩Sa| tam giác tạo

thành Theo nguyên lý bừ trừ, ta có

|Sa∪Sb∪Sc|=|Sa|+|Sb|+|Sc| −t+|Sa∩Sb∩Sc|

Do|Sa∪Sb ∪Sc|62k−1và |Sa∩Sb∩Sc|>0nên t>|Sa|+|Sb|+|Sc| −(2k−1) Ta xét2trường

hợp

Nếu|Sa|+|Sb|+|Sc|>3k−1 thì t>k nên ngồi tam giác ABC, ta cịn có thêm k tam giác tổng cộng có k+ tam giác

Nếu |Sa|+|Sb|+|Sc| 63k−1, không tính tổng qt, ta giả sử |Sa|> |Sb| >|Sc| rõ ràng, ta có

|Sb|+|Sc|6

3(3k−1)⇔ |Sb|+|Sc|62k−1

Khi đó, ta xóa điểm B, C khơng q 2k đoạn thẳng cịn lại k2+ 1 đoạn thẳng nối 2k điểm Theo giả thiết quy nạp có k tam giác Vậy theo ngun

lý quy nạp, khẳng định với mọin

Bài 105 Giả sửmlà số nguyên dương Tập conA tập hợp số nguyên dương gọi tập m-đầy tổng phần tử A không vượt m với k = 1,2, , m, tồn phần tử phân biệt Acó tổng bằngk Ví dụ, tậpA={1; 2; 3}là tập6-đầy Chứng minh tồn tậpm-đầy m6= 2,4,5,9

Lời giải

Trước hết, ta chứng minh nhận xét sau

Nhận xét Xét tập tập số nguyên dương

A={a1, a2, , an:a1 < a2 <· · ·< an}

Đặt m = n

X

i=1

ai Khi A tập m-đầy a1 = i−1

X

j=1

aj + với i = 2, n

Chứng minh

a) Giả sử A tập m-đầy Khi hiển nhiên ta phải có a1 = Tiếp theo, ∃i ∈ {2, , n} >

i−1

X

j=1

ai+ s = i−1

X

j=1

aj + < m tổng phần tử phân biệt A, trái với

tính m-đầy A Vì vậy, phải có i−1

X

j=1

(99)

b) Giả sửa1 = vàai i−1

X

j=1

aj+ 1với i= 2, n(∗) Ta chứng minh Alà tập m-đầy, quy

nạp theo n Thật vậy, hiển nhiên ta có điều cần chứng minh khin = Giả sử ta có điều cần chứng minh n=k−1(k ∈Z, k>2) Xét tập

A={a1, a2, , ak:a1 < a2 <· · ·< ak}

Hiển nhên tập B = A\ {ak} thỏa mãn điều kiện (∗) Vì vậy, theo giả thiết quy nạp, B tập m0-đầy, với m0 =

k−1

X

i=1

ai Do đó, với số s, mà 16 s 6m0, tồn phần tử phân biệt

của A có tổng s (1)

Xét s, mà m0+ 16s 6m Do ak 6m0+ 1(theo (∗)) nên s >ak Do đó, s có dạng s=ak+a,

với a∈ N a 6m0 (vì s 6m =m0 +ak) Nếua = s=ak Trường hợp a >1, doa m0

và B tập m0-đầy nên phải tồn phần tử phân biệt ai1, ai2, , air ∈ B (cũng tức thuộc A) cho a=ai1 +ai2 +· · ·+air Do đós =ai1+ai2+· · ·+air+ak, với ai1, ai2, , air

và ak phần tử phân biệt thuộc A (2)

Từ (1) (2), suy A tập m-đầy Theo nguyên lí quy nạp, ta có điều cần chứng minh Ta trở lại toán Bằng cách kiểm tra trực tiếp, dễ thấy không tồn tậpm-đầy khim = 2,4,5,

8,9 Với m= 1,3,6,7,10, tập{1},{1; 2},{1; 2; 3},{1; 2; 4},{1; 2; 3; 4}, tương ứng, tậpm-đầy Tiếp theo ta chứng minh có tậpm-đầy, vớim >10, tập (m+ 1)-đầy Giả sử

A={a1, a2, , an:a1 < a2 <· · ·< an} ⊂Z

là tập m-đầy, với m >10 Từ định nghĩa tập m-đầy, hiển nhiên suy phải có m = n

X

i=1

ai Xét hai

trường hợp sau:

Trường hợp Tồn i cho < a1+a2+· · ·+ai−1+ (3) Gọi j số lớn sối thỏa mãn (3) Ta cój >3, a1 = 1, a2 = a3 ∈ {3; 4} Nếu j = n, nghĩa an

n−1

X

i=1

ai+ 1, T = (A\ {an})∪ {an+ 1}

tập có tính chất (a) X

a∈T

a=m+ 1;

(b) a1 = 1,ak k−1

X

i=1

ai+ 1với k = 2, n−1 (do Alà tập m-đầy) đồng thời

(an+ 1) n−1

X

i=1

ai+

Vì theo Nhận xét, T tập (m+ 1)-đầy Xét 36 j < n Khi từ định nghĩa j theo Nhận xét, ta có

aj < j−1

X

i=1

ai+ aj+1 =

j

X

i=1

ai+

Suy aj+1 6=aj + Do đó, với lí tương tự trường hợp j =n, tập T = (A\ {aj})∪ {aj+ 1}

(100)

Trường hợp Với i = 2, n, có >a1+a2+· · ·+ai−1+ Khi đó, A tập m-đầy nên theo Nhận xét, ta có > a1 +a2 +· · ·+ai−1 + với i = 2, n Suy A = {1; 2; 4; .; 2n−1} và m = 2n−1 Do m > 10nên n > 4 Với n = 4, dễ dàng

kiểm tra tập {1; 2; 3; 4; 6}là tập 24-đầy Giả sử có

B ={b1, b2, , bs:b1 < b2 <· · ·< bs}

là tập 2k-đầy (k >4) Khi dễ thấy tập

B0 ={1,2b1,2b2, ,2bs−1,2bs−1}

là tập 2k+1-đầy Từ lập luận suy ra, từ tập Am-đầy (m > 10) ta xây dựng tập (m+ 1)-đầy

Bài toán giải

Bài 106 Người ta tô tất cạnh đường chéo một2017-giác bởik màu, cho điều kiện sau đồng thời thỏa mãn

1 Với màu x với cặp đỉnh A, B 2017-giác đều, đoạn thẳng AB tô màu x, tồn đỉnh C (của 2017-giác đều) cho đoạn thẳng AC BC tô màu x;

2 Với X, Y, Z đỉnh đôi phân biệt tùy ý 2017-giác đều, tất cạnh tam giác XY Z tô tối đa 2màu

Chứng minh rằngk 62 (2017-giác đa giác có2017 đỉnh)

Lời giải

Ta giải phương pháp phản chứng Giả sử ngược lại, k > Ta gọi tập hợp đoạn thẳng (mỗi đoạn thẳng cạnh đường chéo giác cho) có chung đầu mút tô màu mộtchùm Nếu đầu mút chung đoạn thẳng chùm đỉnh A (của giác cho) ta gọi chùm chùmA Tập hợp tất đầu mút khác A tất đoạn thẳng thuộc chùm A kí hiệu SA; chẳng hạn, với chùm A hình

đây thìSA={X, Y, Z, U, V}

X Y

Z

U V

A

Ta nói chùmg A có màu x (hoặc màu chùm A x) tất đoạn thẳng thuộc chùm tơ màux Ta nói chùm A có tính chấtP tập SA tồn điểm, gọi làB, cho

tập hợp tất đoạn thẳng nối B với tất điểm lại trongSA chùm đồng thời

(101)

x x

x y

y A

D C

B

Xét hai đỉnhX, Y tùy ý đa giác cho giả sử đoạn thẳng XY tơ màu x Vìk>3 nên tồn màu khác x, gọi lày Do đoạn XY không tô màu y nên theo điều kiện1/ ra, tồn đỉnh Z cho đoạn ZX ZY tô màuy Như thế, đoạn thẳng ZX, ZY tạo thành chùm, chùm Z, mà SZ có điểmX có tính chất: tất đoạn thẳng nối X với tất điểm lại SZ tạo thành chùmg đồng thời màu chùm (màu x) khác với màu chùmZ (màuy) Do đó, chùmZ vừa nêu chùm có tính chấtP Điều cho thấy tồn chùm có tính chất P Vì đa giác có 2017đỉnh đỉnh sinh hữu hạn chùm nên số chùm hữu hạn đó, có hữu hạn (lớn 0) chùm có tính chất P Vì thế, tồn chùm M có tính chất P cho tập SM tập gồm nhiều điểm nhất,

các tập ST mà chùmg T chùm có tính chấtP Giả sử màu chùm M màu x Vì chùm M có

tính chất P nên tập SM tồn điểm, gọi N, cho tất đoạn thẳng nối N với

tất điểm lại SM tô màu khácx, gọi y Vìk >3nên ngồi màu x y, phải có màu nữa, gọi z, sử dũng để tô cạnh, đường chéo đa giác cho Vì đoạnM N tô màu x, khác z, nên điều kiện 1/của ra, phải tồn đỉnh Qsao choM Qvà N Qcùng tô màuz Do N Qđược tô màuz, khácy, nên hiển nhiênQ /∈SM

Xét điểmX bất kì, khác N, thuộcSM

z

z x

y

x M

Q

N X

Do tam giác M XQcó M X tơ màu x (vì X ∈SM) M Q tô màu z nên theo điều kiện 2/của ra, đoạnQX phải tô màuxhoặc màu z NếuQX tơ màu xthì tam giác N QX vi phạm điều kiện 2/của (do N X tơ màu y(vì X ∈SM X 6=N),N Q

tô màu z) Suy ra, QX tô màu z Như vậy, tất đoạn thẳng nối Q với tất điểm thuộc tập hợp {M} ∪SM tô màu z Do đó, ta có chùmQ, với SQ ={M} ∪SM Hơn nữa,

do trongSQ có điểmM mà tất đoạn thẳng nối M với tất điểm lại SQ (chính

là tất điểm thuộc SM) tô màu (màu x) nên chùm Q chùm có tính chất P

Điều mâu thuẫn với định nghĩa chùm M, tập SQ gồm nhiều điểm tập SM Mâu thuẫn

(102)

Bài 107 Cho bảng hình vngk×k với k ∈Nvàn số nguyên dương cho trước Người ta tơ bảng hình vng n màu cho Hỏi có cách tơ màu khác bảng hình vng cho, biết hai cách tô màu coi giống chúng ảnh qua phép quay quanh tâm bảng hình vng

Lời giải

A B

C D

O

T1

T2

T3

T4

Trường hợp 1.k số chẵn, ta chia bảng hình vng thành miền MiềnT1: Không chứa ô vuông nằm cạnh OA

MiềnT2: Không chứa ô vuông nằm cạnh OB. MiềnT3: Không chứa ô vuông nằm cạnh OC. MiềnT4: Không chứa ô vuông nằm cạnh OD

Các phép quay nói sau ngược chiều kim đồng hồ Số cách tô màu miền a =nk

2

4 . Ta gọi cách tơ màu chu kìd dlà số nguyên dương nhỏ cho cách tơ màu ảnh qua phép quay d·90◦ khơng đổi Ta thấy d 1,2,4 Số cách tơ màu chu kì d= làa ta cần tơ màu miềnT1 cịn miềnT2, T3, T4 ảnh củaT1 qua phép quay 90◦,180◦,270◦ Số cách tơ màu chu kì d = a2−a vì ta cần tơ màu miền T1, T2 miền T3, T4 ảnh T1, T2 qua phép quay180◦ Số cách tô màu chu kì d=

a4−(a2−a)−a =a4 −a2 Khi đáp số tốn

a4−a2

4 +

a2−a

2 +a=

a4+a2+ 2a

4

Trường hợp k số lẻ Số cách tô màu riêng cho ô tâm O khơng bị ảnh hưởng phép quay Các cịn lại ta chia làm miền giống trường hợp Đặta=nk

2−1

4 Và kết bài toán

n

a4−a2

4 +

a2−a

2 +a

=n· a

4+a2+ 2a

4

Bài toán giải

Bài 108 Trong mặt phẳng, cho đa giác lồi có17 đỉnh A1A2A3 A17, với cặp đỉnh Ai, Aj

(16i < j617) số đỉnh đa giác, ta vẽ hai vector−A−−iA→j

(103)

trên cạnh −→0 Chứng minh n6204 Lời giải

Ta gọiS số cặp vector kiểu n−→AC,−BC−→o cặp n−→CA,−CB−→o tức số cặp vector có chung điểm đầu chung điểm cuối

Với tam giác mà tổng vector đặt cạnh −→0 số cặp Với tam giác mà tổng vector đặt cạnh khác−→0 số cặp Vậy S =

17

−n

= 1360−2n

Với điểm Ai (16 i6 17) ta gọi d1 vector nhận Ai điểm đầu ci số vector nhận Ai

điểm cuối Ta có di+ci = 16 S =

17

X

i=1

ci

+

di

, mặt khác

ci

+

di

= c

2

i +d2i

2 −

ci+di >

(ci+di)2

4 −

ci+di

2 =

162

4 −

16 = 56

NênS >56.17 = 952 Suy 1360−2n>952 Hay n6204

Bài 109 Có6 đội bóng đá tham gia giải thi đấu bóng đá học sinh trường Trung học phổ thông chuyên Biết rằng, giải đấu kéo dài 45 ngày tổ chức theo thể thức thi đấu vòng tròn (nghĩa là, hai đội thi đấu với trận) Hỏi ban tổ chức giải có tất phương án xếp lịch thi đấu cho điều kiện sau đồng thời thỏa mãn:

1 Ba trận đấu có góp mặt 6đội diễn ba ngày liên tiếp, ngày trận

2 Kể từ trận đấu thứ ba, hai trận đấu liên tiếp có hai ngày khơng có trận đấu

(Hai phương án xếp coi khác tồn trận đấu thứ k, (k > 1)sao cho trận đấu đó, hai đơi gặp phương án khác hai đội gặp phương án tồn trận đấu thứ k, (k>3)mà thời gian trận trận thứ k+ 1ở phương án khác phương án kia)

Lời giải

Theo giả thiết ra, tổng số trận đấu toàn giải

5·6

2 = 15 (trận)

Vì thế, thời gian kéo dài giải yêu cầu lịch thi đấu, ban tổ chức giải xếp trận đấu giải cách thực lần lượt3 bước sau:

Bước1 Sắp xếp 3trận đấu cho 3ngày giải, đảm bảo điều kiện1được thỏa mãn Bước2 Ấn định11ngày thi đấu, trong41ngày (từ ngày thứ4đến ngày thứ 44của giải), đảm bảo trước ngày tổ chức trận thứ4, 2ngày liên tiếp khơng có trận đấu, sau ngày tổ chức trận thứ 14phải có ngày khơng có trận đấu

(104)

Khi đó, kí hiệu s1, s2, s3 s tương ứng số phương án thực bước 1, số phương án thực bước2, số phương án thực bước3và phương án xếp lịch thi đấu cho tồn giải, theo qui tắc nhân, ta có

s=s1·s2·s3 (1)

Tínhs3.Do cách xếp 12trận đấu vào 12ngày thi đấu hoán vị của12trận đấu

đó nên ta cós3 = 12! (2)

Tính s1. Vì 3 trận đấu phải có góp mặt cả 6 đội nên có

6

cách chọn

đội cho trận đầu tiên,

4

cách chọn đội cho trận thứ hai

2

cách chọn 2đội cho trận

thứ ba Vì thế, theo qui tắc nhân, ta có s1 =

6

·

4

·

2

= 6!

(2!)3 (3)

Tínhs2 Ta ấn định11ngày thi đấu cách: Ngày khơng có trận đấu, ghi 0và ngày có trận đấu, ghi1

Bằng cách trên, ứng với phương án thực bước2, ta có dãy số gồm11số1và 30số

0, thỏa mãn điều kiện trước số đầu tiên, sau số cuối cùng, hai số liên tiếp có hai số0 Ngược lại, ứng với dãy có tính chất vừa nêu, ta có phương án thực bước Vì thế, kí hiệu S tập hợp tất dãy vậy, ta có

s2 =|S| (4)

(|X|kí hiệu số phần tử tập hữu hạn X)

Với dãy thuộc S, ta xóa số trước số đầu tiên, số cặp hai số

liên tiếp 2số sau số 1cuối Khi đó, số bị xóa dãy thuộc S

2·12 = 24(số)

Vì thế, ứng với dãy thuộc S, ta có dãy gồm 17 số, có 11số (= 30−24)

số Do đó, cách xóa nêu trên, ta thiết lập ánh xạ f từ tập S đến tập T gồm tất dãy có 17số, có 11 số1 số Dễ dàng chứng minh f song ánh từ S đến T Bởi

|S|=|T| (5)

Hiển nhiên, số phần tử tập T số cách chọn vị trí (để ghi số 0) từ 17 vị trí cho trước Do đó, từ (4) (5), ta có

s2 =|S|=|T|=

17

= 17!

6!·11! (6)

Tóm lại, từ (1), (2),(3) và(6), ta

s= 6!

(2!)3 ·(12!)·

17! 6!·11! =

3·17!

2

(105)

Bài 110 Có2017tham dự Festival quốc tế Biết rằng, nhóm tùy ý gồm7người (trong 2017 người vừa nêu) có tối đa 12 cặp quen Hỏi, Festival có tối đa cặp quen nhau?

Lời giải

Dưới đây, nói Festival, ta hiểu Festival mà ở, nhóm tùy ý người (trong số người tham dự Festival) có khơng q12 cặp quen

Trước hết, xét tính nhận từ toán cách thay 2017 bởi số nguyên dươngn >7 tùy ý; nghĩa xét Festival có n người tham dự (n>7) Với Festival X có n người tham dự, kí hiệufX(n)là số cặp quen Festival Bằng quy nạp theon >7,

ta chứng minh

(

fX(n)6k2 n = 2k, k>4, fX(n)6k(k+ 1) n = 2k+ 1, k >3,

(1)

với Festival có n người tham dự Thật

• Với n= = 2·3 + 1, theo giả thiết ra, ta có fX(7) 612 = 3(3 + 1),

với Festival có người tham dự Như (1) đúng, vớin =

• Giả sử (1) với n= 2k−1 = 2(k−1) + 1, k >4;nghĩa là, ta có fX(2k−1)6(k−1)k, k>4,

với Festival X có2k−1người tham dự Giả sử ngược lại, tồn Festival X có

2k người tham dự Festival

fX(2k)>k2+

Khi đó, cách cho số thích hợp cặp quen trở thành khơng quen nhau, ta có Festival Y có2k người tham dự

fY(2k) =k2+ (2)

Dễ thấy, Festival Y, phải tồn người quen với tối đa k người khác, ngược lại, người quen với k+ người khác

fY(2k)>

2k(k+

2 =k

2+k

>k2 + (dok >4),

mâu thuẫn với (2) Tách người quen với tối đa k người khác khỏi Festival Y, ta có Festival Z có2k−1người tham dự

fZ(2k−1)>k2 + 1−k = (k−1)k+ 1,

trái với giả thiết quy nạp Vì vậy, ta có điều phải chứng minh

• Gải sử (1) với n= 2k, k >4;nghĩa là, ta có fX(2k)6k2, k>4,

với Festival X có2k người tham dự Xétn = 2k+ Ta cần chứng minh

(106)

với FestivalX có 2k+ 1người tham dự Giả sử ngược lại, tồn Festival X có

2k+ người tham dự

fX(2k+ 1)>k(k+ 1) + =k2+k+

Khi cách cho số thích hợp cặp quen trở thành khơng quen nhau, ta có Festival Y có2k+ người tham dự

fY(2k+ 1) =k2+k+ (3)

Dễ thấy, Festival Y, phải tồn người quen với tối đa k người khác, ngược lại, người quen với k+ người khác

fY(2k+ 1)>

(2k+ 1)(k+ 1)

2 >k

2+k

>k2+

(do k > fY(2k+ 1) ∈ Z), mâu thuẫn với (3) Tách người quan với tối đa k người

khác khỏi Festival Y, ta có FestivalZ có2k người tham dự

fZ(2k)>k2+k+ 1−k=k2+ 1,

trái với giả thiết quy nạp Vì vậy, ta có điều cần chứng minh

• Vậy, tóm lại, theo ngun lí quy nạp, (1) chứng minh Trở lại toán ra.

Ta có n= 2017 = 2×1008 + Vì thế, theo kết chứng minh trên, ta có fX(2017)61008×1009 = 1017072,

với FestivalX có 2017 người tham dự

Hơn nữa, xét nhóm có 2017 người, mà 2017 người tách thành hai nhóm con, nhóm gồm có 1008 người nhóm gồm 1009 người, cho hai người nhóm khơng quen người nhóm quen với tất người nhóm Dễ thấy, nhóm người cho ta Festival X0 có 2017 người tham dự có

fX0(2017) = 1008×1009 = 1017072

Vậy số cặp quen tối đa Festival nêu là1017072 Bài 111 Với n∈ {1; 2; 3}, ta gọi số tự nhiên k làsố kiểu n nếu: k = k số hạng dãy 1, n+ 2,(n+ 2)2,(n+ 2)3, ; hoặc k là tổng số số hạng dãy trên. Chứng minh số nguyên dương biểu diễn dạng tổng số kiểu với số kiểu số kiểu

Lời giải

Xét dãy số (ak) với

        

a0 =a1 = a3k+2 = 3k+1 a3k+3 = 4k+1 a3k+4 = 5k+1

(k ∈N) Đặt Sk =a0+a1 + +ak (k ∈N∗) Ta chứng minh

mọi số nguyên dương n 6 Sk viết thành tổng số số hạng có số khác dãy (ak) Thật vậy,

(107)

2 Với k = S2 = 5, suy n = =a0, n = =a0+a1, n= =a2, n= =a3, n = =a4 Giả sử khẳng định đếnk, tức số nguyên dươngn 6Sk viết thành tổng số số

hạng có số khác dãy(ak) (1)

Ta chứng minh khẳng định đến k+ 1, tức cần chứng minh số nguyên dương n 6Sk+1 viết thành tổng số số hạng có số khác dãy(ak) Có hai khả xảy

Khả n 6 Sk theo (1), ta có n viết thành tổng số số hạng có số khác

nhau dãy (ak)

Khả Sk < n Sk+1, xét m = n−ak+1 Sk+1−ak+1 = Sk hay m số nguyên dương

không vượt Sk nên theo (1), m viết thành tổng số số hạng có số

khác dãy (ak) Suy n=m+ak+1 viết thành tổng số số hạng có số khác dãy(ak)

Theo nguyên lý quy nạp ta có khẳng định với n∈N∗

Ta có điều phải chứng minh Bài 112 Có n>2đội bóng tham gia giải đấu bóng đá Giải đấu tổ chức theo hình thức thi đấu vịng trịn: đội bóng thi đấu với trận Cách tính điểm sau: sau trận, đội thắng được3 điểm, đội thua được0điểm, hịa cả2 đội

1điểm Sau giải đấu kết thúc, người ta thấy khơng có 2đội có số điểm Hỏi khoảng cách tối thiểu đội đứng đầu đội đứng cuối bao nhiêu?

Lời giải

Câu trả lời n−2; 3nếu n = 3; n−1 n >4

1 Với n= rõ ràng2 đội khơng hịa nhau, khoảng cách điểm là2

2 Với n= rõ ràng đội đứng đầu phải thắng nhất1 trận Nếu đội đứng đầu thắng2 trận đội có điểm Trong trường hợp này, hai đội cịn lại có điểm Khoảng cách đội đứng đầu đứng cuối trường hợp 6điểm Giả sử đội đứng đầu thắng hòa

1 Nếu 2đội lại hịa rõ ràng tổng số điểm đội phải là4,2,1; khoảng cách đội đứng đầu đứng cuối bảng là3 Nếu đội cịn lại khơng hịa nhau, dễ thấy đội cuối bảng phải đội thua Trong trường hợp này,2 đội đầu bảng hịa có số điểm, mâu thuẫn Vậy, với n = khoảng cách tổi thiểu đội đứng đầu đứng cuối là3

3 Vớin >4 Hiển nhiên nđội có số điểm đôi khác số nguyên, khoảng cách đội đứng đầu đội đứng cuối>n−1 Ta nếun >4thì đánh giá hiển nhiên chặt Cụ thể hơn, ta chứng minh qui nạp khẳng định sau: với n >4, tồn giải đấu với n đội cho sau giải đấu kết thúc, tổng số điểm đội 2n−3,2n−4, , n−2

Ta lập luận quy nạp Với n > ta dễ dàng xây dựng giải đấu với khoảng cách đội đầu đội cuối Chẳng hạn với đội A, B, C, D mà A thắng B, hòa với C, D; B hòa với C thuaD; C hòa với Dthì tổng số điểm đội tương ứng 5,4,3,2 Giả sử tồn giải đấu với n đội với tổng số điểm lân lượt 2n−3,2n−4, n, , n−2 Để xây dựng giải đấu với n+

đội, ta bổ sung đội thứ n+ 1vào giải đấu Ta để đội n+ 1thắng đội thứ 1(đội có 2n−3điểm), thua đội thứ 2,3;thắng đội thứ 4, thua đội thứ 5,6, đội thứ n+ phải gặp

1,2,3 đội cuối (tùy theo n ≡ 1,2 hay 0(mod3)) Trong trường hợp thứ (n ≡ 1(mod3)), ta để đội thứ n+ hòa đội thứ n; trường hợp số hai, ta để đội thứ n+ hòa đội thứ n−1

và thua đội thứn; trường hợp thứ ba, ta để đội thứn+ hòa với đọi thứn−2, thua đội thứ n−1 thắng đội thứ n Bằng tính tốn đơn giản, ta dễ dàng kiểm tra ba trường hợp ta thu giải đấu vớin+ đội phân bố điểm là2n−1,2n−2, , n−1 Bài toán

(108)

Bài 113 Trên mặt phẳng tọa độOxy, cho hình chữ nhậtABCDvớiA(0; 0),B(m,0),C(m, n), D(0, n) đóm, n số tự nhiên lẻ Hỏi phân chia hình chữ nhật ABCD thành tam giác khơng có điểm chung trong, cho thỏa mãn đồng thời điều kiện sau:

1 Mỗi tam giác có cạnh nằm đường thẳng x=i y =j(i, j ∈N)

(Gọi cạnh đẹp) đường cao tam giác ứng với cạnh có độ dài

2 Mỗi cạnh khơng đẹp cạnh chung hai tam giác;

3 Có nhiều tam giác có hai cạnh đẹp? Lời giải

Ta chứng minh thực phép phân chia theo yêu cầu đề cách chứng minh: với phép phân chia thỏa mãn 1) 2) ln tồn hai tam giác mà tam giác có hai cạnh đẹp (*) Thật Xét tất trung điểm cạnh không đẹp tất tam giác Hai trung điểm nối với đoạn chúng hai trung điểm hai cạnh không đẹp tam giác Xét đồ thị gồm đỉnh tất trung điểm nói cạnh đoạn nối nói Vì cạnh khơng đẹp cạnh chung tam giác nên đỉnh đồ thị có bậc 0hoặc

1 Nếu có đỉnh bậc cạnh khơng đẹp nhận đỉnh làm trung điểm cạnh chung hai tam giác mà tam giác có hai cạnh đẹp (*) chứng minh

2 Nếu có đỉnh bậc cạnh khơng đẹp nhận đỉnh làm trung điểm cạnh chung tam giác có cạnh khơng đẹp tam giác có hai cạnh đẹp Mặt khác tổng số bậc đỉnh số chẵn nên tồn đỉnh có bậc Và có tam giác có hai cạnh đẹp (*) chứng minh

3 Nếu tất đỉnh bậc 2: ta chia đồ thị xét thành vòng Nhận thấy tam giác có cạnh đẹp đường cao ứng với cạnh có độ dài nên đường trung bình tam giác ln nằm đường thẳngx=i+1

2 hoặcy=j+

2(i, j ∈N) Điều

này có nghĩa vịng nói chu trình gồm đoạn thẳng song song với trục Ox Oy nối tâm hình vng đơn vị chứa hình chữ nhật ABCD Hiển nhiên tâm hình vng đơn vị ln thuộc vào chu trình

Lại có, chu trình gồm cạnh song song với Ox song song với Oy nên số tâm hình vng đơn vị chu trình ln số chẵn Điều mâu thuẫn vớim.n số lẻ Vậy

bài toán chứng minh xong

Bài 114 Tại thị trấn có đúng2017 người người thị trấn sở hữu mũ Vào ngày năm mới, tất người thị trấn đem mũ tặng cho người thị trấn (khơng giữ lại mũ mình) Chứng minh chọn nhóm gồm 673 người thị trấn thỏa mãn a, b hai người tùy ý nhóm a khơng tặng mũ cho b b không tặng mũ cho a

Lời giải

Gọi X tập tất 2017 người thị trấn Một tập A X gọi “tốt” người A nhận mũ từ người A Theo nguyên lý cực hạn, tồn tập S X tập “tốt” có số phần tử lớn

Kí hiệu T tập tất người nhận mũ từ phần tử thuộc S Ta có S ∩T = ∅

(109)

ĐặtU =X\(S∪T) Vớia ∈U tùy ý, đặt B =S∪ {a} Do tính lớn củaS nên B không tập “tốt”, nghĩa tồn phần tử x ∈ B gửi mũ cho phần tử thuộc B Mà x /∈S⇒x=a agửi mũ cho phần tử thuộc S Doa∈U tùy ý nên phần tử thuộc U gửi mũ cho phần tử thuộc S Như U tập “tốt”, suy |U|6|S|, suy

|S|>2017− |S∪T|= 2017− |S| − |T|>2017−2|S| ⇒3|S|>2017⇒ |S|>673

Bài toán giải

Bài 115 (Bulgarian Winter Math 2001) Ivan Peter tham gia trò chơi sau Lần lượt, người điền chữ số lên hàng theo thứ tự từ từ trái qua phải 2001 chữ số dừng lại Luật chơi là, số thu viết hệ số không tổng hai số phương Peter thắng ngược lại Giả sử rằng, Peter người bắt đầu trò chơi Chứng minh Peter ln có chiến thuật để thắng Ivan

Lời giải

Nhận xét Một số hệ số biểu diễn thành nhóm mà nhóm gồm chẵn chữ số liên tiếp chẵn chữ số liên tiếp số khơng thể viết thành tổng hai số phương Giả sử nhận xét đúng, ta xây dựng chiến thuật cho Peter sau

Lần đầu tiên, Peter viết số

Các lần Ivan viết số Peter viết số

Ngoại trừ trường hợp đặc biệt, số từ vị trí đến 1996 gồm tồn chữ số (khi cịn chữ số ) lượt viết chữ số cuối Peter (lượt 1997, 199, 2001), Ivan (lượt 1998,2000) Trong trường hợp này, chiến thuật Peter thay đổi sau

Lượt 1997, Peter viết số 1.

Lượt 1998, Ivan viết số thì, lượt 1999 lượt 2001, Peter viết số 1.

Lượt 1998, Ivan viết số thì, lượt 1999, Peter viết số ; lượt 2000 Ivan viết số lượt 2001 Peter viết lại số lượt 2000

Như số thu nằm hai trường hợp

1 Trường hợp thường, ta thấy số thu số thỏa mãn nhận xét nên hiển nhiên số thu viết thành tổng hai số phương Lưu ý chữ số Perter không làm ảnh hưởng tới nhận xét khơng mang giá trị biểu diễn hệ số

2 Trường hợp đặc biệt, kiểm tra trực tiếp, số thu thuộc tập hợpX ={21; 23} ∪ {24; 27} Dễ thấy rằng, phần tử thuộc X không viết thành tổng hai số phương Như vậy, hai trường hợp suy Peter người thắng Vấn đề cịn lại tốn chứng minh nhận xét Do số thu có viết thành nhóm chẵn chữ số liên tiếp chẵn chữ số liên tiếp nên ta viết số thu thành tổng số dạng 1100 002 = 3.4k (có 2k chữ số 0) Do đó, số thu có dạng T = (4n+ 3).4m Bây giờ, việc lại ta chứng minh T =x2+y2 khơng có nghiệm ngun Phản chứng, phương trìnhT =x2+y2 có nghiệm nguyên Do T = (4n+ 3).4m nên T có ước nguyên tốp≡3 (mod 4)với số mũ lẻ Đến đây, dùng định lýF ermat

nhỏ số học suy điều vô lý

Bài 116 Cho bảng vng cỡ 2n×2n (với n số ngun dương) Ta gọi đường chéolà ô vuông tập hợp ô vuông phân biệtC1, C2, , Ck(vớik số nguyên

(110)

gọi rời nhaunếu chúng khơng có vng chung Hỏi bảng vng cho phân hoạch thành đường chéo rời nhau?

Lời giải

Gọi bảng ô vuông làABCD Tô màu đen trắng cho ô vuông xen kẽ theo kiểu bàn cờ cho góc A, C màu trắng góc B, D màu đen Do đường chéo tập hợp ô vuông màu, nên ta cần tìm số đường chéo rời mà tập vng màu trắng phân hoạch thành Ta điền vào ô trắng số số hình vẽ

C B A

D

1 1 • • • 1

0 • • • 0 1

1 1 • • • 0

0 • • • 1 1

1 0 • • • 0

1 • • • 1 1

• • •

• • •

• • •

• • •

• • •

• • •

Theo đó, màu sắc số điền đối xứng qua đường chéoBD Đồng thời phía BD, cặp kề điền số khác Giả sử tập ô trắng phân hoạch thành m đường chéo rời Khi đường chéo có hai kề điền số1thì ta tách đường làm đơi vị trí điểm chung hai Ta tiếp tục làm cịn Do bảng có đúngncặp số1kề điền số nên số lần tách nhiều làn, số đường chéo tạo thành nhiều m+n Trên đường chéo bây giờ, hai ô kề có số0, nên số điền 1nhiều số ô điền 0nhiều Mặt khác dễ thấy tổng số ô điền nhiều tổng số ô điền bảng 2n Do số đường chéo lúc 2n Vì m+n > 2n, hay m > n Kết hợp với tập đen, ta có số đường chéo 2n Cuối ta cách phân hoạch thỏa mãn: Nối ô đen hai hàng đầu nối ô trắng hai hàng đầu lại, hai hàng đầu phân hoạch thành hai đường chéo Do ta có

thể phân hoạch 2n hàng thành 2n đường chéo

Bài 117 Trên bảng ô vng 34×34, có tất vng điền dấu + Mỗi lần thao tác, ta chọn dòng cột tùy ý bảng đổi dấu tất dấu đó, từ

+ sang −và từ − sang + Hỏi sau số lần thao tác, thu số dấu +là:

1 17

2 hay không?

Lời giải

Ta thấy cần quan tâm đến dòng/cột bị thay đổi số lẻ lần Sau số lần thao tác, ô dấu+là ô bị tác động số chẵn lần, tốn trở thành đếm số ô bị tác động số chẵn lần Gọi plà số dòng bị thay đổi số lẻ lần q số cột bị thay đổi số lẻ lần Khi đó, có hai nhóm bị tác động số chẵn lần

(111)

Các thuộc trong pdịng trong q cột nêu trên, có pq ơ thế.

Do đó, dễ dàng tính có (34−q)(34−q) +pq= 342−34(p+q) + 2pq dấu +còn lại

a) Từ dễ thấy biểu thức342−34(p+q) + 2pq ln số chẵn nên nhận 17 dấu +

b) Ta đưa phương trình nghiệm ngun khơng âm

(34−q)(34−q) +pq= 8⇔342−34(p+q) + 2pq =

⇔172−17(p+q) +pq= 4−172 ⇔(17−p)(17−q) =−15·19 Do19là số nguyên tố nên hai thừa số 17−p,17−q, phải có thừa số chia hết cho 19 Chú ý p, q ∈ {0,1,2, ,34} nên |17−p|6 17và |17−q| 617 Vì khơng có số chia hết cho số19 Suy khơng tồn p, q thỏa mãn đẳng thức trên, tức khơng thể có dấu +

Bài 118 thi tốn gồm có phần thi (phần thi đầu phần thi sau) có tất 28 câu hỏi phần thi Mỗi người giải xác câu hỏi Mỗi cặp câu hỏi giải người chơi

1 Tìm số người chơi thi

2 Chứng minh tồn người chơi không giải câu câu hỏi phần thi đầu

Lời giải

1 Giả sử có nngười chơi A1, A2, , An 28 tậpT1, T2, , T28 đóT1, T2, , Tm làm

bài phần thi thứ Ta gọiS số baAk, Ti, Tj Ak giải Ti, Tj.Theo

điều kiện đề ta có |S| =n

7

=

28

Suy n = 36 Vậy có tất 36 người chơi

trong thi

2 Với Ti, giả sử có r người A1, A2, , Ar giải Ti, A1, A2, , Ar phải giải

được thêm khác Ti nên có 6r ba tính S có chứa Ti Mặt khác,

với Tj(j 6=i,1 6j 628), có xác người giải Ti, Tj người phải

thuộc A1, A2, , Ar Có tất 2.27 = 54 chứa Ti trongS Vậy 6r = 54 hay r = Điều

này có nghĩa tập giải người Giả sử điều cần chứng minh sai, số tập giải người phần thi đầu 1, Gọi x, y, z số người giải 1, 2, theo thứ tự Ta có

x+y+z = 36 (1)

Ta đếm số cặp (Ai, Tj)trong tập Tj phần thi giải bởiAi:

x+ 2y+ 3z = 9m (2)

(Do giải người) Ta đếm số baAk, Ti, Tj Ak giải bàiTi, Tj

ở phần thi

2

.y+

3

.z=

m

2

(3)

Từ (1),(2),(3) ta thu đượcx=m2−19m+ 108, y =−2m2+ 29m−108, z=m2−10m+ 36 Tuy nhiên, y=−2m2+ 29m−108 =−2

m−29

4

2

−23

8 <0, vô lý, doy số nguyên không

âm Vậy điều ta giả sử sai

(112)

Bài 119

1 Có 4cặp vợ chồng xếp ngồi ghế dài Biết người vợ ngồi cạnh chồng ngồi cạnh người phụ nữ khác Hỏi có cách xếp chỗ ngồi thỏa mãn

2 Có n(n ∈N∗, n

>4)cặp vợ chồng tham dự buổi tiệc Biết người trò chuyện với tất người khác, trừ vợ chồng Các trị chuyện lập thành nhóm người C1, C2, , Ck với tính chất sau: Khơng có cặp vợ chồng

nào nhóm hai người khơng phải vợ chồng có nhóm để họ trị chuyện Chứng minh k>2n

Lời giải

1 Trước hết ta có 4!cách xếp chỗ cho 4phụ nữ Ta cố định người đếm Nếu giữa 2 nữ mà có nam phải có nhất 2 nam, hai nam mà có

nữ phải có nữ

Ta coi người phụ nữ nhóm, ta có cách biểu diễn nhóm sau:

4 = = + = + = + + = + + +

1 Nếu nữ tạo thành nhóm (1,1,1,1) : có nữ phải ngồi người đàn ông, không thỏa mãn

2 Nếu 4nữ tạo thành nhóm (2,1,1) : nhóm(2,1) phải có 2nam; nhóm (1,1) phải có nam Như có cách xếp chỗ thỏa mãn: GBBGGBBG

3 Nếu 4nữ tạo thành nhóm (2,2), ta có dạng sau:

• GGBBBBGG: Có 2!cách

• BGGBBBGG GGBBBGGB: có 1·2 = cách

• BBGGBBGG GGBBGGBB: có 1·2 = cách

• BGGBBGGB: có cách

Vậy trường hợp có cách

4 Nếu 4nữ tạo thành nhóm (3,1)ta có dạng sau:

• GGGBBBBG GBBBBGGG

• BGGGBBBG GBBBGGGB

• BBGGGBBG GBBGGGBB

Tương tự, ta có2 (2! + + 1) = cách Nếu 4nữ tạo thành nhóm (4), ta có dạng sau:

• GGGGBBBB BBBBGGGG

• BGGGGBBB BBBGGGGB

• BBGGGGBB

Tương tự, ta có3!·2 + 2!·2 + 2! = 18 cách

Vậy tổng số cách xếp chỗ thỏa mãn 4!(1 + + + 18) = 816 cách

2 Gọi gi(i= 1,2, ,2n) số nhóm mà người thứ i tham gia trị chuyện Do người thứ i nói

chuyện với cặp vợ chồng (A, B)và tồn hai nhóm khác chứa A chứa B nên ta có gi >2, ∀i= 1,2, ,2n

Trường hợp 1. Tồn tại i sao cho gi = 2 Giả sử Cm ∩Ch = {i} Khi cặp vợ chồng khơng phải vợ chồng i có người tham gia vào nhómCm người tham

(113)

phải bạn đời nhóm Ch \ {i} Do nhóm phân biệt nên có tất (n−1) (n−2) nhóm Do đó, k >(n−1) (n−2) + 2>2n với n >4

Trường hợp 2. gi > 3 với mọi i = 1,2, ,2n. Khi ta gán cho người thứ i biến số xi ∈R Xét hệ phương trình 2n ẩn:

X

i∈Ct

xi = 0; t = 1,2, , k Giả sử k <2n Khi hệ

trên có số phương trình số ẩn nên tồn i cho xi khác Đặt yt =

X

i∈Ct xi ; M = (i, j) với i, j vợ chồng ;M∗ = (i, j) với i, j không vợ chồng Khi đó,

k

X

t=1 y2t =

k

X

t=1

X

i∈Ct xi

!2

=

2n

X

i=1

gix2i +

X

(i,j)∈M∗ xixj

=

2n

X

i=1

(gi−1)x2i +

2n

X

i=1

x2i + 2X i6=j

xixj−2

X

(i,j)∈M xixj

=

2n

X

i=1

(gi−2)x2i +

2n

X

i=1 xi

!2

+ X

(i,j)∈M

(xi−xj)2

Vậy

k

X

t=1

yt2 = 0⇔xi = 0;∀i= 1,2, ,2n Hay

yt= 0,∀t= 1,2, , k ⇔xi = 0,∀i= 1,2, ,2n

Vậy hệ có nghiệmxi = 0,∀i= 1,2, ,2n (vô lý) Do đók >2n

Bài 120 Tại đỉnh hình lục giác cho trước, có bướm đậu Vào thời điểm, tất cả6 bướm bay lên khỏi đỉnh lục giác lại vào thời điểm, bướm đậu xuống đỉnh lục giác cách tùy ý, cho đỉnh có bướm đậu (các bướm không thiết phải đậu xuống vị trí trước bay lên mình) Chứng minh tồn tại3con bướm mà trước bay lên, chúng đậu đỉnh, gọi A, B,C, sau đậu xuống, chúng đậu đỉnh, gọi A0, B0, C0, có tính chất: tam giác tạo đỉnh A0, B0, C0 tam giác tạo đỉnh A,B,C

Lời giải

Nhằm tránh dài dịng, phần trình bày đây, nói “tam giác”, ta hiểu “tam giác có ba đỉnh ba đỉnh lục giác cho”

Cách Ta gọi đường chéo nối hai đỉnh đối xứng qua tâm lục giác đường chéo lục giác (xem hình đây) Như vậy, lục giác có đường chéo

Dễ thấy, hai tam giác, mà tam giác có cạnh đường chéo lục giác đều,

là hai tam giác (∗)

(114)

Trường hợp 1. Sau đậu xuống, X và Y lại đậu hai đầu mút đường chéo Khi đó, theo nhận xét(∗),X, Y với bướm Z tùy ý khác là3

con bướm thỏa mãn yêu cầu đề

Trường hợp Sau đậu xuống, X Y đậu hai đầu mút hai đường chéo Gọi Z bướm mà sau đậu xuống, X đậu hai đầu mút đường chéo Theo nhận xét (∗), X, Y Z bướm thỏa mãn yêu cầu đề

Bài toán chứng minh

Cách Ta phân chia tam giác thành ba loại, hình (kể từ trái qua phải loại1, loại loại 3):

A B

C

F

E D

A B

C

F

E D

A B

C

F

E D

Dễ thấy, hai tam giác loại hai tam giác khác loại không Vì thế, theo đề bài, ta phải chứng minh tồn bướm, mà trước bay lên sau đậu xuống, chúng đậu đỉnh hai tam giác loại

Bằng cách đếm trực tiếp, ta thấy có6tam giác loại1,12tam giác loại 2và2tam giác loại

3 Như thế, số tam giác loại2 lớn tổng số tam giác thuộc hai loại lại (12>6 + 2) Suy ra, phải có3 bướm mà trước bay lên, chúng đậu đỉnh tam giác loại sau đậu xuống, chúng đậu đỉnh tam giác loại Ta có điều phải chứng minh

Bài toán giải

Bài 121 Tại Hội khỏe Phù Đổng gồm có n vận động viên tham gia vào 12môn thi đấu, môn thi đấu có 24 vận động viên Biết hai thành viên tham gia chung khơng q mơn thi Tìm giá trị nhỏ n

Lời giải

Nhận xét Hai vận động viên tham gia chung không môn thi đương đương với hai mơn thi chung khơng q vận động viên Ta đưa việc đếm bộS có dạng(X, Y, Z)

với hai mơnX, Y có vận động viên Z tham gia chung Đếm theo môn thi đấu, ta có S6

12

Đếm theo vận động viên: ta gọi x1, x2, , xn lần lượt số môn mà vận động viên 1,2, , n tham gia Bằng cách đếm số lượt tham gia ta có

x1+x2+ .+xn= 288

Điều dẫn đến S=

x1

2

+

x2

2

+ .+

xn

= 2(x

2

(115)

Qua hai cách đếm ta suy

12

>S = 2(x

2 1+x

2

2+ .+x

n−288)

hay

x21+x22 + .+x2n 6420 Áp dụng bất đẳng thức Cauchy – Schwarz, ta có

x21 +x22+ .+x2n > (x1+x2+ .+xn)

n =

2882

n Suy

420 > 288

2

n hayn>198 Để ý với n>198 ước lượng

288

n

= 1, ta dự đốn cực trị xảy

sốxi nhận giá trị Do vậy, với i∈ {1,2, , n} ta đánh giá (xi−1)(xi−2)>0⇔x2i >3xi−2,∀i= 1, n

Đến đây, ta suy

420>x2

1+x22+ .+x2n >(3x1−2) + (3x2−2) + .+ (3xn−2) = 3(x1+x2+ .+xn)−2n= 3.288−2n

và thu n >222 Vì bất đẳng thức xảy có 156 số xi 66số xi nên

giá trị nhỏ cần tìm là222

Bài toán giải

Bài 122 Trong mặt phẳng cho tập hợp A gồm 20182 điểm phân biệt đánh số từ 1 đến

20182 sao cho ba điểm chúng không thẳng hàng Một tứ giác gọi là "đẹp" đỉnh thuộc A đánh số số thỏa mãn hai điều kiện sau:

1 Là số tự nhiên cách 2018 đơn vị, tức số lập thành cấp số cộng có cơng sai 2018

2 Là số tự nhiên liên tiếp có chứa số chia hết cho 2018 số phải số lớn

Nối tất điểm thuộc tập hợp A lại với cho điểm thuộcA thuộc tứ giác Tìm số lớn tứ giác "đẹp" tạo thành

Lời giải

Viết số từ 1đến 20182 vào bảng vng 2018×2018 như sau:

1 · · · 2017 2018

2019 2020 2021 · · · 4035 4036

· · · ·

(116)

Khi ta thấy số thỏa mãn đỉnh tứ giác đẹp phải bốn ô vuông liên tiếp cột hàng bảng Ta gọi số "đẹp" Vậy để tìm số "đẹp" ta lát bảng vng 2018×2018 bảng vng 1×4 (1 hàng, cột) 4×1 (4 hàng, cột), gọi chung quân domino, cho khơng có hai qn domino có ô vuông chung điểm thuộc tập Achỉ thuộc tứ giác nên số thuộc đẹp Ta đếm số quân domino tối đa dùng, hiển nhiên số quân domino tối đa không vượt

20182 : = 10092 Tô đen ô thuộc hàng chẵn cột chẵn bảng ô vuông tô đỏ cịn lại, số tơ đỏ

2018

2 ×

2018

2 = 1009

2

Để ý quân domino chứa ô tô đỏ số ô tô đỏ lẻ nên ta khơng thể lát kín bảng vng quân domino trên, tức số quân domino tối đa là10092. Ta cách lát sử dụng10092−1 quân domino gồm hai bước sau

Bước 1.Lát hoàn toàn 2016 hàng bảng với quân domino4×1, hàng liên tiếp ta lát 2018 quân domino ta dùng2018×(2016 : 4) = 1009×1008quân domino Bước Lát hai hàng cuối bảng với quân domino 1×4, hàng lát liên tiếp

2018

= 504 trừ hai ô cuối hàng nên ta dùng tới504×2 = 1008quân domino Khi ta dùng 1008×1009 + 1008 = 1008×1010 = 10092−1 quân domino

Vậy số tứ giác "đẹp" lớn tạo thành 10092−1. Bài 123 Viếtn số nguyên dương a1, a2, ,an bảng Xét phép biến đổi sau

Bước Xếp thứ tự từ đến n số có bảng

Bước Cộng số thứ với 1, số thứ2 với 2, số thứ 3với 3, , số thứn với n

1 Chứng minh thực hữu hạn phép biến đổi để thu số bảng 2(a1 +a2+· · ·+an) chia hết cho n

2 Chứng minh a1+a2+· · ·+an chia hết cho n thực hữu hạn

phép biến đổi để thu số bảng Lời giải

1 Nhận xét Sau lần thực phép biến đổi, tổng số có mặt bảng tăng lên

1 + +· · ·+n= n(n+ 1)

Giả sử sau m lần thực phép biến đổi, ta thu số k Khi tổng số có bảng n×k Nhưng tổng a1 +a2+· · ·+an+m

n(n+ 1)

2 nên

a1+a2+· · ·+an+m

n(n+ 1)

2 =nk Suy ra2(a1+a2+· · ·+an) = n[2k−(n+ 1)m] chia hết

chon

2 Trên bảng có số a1, a2, , an mà tổng chúng chia hết cho n Ta thực

(117)

Lần Cộng số m thêm M thêm 1, số lại cộng ngẫu nhiên theo nguyên tắc cho đề Khi có số m+ 2,M + số b3,b4, , bn

Lần Cộng số m+ thêmn sốM + thêm n−1, sốbi cộng pi lần

sẽ cộng thêm n+ 1−pi lần

Sau lần thế, ta thu số m+n+ 2, M +n, số cịn lại số khác cộng thêm n+ Giả sử số thu bảng f(a1, a2, , an) gọi m1, M1 số nhỏ lớn bộf(a1, a2, , an) Khi

|M1−m1|=|M +n−(m+n−2)|=|M −m−2|

Như vậy, sau biến đổif, hiệu số lớn số nhỏ giảm 2, nên sau hữu hạn lần tác động biến đổi f, ta hiệu hai số khơng vượt q Ngồi ra, tổng số bảng chia hết chon tăng lênn(n+ 1) Giả sử đó, số lớn bảng xvà số nhỏ lày với |x−y|6 Nếu x6=y x=y+ Khi có r>1 số bảng x vàn−r>1 số bảng bảngx+ Tổng số bảng

x·r+ (x+ 1)(n−r) = −r+ (x+ 1)n

Nhưng tổng số bảng chia hết cho n nên r chia hết cho n Do r >1 n−r >1 nên q6r6n−1nên ta có mâu thuẫn Vậy x=y, ta có số bảng

Bài toán giải

Bài 124 Trong mặt phẳng, xét điểm đôi phân biệt Người ta muốn vẽ đường tròn qua điểm Hỏi vẽ nhiều đường tròn?

Lời giải

Kí hiệu điểm xét Ai, i = 1,7 Giả sử vẽ n đường tròn, n ∈ N∗, thỏa mãn điều kiện

đề Với i= 1,7, kí hiệu si số đường tròn qua điểm Ai Do đường tròn qua

điểm nên

7

X

i=1

si = 4n

Với mỗii= 1,7, gọi mi số cặp đường tròn mà hai đường tròn cặp qua điểm Ai

Đặt m=

7

X

i=1

mi, ta có

m=

7

X

i=1

si

=

7

X

i=2 si2−

7

X

i=1 si

!

=

7

X

i=1

si2−2n > 2·

(4n)2

7 −2n =

2n(4n−7)

7 (1)

Do

X

i=1 si2 >

1 7·

7

X

i=1 si

!2

.Mặt khác, đường tròn tùy ý (trong n đường tròn vẽ được) cắt tối đa điểm số điểm Ai nên

m62

n

2

=n(n−1) (2)

Từ (1) (2), suy 2n(4n−7)

7 6n(n−1) (3)

(118)

1 Mỗi đường tròn qua điểm qua điểm có đường trịn;

2 Hai đường tròn tùy ý cắt điểm điểm xét

Xét điểm tùy ý điểm, giả sử làA1 Vì 2/nên phép nghịch đảoP tâmA1, tỉ số k 6= 0, biến đường tròn quaA1 thành đường thẳng không qua A1 đôi cắt nhau, tạo thành tứ giác tồn phần Hình Vì đường trịn đường trịn khơng qua A1, qua điểm số điểm khác A1 (theo 1) cắt đường tròn đường tròn lại điểm điểm (theo 2) nên phép nghịch đảo P biến đường trịn khơng qua A1 thành đường tròn phân biệt, đường tròn qua điểm điểm A, B, C, D, E, F hình Tuy nhiên, dễ thấy, có tối đa đường tròn phân biệt, đường tròn qua điểm điểm vừa nêu Mâu thuẫn nhận chứng tỏ n 6= Do đó, n66

E

A1

C A

B

D

F Hình

Xét điểm gồm đỉnh tam giác nhọn ABC, trực tâm H tam giác chân đường cao D, E, F hạ từ A, B, C xuống cạnhBC, CA, AB (xem Hình đây)

A

D F

H

E

B C

Hình

Dễ thấy, đường tròn(AF HE),(BF HD),(CDHE),(ABDE),(BF EC)và (AF DC)thỏa mãn điều kiện đề Vậy, vẽ nhiều đường trịn thỏa mãn yêu cầu đề

(119)

Bài 125 Cho số nguyên dương a1, a2, · · · , a2018 Xét tập S2018 =

(−1)k1√a

1+ (−1)k2

a2+ .+ (−1)k2018

a2018; ki ∈N∗ ∀i= 1, ,2018

Chứng minh tích tất phần tử thuộc S2018 số phương Lời giải

Với n ∈ {1, 2, 3,· · · , 2018}, ta kí hiệu Sn=

(−1)k1√a

1 + (−1)k2

a2+· · ·+ (−1)kn

an; ki ∈N∗,∀ i= 1,· · · , n

Nhận xét

1 Sn có2n phần tử số thực đối 2 Với e∈Sn−1 e±

an∈Sn

ĐặtAn =

Y

e∈Sn

elà tích tất phần tử thuộcSn Ta chứng minhAnlà số phương với n ∈ {1, 2, 3, , 2018}, từ ta có điều phải chứng minh Xét đa thức Pn(x) =

Y

e∈Sn

(x+e)

thì An =Pn(0) Theo nhận xét ta có Pn(x) =

Y

e∈Sn−1

(x+e+√an) (x+e−

an) =Pn−1(x+

an)Pn−1(x−

an)

S1 =

−√a1;

a1 Ta có P1(x) = x−

a1

x+√a1

=x2−a1 ∈Z[x] S2 =

−√a1−

a2; −

a1+

a2;

a1−

a2;

a1+

a2 Ta có P2(x) = (x+

a1+

a2) (x+

a1−

a2) (x−

a1+

a2) (x−

a1−

a2)

=x4−2 (a1+a2)x2+ (a1−a2)

∈Z[x] MàP2 x+

a3

=A2(x) +

a3·B2(x); P2 x−

a3

=A2(x)−

a3·B2(x)với A2(x);B2(x)∈Z[x] P2(x)∈Z[x] Từ ta đượcP3(x) =P2 x+

a3

P2 x−

a3

=A2

2(x)−a3.B22(x)∈Z[x]

Bằng phương pháp quy nạp ta chứng minh Pn(x)∈Z[x] ∀n∈ {1,2,3, ,2018} Lại có Sn chứa số đối nhau, giả sử là±e1;±e2; .;±e2n−1

Pn(x) =

Y

e∈Sn

(x+e) =

2n−1

Y

i=1

(x+ei) (x−ei) =

2n−1

Y

i=1

x2−e2i suy Pn(x) =Pn(−x)∀x

Do đóPn(x) =Qn(x2) với Qn(x)∈Z[x] Như An=Pn(0) = Pn−1(−

an)Pn−1(

an) = Pn2−1(

an) = Q2n−1(an)

là số phương

Bài 126 Với tậpA gồm n điểm phân biệt mặt phẳng (n>2), kí hiệu T(A) tập hợp vector có điểm đầu điểm cuối thuộc A Hãy xác định giá trị lớn giá trị nhỏ |T(A)| (Kí hiệu |T(A)| số phần tử tập hợp |T(A)|)

(120)

Số đoạn thẳng có hai đầu mút khác thuộcA n(n−1)

2 nên số vector khác vector

− →

0 thuộcA khơng vượt qn(n−1), kể vector−→0 thì|T(A)|6n2−n+1 Ta chứng minh tồn cấu hình cho |T(A)|=n2−n+ 1 Xét n điểmA

1, A2, , An cho A1, A2, , An∈A thỏa mãn

điều kiệnA1Aj < AjAj+1 vớij = 2, 3, , n Khi dễ dàng chứng minh được|T(A)|=n2−n+1 Xét tập A gồm n điểm A1, A2, , An Giả sử A1An = max

16i<j6n{AiAj}

gọi d đường thẳng qua A1 vuông góc với A1An Khi tất

điểm thuộc tập A thuộc nửa mặt phẳng chứa An có bờ d có A1 thuộc d Như có 2(n−1) vector khác

− →

0 đôi phân biệt

−−−→

A1A2, ,

−−−→

A1An,

−−−→

A2A1, ,

−−−→

AnA1 Do |T(A)|>2(n−1) + = 2n−1 Ta cần tập hợp mà|T(A)|= 2n−1 Thật vậy, trục số đặtAk

vào điểm có tọa độ k với −A−−iA→j tồn k cho

−−−→

A1Ak =

−−−→

A1Ak

hoặc −−−→AkA1 =

−−−→

A1Ak, nênmin|T(A)|= 2n−1

d

A1 An

Bài toán giải

Bài 127 Cho số nguyên dươngn >2 Xét tập hợp

A={(a1;a2; .;an)|ak ∈ {0; 1}, k= 1; 2; .;n}

Với phần tử a = (a1;a2; .;an) ∈ A, ak gọi số thứ k a (k = 1; 2; .;n)

Ký hiệu = (0; 0; .; 0)∈ A Xét ánh xạf :A −→ A thỏa mãn đồng thời hai điều kiện: (i) f 0= 0;

(ii) Mọia, b∈ A thỏa mãna vàb có đúngp số khác nhau, thìf(a)và f(b)cũng có p số khác

1 Với i= 1; 2; .;n, ký hiệu ei ∈ A phần tử có số thứ i 1, số

lại Chứng minh rằng: {f(ei)|i= 1; 2; .;n}={ei|i= 1; 2; .;n} 2 Phép cộng A định nghĩa sau: với

a= (a1;a2; .;an)∈ A, b= (b1;b2; .;bn)∈ A

thì c=a+b = (c1;c2; .;cn)∈ A xác định

ck =

(

0 ak =bk ak 6=bk

(k = 1; 2; .;n)

Chứng minh rằng∀a, b, c∈ A :a+b+c= f(a) +f(b) +f(c) = Lời giải

1 Với mỗii = 1; 2; .;n, xét ei có số khác nhau, nên f(ei) f(0) =

có số khác nhau, suy f(ei) =em, với m đó, m ∈ {1; 2; .;n} Mặt khác

∀i6=j, ei ej có hai số khác nhau, nên f(ei) f(ej) có hai số khác

nhau, suy f(ei) 6= f(ej), f đơn ánh từ tập {ei|i = 1; 2; .;n} vào Như

vậy

{f(ei)|i= 1; 2; .;n}={ei|i= 1; 2; .;n}

2 Xétx= (x1;x2; .;xn)∈ A, đặtf(x) =y= (y1;y2; .;yn)∈ A, x có p số 1, thìy=f(x)cũng có đúngpchỉ số bằng1 Giả sửf(ei) = em Xét phần tửx= (x1;x2; .;xn)

(121)

Nếu xi = 1, thì x và ei khác ở p−1 chỉ số nêny và em cũng khác ở p−1 chỉ số Mà y cóp số 1, nên ym =

Nếuxi = 0, tương tự,x vàei khác ởp+ 1chỉ số nêny vàem cũng khác ở p+ 1 số, nên tương tự ym =

Cả hai trường hợp cóxi =ym Từ hai điều trên, ta suy ra: nếuf(ei) = em, số thứi

của x∈ A số thứ m f(x), nên f hoán vị tập {ei} theo thứ tự nào, f hốn vị

các số x theo thứ tự ấy,∀x ∈ A Từ đó, với a, b, c thỏa mãn đề bài,f hoán vị số củaa, b, c theo thứ tự, nên số f(a) +f(b) +f(c) hoán vị theo thứ tự a+b+c= Ta có điều phải chứng minh

Bài tốn giải

Bài 128 Cho n= 2018·2019 Gọi A tập hợp (a1;a2; ;an−1;an) có tính thứ tự

cho ∈ {0; 1} với i= 1,2, , nvà n

X

i=1

ai = 20182 Có số cho

k

X

i=1 >

k

2 k

X

i=1

an−k+1 > k

2 với mọik = 1,2, , n?

Lời giải

Ta giải toán tổng quát thay 2018 bởim∈Z+ Bài toán cho tương đương với toán sau. Trong hệ trục tọa độOxy, xét lưới điểm nguyên hình chữ nhật có đỉnh bên trái làO(0; 0)

và đỉnh bên phải A(m2;m) Đặt B(m;m) và C(m2 −m; 0), hỏi có đường từ O →Asao cho bước, ta sang phải lên đơn vị, gọi đường đơn, không vượt lên OB không xuống AC?

Ở đây, số 0; tương ứng với bước lên trên, bước sang phải; điều kiện tổngk số đầu tổng k số cuối không nhỏ k

2 tương ứng với số lượng bước lên không vượt số lượng

bước sang phải Để thuận tiện, ta gọi đường cắt d có phần nằm hai phía d Trước hết, ta chứng minh bổ đề sau

Bổ đề Số đường đơn từ O →A(m;n), có cắt đường thẳng y=x

m+

m+n

Thật vậy, Xét đường thẳng (d) : y = x+ 1, rõ ràng đường đơn

cắt y = x có điểm chung với đường (d) Tại điểm chung đó, ta thực đối xứng trục để đường xuất phát từ O → A0(n−1;m+ 1) Trong hình bên, đường cũ nét đứt, đường nét liền Rõ ràng phép đối xứng trục song ánh, biến đường cần tìm (cắty =x), thành đường từ O →A0; đó, số lượng đường cần tìm

m+n n−1

Trở lại toán, số đường đơn từ O → A(m2;m)

m2 +m m

vì số cách chọn m lần lên tổng số m2+m lần di chuyển, số đường cắt OB bằng số đường cắt AC

m2+m m−1

(122)

đường thẳng này, ta đưa đếm số đường đơn từ O → A00(m−2;m2+ 2) Suy số đường trường hợp

m2+m m−2

Vậy theo nguyên lý bù trừ, kết cần tìm

m2+m m

−2

m2+m m−1

+

m2+m m−2

Thay m= 2018, ta có số lượng đường đi, số thỏa mãn đề Nhận xét Bài tốn nhìn có vẽ xử lý truy hồi bù trừ trực tiếp, thật không dễ Việc tiếp cận theo hướng dùng “ lưới ngun” địi hỏi nhiều kinh nghiệm kỹ thuật liên quan, tốn sau mơ hình hóa xong cịn phait thêm bước “đối xứng hai lần” giải triệt để Đường tốn cịn gọi đường Dyck Catalan, liên quan đến bìa tốn tiếng như: bỏ phiếu bầu cho ứng cử viên cho ứng viên thắng ứng viên thời điểm, mua vé với tiền đồng, đồng cho khơng có cần phải chờ tiền thối lại,

Bài tốn mơ theo dãy đường chéo, lên xuống đơn vị, từ (0; 0) đến

(n2+n;n2−n) sao cho đường không xuống trục hồnh khơng vượt lên trêny=n2. Thực trình bày theo hướng việc lấy đối xứng sáng sủa

Ứng với n = 1,2,3,4 ta có giá trị 0,4,100,2755 dãy số không quen thuộc nên không rút gọn đáp số Dưới số kết tương tự đường đơn đề

Số đường đơn từ (0; 0)→(m;n) mà khơng có điểm chung với y=xlà m−n m+n

m+n m

Số đường đơn từ (0; 0)→(m;n) mà không vượt qua y=x là

m+n n

m+n n−1

Số đường gồmn bước mà không vượt qua y=x là

n

X

i=n/2

n!(2i+ 1−n) (i+ 1)!(n−i)! =

n

n

2

Số đường đơn từ (0; 0)→(m;n) mà khơng có điểm chung với y=x+t là

m+n n

m+n m−t

(Việt Nam TST 2003) Tính số cặp đường đơn (0; 0) → A(m;n) và B(p; 0) → C(m;q) với p < m;q < n cho chúng khơng có điểm chung

Bài 129 Chon số thực x1, x2, , xn Với i∈ {1,2,3, , n}, gọiai số sốj

cho |xi−xj|61 bi số số j cho |xi−xj|62 (các số i, j nhau)

a) Chứng minh tồn i mà bi 63ai

b) Gọi A số cặp (i, j) có thứ tự mà |xi−xj| B số cặp (i, j) có thứ tự mà

|xi−xj|62(các số i, j nhau) Chứng minh B 63A

Lời giải

a) Khơng tính tổng qt, giả sử số cho xếp tăng dần Xét k = max{a1, a2, , an} giả sử =k, tồn k số dãy

(123)

Ngồi tính lớn k nên |xu−1−xi| > 1, |xv+1−xi| > Trong [xu;xv], có k số j để |xj −xi| < Còn trước xu, xét hai số xr, xs cho xr xs

|xr−xi|62, |xs−xi|62thì

|xr−xs|=xs−xr = (xi−xr)−(xi−xu)<2−1 =

nên có khơng q k sốj để |xj−xi|62 ngược lại, có nhiều hơnk số liên tiếp

dãy cách không đơn vị, mâu thuẫn với tính lớn k Tương tự với số sau xv, nên bi 63k ⇒bi 63ai

b) Ta chứng minh quy nạp theon Với n= 1, rõ ràngA =B = nên khẳng định hiển nhiên Giả sử kết với n>1, ta chứng minh với n+ Xét dãy số thực T = (x1, x2, , xn+1) giả sửx1 6x2 6· · ·6xn+1, kí hiệu AT, BT số cặp thứ tự

chỉ số (i, j)tương ứng với định nghĩa đề Giả sử k >1là số lượng lớn số T chứa đoạn độ dài Gọi xi số cuối dãy mà đoạn [xi−1;xi+ 1]có chứa k số (kể xi) GọiT0 dãy sau bỏ xi Khi đó, số lượng

các số thuộc T0 có [xi −1;xi + 1] k −1, xi bị bỏ thuộc 2k−1

cặp AT (gồm k−1 số ∗ thuộc đoạn tạo thành cặp có dạng (xi,∗), (∗, xi) với (xi, xi)) Do

AT =AT0 + 2k−1

Ta viết [xi−2;xi+ 2] = [xi −2;xi −1]∪[xi−1;xi+ 1]∪[xi + 1;xi + 2], trừ đoạn

hai đoạn đầu cuối chứa tối đak phần tử T Hơn nửa, định nghĩa số xi nên đoạn [xi+ 1;xi+ 2]có tối đa k−1phần tử T Suy có tối đa2(k−1) +k= 3k−2phần tử T0 (khơng tính xi) thuộc[xi−2;xi+ 2], suy

BT 62(3k−2) + +BT0 = 3(2k−1) +BT0

Áp dụng giả thiết quy nạp, ta có BT0 <3AT0 nên từ điều suy

BT 63(2k−1) +BT0 <3(2k−1) + 3AT0 = 3(AT0 + 2k−1) = 3AT

Theo ngun lí quy nạp, ta có điều phải chứng minh

Bài toán giải

Nhận xét Ta thử xét số ước lượng A, B số x1, x2, , xn đặc biệt

Nếun số thìai =bi =n nên A=B.

Nếun số chẵn liên tiếp thì ai = 1, ∀i= 1,2, , n; còn b1 =bn = 2, bi = 3, ∀i= 2, , n−1 nên A=n, B = 3n−2 rõ ràng B

A = 3−

2

n →3 chứng tỏ3 số tốt

Bằng phương pháp tương tự trên, ta chứng minh với0< a < bthì đặt cặp số có thứ tự (i, j) thỏa mãn |xi−xj|6 a cặp số có thứ tự (i, j) thỏa mãn|xi −xj| b A, B có A <2(k−1)B với k =

b a

Bài 130 Viếtn số nguyên dương a1, a2, ,an bảng Xét phép biến đổi sau

Bước Xếp thứ tự từ1 đến n số có bảng

Bước Cộng số thứ với1, số thứ 2với 2, số thứ 3với 3, , số thứn với n

1 Chứng minh thực hữu hạn phép biến đổi để thu số bảng 2(a1 +a2+· · ·+an) chia hết cho n

(124)

phép biến đổi để thu số bảng Lời giải

Nhận xét.Sau lần thực phép biến đổi, tổng số có mặt bảng tăng lên1+2+· · ·+n=

n(n+ 1)

2

Giả sử saum lần thực phép biến đổi, ta thu số làk Khi tổng số có bảng n×k Nhưng tổng a1 +a2+· · ·+an+m

n(n+ 1)

2 nên a1+a2+· · ·+

an+m

n(n+ 1)

2 =nk

Suy ra2(a1+a2+· · ·+an) = n[2k−(n+ 1)m]chia hết chon Trên bảng có số a1,a2, , an mà tổng chúng chia hết chon Ta thực phép biến đổi theo nguyên tắcf sau Gọi m

và M số nhỏ bảng (nếu có nhiều số m M, ta chọn 2số đại diện)

1 Lần Cộng số m thêm M thêm 1, số lại cộng ngẫu nhiên theo nguyên tắc cho đề Khi có số m+ 2, M + số b3, b4, , bn

2 Lần 2.Cộng số m+ thêm n sốM + thêm n−1, số bi cộngpi lần

sẽ cộng thêm n+ 1−pi lần

Sau lần2như thế, ta thu số m+n+ 2,M+n, số cịn lại số khác cộng thêm n+ Giả sử số thu bảng f(a1, a2, , an) gọi m1, M1 số nhỏ lớn f(a1, a2, , an) Khi |M1−m1|= |M +n−(m+n−2)|= |M −m−2| Như vậy, sau biến đổi f, hiệu số lớn số nhỏ giảm 2, nên sau hữu hạn lần tác động biến đổif, ta hiệu hai số khơng vượt Ngoài ra, tổng số bảng chia hết cho n tăng lên n(n+ 1) Giả sử đó, số lớn bảng x số nhỏ yvới |x−y|61 Nếux6=ythì x=y+ Khi cór>1 số bảng bằngx vàn−r >1số bảng bảng x+ Tổng số bảng x·r+ (x+ 1)(n−r) = −r+ (x+ 1)n Nhưng tổng số bảng chia hết cho n nên r chia hết cho n Do r> n−r >1 nên q 6r 6n−1 nên ta có mâu thuẫn Vậyx=y, ta có số bảng Điều phải chứng minh

Bài 131 Ghi lên bảng2018 số nguyên dương đầu tiên:1,2,3, ,2018 Thực thuật tốn sau: lần cho phép xóa hai sốa,b mà khơng có số bội số thay chúng hai số ước số chung lớn bội chung nhỏ a, b Hỏi ta thực thuật tốn vô hạn lần không? Tại sao?

Lời giải

Với hai số nguyên dương a, b ta có ab= (a, b)[a, b].

a+b <(a, b) + [a, b]

Sau lần thực thuật tốn thì:

1 Các số bảng số nguyên dương

2 Tổng tất số bảng tăng thêm đơn vị

3 Tích tất số bảng khơng thay đổi ln 2018! Do tổng số bảng bị chặn trên: Thực vậy, a1, , a2018 số nguyên dương mà a1· · ·a2018 = 2018!

16aj 62018! Do đóa1+· · ·+a2018 62018·2018! Vậy tổng số bị chặn Thành thử sau số hữu hạn bước ta khơng thể thực thuật tốn

(125)

Bài 132 Trong kỳ thi chọn HSG lớp 12 mơn tốn năm học 2018-2019 có 20 học sinh tham gia Một nhóm “tình bạn ”là nhóm gồm học sinh đơi quen biết nhau, nhóm “xa lạ ”là nhóm gồm học sinh đôi không quen biết Gọi tổng số nhóm “tình bạn ”và nhóm “xa lạ ”trong kỳ thi n, tìm giá trị nhỏ n

Lời giải

Ta gọi chung nhóm “tình bạn ”hoặc nhóm “xa lạ ”là nhóm “đặc biệt ” Số nhóm người mà khơng phải nhóm “đặc biệt ”là

20

−n = 1140−n Ta gọi T số (A,{B, C}) cho A quen B C A không quen B C

Trường hợp Với nhóm đặc biệt số (A,{B, C}) 3, số (A,{B, C})

3n

Trường hợp Với nhóm người, mà khơng phải nhóm đặc biệt số (A,{B, C}) 1, trường hợp số (A,{B, C})

20

−n = 1140−n

Vậy T = 1140 + 2n Với bạn học sinh A, giả sử bạnA quen với x học sinh không quen vớiy học sinh Ta có x+y = 19 số (A,{B, C})ứng với học sinh A

x

2

+

y

2

= x(x−1)

2 +

y(y−1)

2 =

x(x−1)

2 +

(19−x)(18−x)

2 >81

Suy T >81·20 = 1620 hay 1140 + 2n>1620 Vậy n >240

Để dấu bằng, ta đánh số bạn học sinh A1, A2, , A20 xây dựng mơ sau: Ai Aj quen i,j tính chẵn lẻ Và Ai không quen Aj i, j khác chẵn lẻ Khi

đó số (A,{B, C}) ứng với học sinh A

10

+

9

= 81, tức dấu xảy

Bài 133 Trên mặt phẳng cho2n2(n> 2)đường thẳng cho khơng có hai đường song song khơng có ba đường đồng quy Các đường thẳng chia mặt phẳng thành miền rời Trong miền đó, gọi F tập tất miền đa giác có diện tích hữu hạn Chứng minh tơ n đường thẳng số 2n2 đường thẳng cho màu xanh cho khơng có miền tập F có tất cạnh màu xanh

Lời giải

Gọi Llà tập đường thẳng cho Chọn tập lớn B ⊆Lsao cho tô đường B màu xanh khơng có miền F có tất cạnh màu xanh Đặt |B|=k, ta rak >n toán giải Ta làm sau Tô đường tập L\B màu đỏ Một điểm gọi màu xanh giao hai đường thẳng màu xanh Thế có

k

2

điểm màu xanh Ta xét đường màu đỏ l Bởi tính lớn B nên phải có miền A ∈F có cạnh màu đỏ nằm l (vì ngược lài miền có hai cạnh đỏ có cạnh nằm l ta tơ l màu xanh thỏa mãn, điều vi phạm tính lớn B) Vì A có ba cạnh, nên hai cạnh màu xanh cắt nhau, nên A có đỉnh xanh, gọi đỉnh xanh liên kết với đường đỏl Vì điểm xanh thuộc bốn miền (giao hai đường xanh), liên kết với nhiều đường đỏ Vì số đường thẳng đỏ nhiều

k

2

Mặt khác, số đường thẳng màu đỏ 2n2 −k, ta

2n2−k 62k(k−1), suy ra

2n2 62k2−k 62k2 ⇒k >m

(126)

Bài 134 Cho số nguyên dương N > Điền vào số 1,2,3, , n2 vào tất vng bảng vng kích thước n×n, số ô vuông Chứng minh tồn hai vng kề (có chung cạnh) mà hiệu hai số khơng nhỏ n

Lời giải

Gọi k số nguyên nhỏ cho tòn hàng cội chứa số thuộc tập

{1,2, , k}

1 11 10

16 14

7 13

12

Chẳng hạn hình vẽ trên, xét theo hàng phần tử lớn hàng 11,16,15,12, số bé 11 Nếu xét theo cột, số lớn 11,16,14,15, số bé 11 So sánh hàng cột đó, thấy hàng hàng chứa số 1,11,4,10 số thuộc tập

{1,2, , k} Giả sử số k thuộc hàng r cộtc, cịn lại hàngr thuộc tập {1,2, , k} Nhận xét Mỗi cột trừ cột cđều chứa số >k+ 1và tất ô cột đều>k+ Suy cột thứ iphải chứa cặp (ai, bi)kề ma 6k−1, bi >k+

1 Nếu tồn ô cột c chứa số > k + cột c chứa cặp (a, b) kề mà a 6 k, b>k+

2 Nếu ô cộtc 6k cchứa cặp (a, b) kề màa 6k−1, b=k Như vậy, trường hợp, tồn cặp (a, b) kề cột thứi mà

max{a1, a2, , an}=A < B = min{b1, b2, , bn}

Nếu (1) xảy A=k, B =k+ Nếu (2) xảy A=k−1, B =k Từ

n

X

i=1

bi >B+B+ +B+ +· · ·+B+n+ =

B +n−1

n

n

X

i=1

ai 6A+A−1 +A−2 +· · ·+A−n+ =

A− n−1

2

n

Suy

n

X

i=1

(bi−ai)>n2 nên tồn tạij: bj−aj >n

Điều phải chứng minh

Bài 135 Một điểm mặt phẳng với hệ trục tọa độ vng góc Oxy gọi điểm nguyên hoành độ tung độ điểm số nguyên Chứng minh với số nguyên dương n, tồn đường tròn chứa n điểm nguyên bên Lời giải

Chọn điểm C =

1 3;

3

Ta chứng minh khơng có hai điểm ngun khác có khoảng cách đến C Thật vậy, giả sử ngược lại có hai điểm nguyên (m, n) (p, q) khác có

khoảng cách đếnC Khi đó, ta có

m−

3

2

+ (n−√3)2 =

p−1

3

2

+ (q−√3)2, hay

m2+n2−p2−q2− 2m

3 +

2p

3 =

(127)

Từ đó, m, n, p, q số nguyên nên 2√3(n−q)là số hữu tỉ Suy n−q= 0, hay n=q Vì m6=p, ( (m, n) (p, q)là hai điểm khác nhau) từ (∗) ta được:

(m−p)

m+p−

3

=

Suy m+p−

3 = 0, điều vơ lí, m, p ∈ Z Điều vơ lí chứng tỏ giả sử sai

ta có điều muốn chứng minh Xét dãy đường tròn tâm C(C, Ri) với i = 1,2,3, , với R1 khoảng cách từ C đến điểm nguyên gần

R1 < R2 < R3 <· · · ,

đồng thời đường tròn dãy qua điểm nguyên Theo phần chứng minh trên, đường tròn dãy qua điểm nguyên Vì thế, với số nguyên dươngn, bên đường trịn tâm C, bán kính R với Rn < R < Rn+1, có n điểm nguyên Bài toán

được chứng minh

Bài 136

1 Có 50 giống phân phối cho xã A, B, C, D Hỏi có cách phân phối biết số giống xã A phải chia hết cho 5; số giống xã B

10; số giống xã C nhiều 4và số giống xã D 15?

2 Có nhà hình vng kích thước 8×8được lát 21viên gạch kích thước1×3 Biết viên gạch ngun vẹn (khơng bị cắt ra) Khi tồn 1ơ trống kích thước 1×1 Hỏi trống nằm vị trí nào?

Lời giải

1 Số cách chia cho xã A có hàm sinh là1 +x5 +x10+ =

1−x5 Số cách chia cho xã B có hàm sinh là1 +x+x2+ +x9 = 1−x

10

1−x Số cách chia cho xã C có hàm sinh x5+x6+ = x

5

1−x Số cách chia cho xã C có hàm sinh x15+x16+ = x

15

1−x Hàm sinh số cách chia f(x) =x20(1 +x5)

(1−x)3 Số cách chia hệ số củax50 khai triểnf(x)là

27 25

+

32 30

= 847

2 Xét bậc nguyên thủy đơn vị 6= Khi đó, với số nguyên dương k ta có k+k+1+k+2 = 0 Coi nhà là 1bảng ô vuông, với 1cách lát gạch cho trước ta điền vào ô bảng số thỏa mãn: ô (i;j) điền εi+j khi viên gạch chiếm 3 số có tổng

bằng Mà tổng ô bảng S = nên ô cịn lại phải (m;n) có tổng m+n chia hết cho Cũng xét cách lát ta điền vào ô bảng1 số thỏa mãn: ô (i;j) điền εi−j

khi viên gạch chiếm số có tổng Mà tổng bảng S = nên cịn lại phải ơ(m;n) có hiệum−n chia hết cho3 Từ suy ô bỏ trống phải

4ô (3; 3),(3; 6),(6; 3),(6; 6) Chỉ cách lát cụ thể để ô trống ô

(128)

Bài 137 Với n >2, gọi f(n) số hoán vị tập {1; 2; 3; .;n} mà khơng có số k đứng liền trước số k+ với mọik ∈ {1; 2; 3; .;n−1} Chứng minh

1 f(n) = (n−1)f(n−1) + (n−2)f(n−2), ∀n>4

2 f(n) =

n

(n+ 1)!

e +

1

, ∀n>2, [x]là phần nguyên x

e= lim n→+∞

1 +

n

n

Lời giải

1 Xét hoán vị của{1; 2; 3; .;n} thỏa mãn đề bài,

(a) Nếu không tồn tạik đểk, n, k+ 1liên thứ tự bỏ n để hoán vị {1; 2; 3; .;n−1} thỏa mãn

(b) Nếu tồn k để k, n, k+ 1liên thứ tự bỏ n k+ 1đi, đồng thời giảm 1ở tất số từk+ 2đến n−1(k =n−2 khơng cần làm bước này), hốn vị {1; 2; 3; .;n−2} thỏa mãn Ngược lại

(c) Từ hoán vị {1; 2; 3; .;n−1} thỏa mãn tạo n −1 hốn vị

{1; 2; 3; .;n} thỏa mãn cách chèn n vào vị trí n−2 vị trí liền sau số 1,2, , n−2 Sẽ có tổng cộng (n−1)f(n−1) hốn vị dạng

(d) Từ hoán vị {1; 2; 3; .;n−2} thỏa mãn tạo n −2 hoán vị

{1; 2; 3; .;n} thỏa mãn cách: với k = 1, n−2, tăng thêm tất số từ k+ đến n−2 (k =n−2 khơng cần làm bước này), sau chèn n, k+ 1vào vị trí liền sau k Sẽ có tổng cộng(n−2)f(n−2)hốn vị dạng

Vậy f(n) = (n−1)f(n−1) + (n−2)f(n−2)∀n>4

2 Ta có

(n+ 1)!

e = (n+ 1)! +∞

X

k=0

(−1)k

k! = (n+ 1)! n+1

X

k=0

(−1)k

k! + (n+ 1)!

+∞

X

k=n+2

(−1)k

k!

Trong đó(n+ 1)! n+1

X

k=0

(−1)k

k! số nguyên,

(n+ 1)!

+∞

X

k=n+2

(−1)k

k!

6(n+ 1)!

+∞

X

k=n+2

1

k!

+∞

X

k=n+2

1

(n+ 2)k−n−1 =

n+ <

Do

(n+ 1)!

e +

1

= (n+ 1)! n+1

X

k=0

(−1)k

k!

Ta chứng minh khẳng định đề quy nạp Rõ ràng kết với 2,3

Nếu cóf(n−1), f(n−2), ta cần chứng minh

1

n n+1

X

k=0

(−1)k(n+ 1)!

k! =

n

X

k=0

(−1)kn!

k! + n−1

X

k=0

(−1)k(n−1)!

(129)

⇔(−1)

n+1

n +

(−1)n(n+ 1)

n + n n−1 X k=0

(−1)k(n+ 1)!

k! = (−1) n + n−1 X k=0 "

(−1)kn!

k! +

(−1)k(n−1)!

k!

#

⇔(−1)n+

n n−1

X

k=0

(−1)k(n+ 1)!

k! = (−1) n

+ n−1

X

k=0

(−1)k(n+ 1) (n−1)!

k! (đúng)

Bài tốn giải

Bài 138 Cơ giáo có tất cả2020 viên kẹo gồm20loại kẹo khác nhau, loại có2viên kẹo Cơ chia hết kẹo cho học sinh mình, người số viên kẹo khơng có học sinh nhận nhiều viên kẹo loại kẹo Cô yêu cầu hai học sinh khác so sánh viên kẹo nhận viết số loại kẹo mà hai có lên bảng Biết cặp học sinh lên bảng lần Gọi tổng số viết lên bảng M

1 Xác định giá trị nhỏ M

2 Với giả thiết tương tự thay 20 loại kẹo khác 19 loại kẹo khác nhau, tìm giá trị nhỏ M trường hợp tương ứng

Lời giải

1 Gọi a1, a2, , a20 số viên kẹo loại kẹo thứ 1,2, ,20 với > Với loại kẹo thứ i

(16i620), ta đếm số bộ(A, B)mà hai học sinh A, B có loại kẹo Số cần đếm

ai

Khi đó, theo giả thiết, tổng số làM hayM =

20 X i=1 20 X i=1

ai = 2020

Áp dụng bất đẳng thứcCauchy−Schawarz ta có M =

20

X

i=1

ai(ai−1)

2 = 20 X i=1 a2 i − 20 X i=1 > 20 X i=1 !2

2·20 −

20 X i=1 = 20202

2·20 − 2020

2 = 101000

Dấu “=” xảy = 101,∀i= 1,2, ,20

Vậy giá trị nhỏ M 101000

2 Như lý luận câu a, ta cóM =

19

X

i=1

ai(ai−1)

2 = 19 X i=1

a2i −

2

19

X

i=1

Nên biểu thứcM đạt giá trị nhỏ 19

X

i=1

a2i đạt giá trị nhỏ

Ta chứng minh 19

X

i=1

a2i đạt giá trị nhỏ |ai−aj|61với 16i, j 619 (1)

Thật Xét bộ4 sốa,b, c, d mà a>b+ 2; c=a−1; d=b+ ta có cd=ab+a−b−1>ab (a+b)2 = (c+d)2 suy ra a2+b2 >c2+d2.

Mở rộng tính chất cho nhiều số ta suy (1) chứng minh Do M đạt giá trị nhỏ cót số có giá trị làk 19−tsố có giá trị k+ 1với 06t 619và giá trị nhỏ

M =

(130)

Ta có

tk + (19−t)(k+ 1) = 2020⇔t= 19k−2001

Do 06t 619 nên 2001

19 6k 2020

19 Từ ta có k = 106, t = 13 Thay vào ta giá trị

nhỏ M

2[13·106

2+ 6·1072−2020] = 106371.

Bài toán giải

Bài 139 Tại hội nghị khoa học có100 đại biểu tham dự Người ta nhận thấy khơng có đại biểu đơi quen Biết tồn số nguyên dương n cho khơng có đại biểu quen q n đại biểu khác với k,1 6 k 6 n có đại biểu quen k đại biểu khác Hãy tìm giá trị lớn củan

Lời giải

Ta chứng minh nmax = 66 Giả sửnmax>66

Cách Khi tồn đại biểu A100 quen 67 đại biểu A1, A2, , A67 Gọi X tập hợp đại biểu A68, A69, , A99 |X|= 32 Khi đại biểu A1, A2, , A67 quen A100 đại biểu thuộc X (vì đại biểu Ai, Aj với 16i < j 667 quen nhau) Suy họ quen

không 33 đại biểu Vì đại biểu quen 34,35, ,65,66 đại biểu khác phải thuộc X Điều vô lý |X|= 32<66−34 +

Cách 2.Xét người A có67người quen Xét người B có nhiều 34người quen Nếu A quenB Khi 98người cịn lại có 66người quen A hiều 33người quen B Do 66 + 33 = 99>98 nên A, B phải có người quen chung, mâu thuẫn Vậy A khơng quenB Vì có 33người có số người quen tương ứng là34,35, ,66nên có 33 người khơng quenA Điều mâu thuẫn A quen với 67 người Vậy nmax > 66 Ta xây dựng ví dụ thỏa mãn n = 66 sau,A1 quen vớiB1, B2, , B66, A2 quen với B2, B3, , B66, , A34 quen vớiB34, B66 Khi khơng có người đơi quen Ak quen với 67−k người (∀k = 1, ,34) Bk quen với k người (∀k = 1, ,34) Bk quen 34 người với (∀k = 35, ,66) Rõ ràng trường hợp thỏa

mãn yêu cầu đề

Bài 140 Gọi S tập hợp (a1, a2, , a164) hoán vị 164 số nguyên dương

1 Có hốn vị(a1, a2, , a164)thuộcS cho với mọii∈ {1,2, ,164}ta ln cóai 6=i ≡i (mod 41)?

2 Tồn hay khơng hốn vị (a1, a2, , a164) thuộc S cho với i ∈ {1,2, ,164} tồn số nguyênbi ∈ {0,1, ,40}thỏa mãna1+a2+· · ·+ai ≡b2i ( mod 41)?

Lời giải

1 Chia số từ 1,2, ,164 thành 41 nhóm theo số dư chia cho 41 rõ ràng nhóm có4 số (4 số vị trí ai, ai+41, ai+82, ai+123 với i= 1,41 hoán vị) Các số hốn đổi vị trí cho

Ta thấy với (x1, x2, x3, x4), ta có cách hoán vị

(x2, x1, x4, x3),(x2, x3, x4, x1),(x2, x4, x1, x3),

(x3, x1, x4, x2),(x3, x4, x1, x2),(x3, x4, x2, x1),

(x4, x1, x2, x3),(x4, x3, x2, x1),(x4, x3, x1, x2)

(131)

2 Ta chứng minh kết quen thuộc “vớiplà số ngun tố có dạng3k+ 2thì{13,23, , p3}

sẽ lập thành hệ thặng dư đầy đủ theo mod p” Thật vậy, giả sử có hai số i6=j cho i3 ≡j3 (mod p) thì i3k ≡j3k (mod p) Theo định lý Fermat nhỏ thì

i3k+1 ≡j3k+1 (mod p)

nên

i3k+1 ≡j3k+1 =j ·j3k ≡j·i3k (mod p)

kéo theo i3k(i−j) .p hay i ≡ j (mod p), vô lý vì i, j ∈ {1,2, , p} và i 6=j. Ta xây dựng

hoán vị thỏa mãn đề cho ≡ i3 (mod 41) với i 164 Với 41 số đầu tiên,

theo nhận xét số {13,23, ,413} có số dư đơi khác chia cho 41 nên ta xếp chúng để số dư thay đổi từ 1→41 Các số 42→ 82; 83→123; 124→ 164 thực hốn vị tương tự Hốn vị thỏa mãn

a1+a2+· · ·+ai ≡13+ 23+· · ·+i3 =

i(i+ 1)

2

(mod 41)

Bài toán giải

Bài 141 Cho bảng ô vuông gồm m hàng n cột Tại góc bên trái bảng người ta đặt quân cờ Hai người chơi luân phiên di chuyển quân cờ, lượt di chuyển di chuyển quân cờ sang phải ô xuống ô Người chơi đến lượt không di chuyển quân cờ thua Xác định điều kiện m, nđể người thực lượt chơi người thắng

Lời giải

Ta tô màu ô bảng ô vuông hai màu trắng đen với ô bên trái bảng màu trắng (tô đan xen bàn cờ) Ta gọi người thứ người thực di chuyển người lại người thứ hai

Gọi ô thuộc hàngp cột q là ơ (p;q) Khi đó:

• Nếu m, ncùng tính chẵn lẻ (1; 1) có màu với (m;n)

• Nếu m, nkhác tính chẵn lẻ (1; 1) khác màu với ơ(m;n)

(132)

• Ở lượt di chuyển đầu tiên, người thứ di chuyển cờ sang ô đen, người thứ hai di chuyển cờ sang ô trắng bất biến toán

• Cờ ln đưa ơ(m;n)điều có nghĩa người thắng phải người lượt chơi phải đặt cờ vào ô (m;n) (và người cịn lại khơng di chuyển cờ được)

Do để người thứ ln thắng (m;n) phải trùng màu với ô mà người thứ di chuyển lần (tức đen) Hay nói cách khác ô (m, n) phải khác màu với ô (1; 1) người thứ ln thắng Điều có nghĩam, n phải khác tính chẵn lẻ Ngược lại, nếum, n có tính chẵn lẽ Theo lập luận người thứ hai ln thắng (dù người thứ có di chuyên nào) Vậy m, nphải khác tính chẵn lẻ người thứ ln thắng

Bài tốn giải

Bài 142 Cho tập hợp X = {1; 2; 3; .; 2018} gồm 2018 số nguyên dương A tập tập X thỏa mãn: với x, y, z ∈ A; x < y < z x, y, z độ dài ba cạnh tam giác Hỏi tập hợp A có nhiều phần tử?

Lời giải

Giả sửA={a1; a2; .; ak} với a1 < a2 <· · ·< ak Nếu ak <2018, đặt t= 2018−ak bi =ai+t

với i = 1, 2, , k tập {b1; b2; .; bk} thỏa mãn toán bk = 2018 Do ta

giả sử ak= 2018 Ta có a1 +a2 > ak a1 < a2 nên

2a2 > ak= 2018⇒a2 >1009⇒a2 >1010 Từ ta có A\ {a1} ⊂ {1010; 1011; ; 2018},

|a| −162018−1010 + = 1009⇒ |a|61010

Xét tập hợp A0 ={1009; 1010; .; 2018}, ta có |a0| = 1010 với x, y, z ∈ A0 mà x < y < z ta có

x+y>1009 + 1010 = 2019>2018 >z,

hay x, y, z độ dài ba cạnh tam giác Vậy max|a|= 1010 Bài 143 Cho số hữu hạn dọc có chiều dài 2017 cố định vài kim loại Trên có hạt mà trượt tự Một vài cặp hạt nối với dây đàn hồi Ant (em) tự di chuyển tất dây Còn Ant (anh) tự di chuyển tất dây mà độ chênh lệch chiều cao điểm cuối ±1 Giả sử xuất phát hạt nào, Ant (em) đến hạt khác Cấu hình hạt mà hạt đặt vị trí số nguyên độ cao hai điểm cuối dây khác nhau, gọi cấu hình chấp nhận Biết có cấu hình chấp nhận thiết bị này, chứng minh tồn cấu hình chấp nhận mà Ant (anh) xuất phát hạt đến hạt khác

Lời giải

Ta cải cách toán giới hạn lý thuyết đồ thị Cho G đường gấp khúc Giả sử đỉnh củaGđược tơ màu thích hợp số màu0,1, ,2017 : điểm cuối cạnh tô màu khác Gọif(x)là màu đỉnhx.Chứng minh đổi màu đỉnh G thích hợp thành màu0,1, ,2017 cho

Với hai đỉnh avà b có đường nối a=x1, x2, , xn=b thỏa mãn với i= 1, , n−1

ta có

(133)

Xét màu thích hợp củaG.Gọi G1 nhánh cực đại Gthỏa mãn (∗) Hãy cách đổi màu số đỉnh mà ta tăng số đỉnh củaG1 Với đồ thị G0, tập đỉnh G0 kí hiệu V(G0).Gọi

min x∈V(G1), y∈V(G−G1)

(|f(x)−f(y)|) =|f(x0)−f(y0)|,

theo định nghĩa |f(x0)−f(y0)|>1

Giả sử f(x0)> f(y0) Ta đổi màu vài đỉnh G−G0 :các đỉnh màu f(y0)thành f(x0)−1 đỉnh màu f(x0)−1thành f(y0) Khi đó, dễ thấy đường gấp khúc tơ màu cách thích hợp Thật vậy, G−G1 ta hoán đổi qua lại màu f(x0)−1 f(y0) Chú ý rằng,

|f(x0)−f(y0)| cực tiểu sau đổi màu nên điều kiện f(x) 6= f(y), ∀x ∈ G1, y ∈ G−G1 xảy

Trong trường hợp ngược lại f(x0)< f(y0), ta đổi màu đỉnh G−G0 : đỉnh màu f(y0) thànhf(x0) + đỉnh màu f(x0) + thànhf(y0)

Như vậy, sau đổi màu tập V(G1) thêm phần tử y0 Bằng cách tiếp tục trình này, cuối ta nhận màu mong muốn toàn Gthỏa mãn (∗)

Bài 144 Cho tập gồm n đường thẳng mặt phẳng cho khơng có đường thẳng số chúng song song, đường thẳng số chúng đồng quy Một điểm gọi

giao điểm điểm nằm hai số n đường thẳng

1 Chứng minh tồn đường thẳng sốn đường thẳng cho nửa phẳng xác định đường thẳng chứa

(n−1)(n−2) 10

giao điểm

2 Tìm tất giá trị n để cho số giao điểm hai nửa phẳng tạo đường thẳng tìm gần với giá trị

(n−1)(n−2) 10

nhất

Lời giải

Với đường thẳng trongn đường thẳng cho, số giao điểm nằm ngồi đường thẳng m= (n−1)(n−2)

2

Mỗi đường thẳng chia mặt phẳng thành phần Ta ký hiệu plà đường thẳng mà khác số giao điểm nửa mặt phẳng tạo p nhỏ Giả sử số giao điểm nửa phẳng tạo bởip làk với k 6m= (n−1)(n−2)

2 Ta định nghĩa ba (d, A, B)là gồm đường

thẳng dvà hai giao điểm A, B cho A B nằm phía d

Theo bất đẳng thức Cauchy, ta thấy với đường thẳngd số ba (d, A, B)không vượt quák(m−k) Vậy tổng số ba N không vượt nk(m−k)

Trong số ba, ta phân chia chúng làm loại Loại I ba (d, A, B)với A, B nằm đường thẳng Loại II ba lại Giờ ta xét trường hợp đặc biệt

Bộ đường thẳng tùy ý trongn đường thẳng trên, ta thấy đường thẳng với giao điểm chúng tạo tạo ba loại I Hơn nữa, ba (d, A, B) tương ứng với đường thẳng Do đó, số ba thuộc loại I tạo bởin đường thẳng

n

4

Bộ đường thẳng tùy ý trongn đường thẳng trên, ta thấy đường thẳng với các

(134)

Hình vẽ cho trường hợp đường thẳng

Do đó, ta có N >4

n

4

+

n

5

=n

n

4

Vậyk(m−k)>

n

4

= m(m−1)

6 Từ đây, ta suy

ra

k >k0 =

1

2 m−

r

m2+ 2m

3

!

Dễ thấy k0 > m

5 n >9và n= 8, ta cók0 >

hm

5

i

= Vậy với n>8 ta có

k >

(n−1)(n−2) 10

Vớin 67, hiển nhiên ta có điều phải chứng minh n64vì hm

5

i

(135)

Vậy trường hợp ta có k>

(n−1)(n−2) 10

Ý (b) suy từ ý (a), ta có với n= 5,6,7 k xác

(n−1)(n−2) 10

Chú ý.Ta để ý đường thẳng tạp nhiều ba loại II

Do đó, chứng minh tồn đường thẳng chia mặt phẳng thành phần cho có phần chứa

1 2−

1 2√15

m giao điểm

Bài 145 Cho số nguyên dươngk Ban đầu,N ô vuông đánh dấu bảng ô vuông vô hạn Ta gọi “hình chữ thập” ô A tập hợp tất ô nằm hàng cột chứa A Tại bước, ta phép đánh dấu A hình chữ thập A chứa k đánh dấu Xác định giá trị nhỏ N để đánh dấu vng bảng theo bước

Lời giải

Ta chứng minh

N =

k+

·

k+ 2

=

(

m(m+ 1) k = 2m m2 k = 2m−1

Kí hiệuN(k)là giá trị cần tìm Đặt f(k) =

k+

·

k+ 2

Đầu tiên ta chứng minh

N(k)>N(k−1) +

k+

khi k>2 (∗)

Sau đánh dấu N(k) ô vuông, tồn ô vuông A cho cột hàng A có

k+

ơ vng đánh dấu, ta xóa cột hàng này, nhận mặt phẳng Trên mặt phẳng mới, ta đánh dấu ô vuông chữ thập ô vuông có k−1 đánh dấu Do lúc đầu mặt phẳng có N(k−1) ô đánh dấu, ta nhận bất đẳng thức(∗)

Vì N(1) = 1, nên từ (∗)suy

N(k)>1 + + + + + + +· · ·

| {z }

ksố hạng

(136)

× × ×

× ×

a1 a2 ×

× ×

× × ×

ap

× ×

× ×

× × ×

× ×

×

q p

Ta cần chứng minh với f(k) đánh dấu, ta đánh dấu ô khác mặt phẳng Ta chứng minh quy nạp Xét hai "cầu thang" độ caop=

k

2

vàq =

k+

, dễ thấy cóf(k)

ơ vng Khik = 1, khẳng định hiển nhiên Giả sử mệnh đề với k−1, dop+q =k nên ta đánh dấu ô a1, a2, , ap ô khác hàng với ô Khi xóa tất

các a1, a2, , ap ô khác hàng với ô giá trị k giảm 1, sử dụng giả thiết

quy nạp ta đánh dấu tất cịn lại

Bài 146 Trên bảng vngn×n, ta đánh dấu số ô vuông cho ô bên trái L ô bên phảiR không đánh dấu, đường mã từ L đến R chứa đánh dấu Hỏi với số n > khẳng định ln có vng liên tiếp đường chéo mà số chúng đánh dấu?

Lời giải

Kết n = 3k+ 1, k số nguyên dương

Nếun = 3k n = 3k+ 2, ta đánh dấu tương ứng hình 18 hình 19, đường từL qua số mang dấu ×

× × × ×

×

× × ×

× × ×

× ×

× ×

× × ×

× ×

×

× × ×

×

Hình 18

× × × ×

× ×

× ×

× × ×

× ×

× ×

× ×

×

× × ×

× ×

×

× ×

(137)

Nếun = 3k+ 1, theo giả thiết củak Với bước, hình 20 Nếu (4,4)hoặc(10,10)

khơng đánh dấu, tốn giải Khơng tính tổng qt, (3,2) chưa đánh dấu Thế (30,20) đánh dấu (nếu khơng tốn giải quyết), (20,30) không đánh dấu (2,3) đánh dấu Trong trường hợp này, ta đánh dấu đường hình 20

1

1

40 30 20 10

40 30 20 10

Hình 20

Bài tốn giải

Bài 147 Giả sửn số số nguyên dương Một ma trận cỡn×n (một bảng hình chữ nhật số n dòng vàn cột) gọi bạch kim thỏa mãn điều kiện:

(i) n2 bảng số nguyên từ đến n;

(ii) Mỗi dòng, cột đường chéo (phái góc bên trái đến góc bên phải) chứa số nguyên từ 1đến n lần

(iii) Tồn tập hợp gồm n phần tử chứa số nguyên từ1 đến n cho khơng có hai phần tử nằm hàng cột nằm đường chéo ma trận

Tìm tất giá trị n cho ma trận n×n bạch kim Lời giải

Dễ thấy khơng có ma trận bạch kim với n = n = Chúng ta chứng minh tồn ma trận bạch kim với mọin >3

Thật vậy, trước hết ta định nghĩa đường ngang tập hợp gồmn phần tử ma trận lấy từ dòng cột khác chứa số nguyên từ1 đến n

Khi n>3 n số lẻ, ta xây dựng ma trận bạch kim cỡ n×n theo cách:

Trước tiên, ta điền vào dịng thứ ma trận số1, n, n−1, ,3,2theo thứ tự từ trái qua phải

(138)

là 1cho đến xuống hàng cuối Chú ý phần tử viết dạng modn, trường hợp lớp thặng dư là0 viết

Ví dụ vớin = ta có ma trận bạch kim sau đây:

 

1 3 2

 =    +    +    

Từ cách xây dựng trên, xác định phần tử dòng i, cột i aij = 2i− j(modn)

(16i, j 6n)

Cố định dòng thứi, phần tử cột khác j j0 khác

Cố định cột thứ j, phần tử dòng khác i i0 khác n lẻ

Đường chéo, phần tử dòng thứ 2, đường ngang Hơn nữa, đường chéo đường ngang Vì thế, ma trận xác định bạch kim

Xét trường hợp n >8là chẵn, n−3>5 lẻ

Xét ma trận bạch kim cỡ(n−3)×(n−3) ký hiệu Cn−3 theo cách xác định Ta bắt đầu xây dựng ma trận bạch kim cỡn×n sau:

Cn−3 C3

, C3 =

 

n−2 n n−1

n n−1 n−2

n−1 n−2 n

 

Tiếp theo, ta cần xác định ma trận cỡ 3×(n−3)và ma trận cỡ (n−3)×3 kề với Cn−3

Do (n−3)−1 = n−4 >4 đường chéo Cn−3 khác với đường chéo chính, ta chọn đường chéo khác

Ta đường chéo, ngoại trừ đường chéo ma trậnCn−3 đường ngang Chọn đường ngang củaCn−3 chiếu phần tử đến dịng rỗng cột rỗng Sau đó, thay tất phần tử chọn dòng ngang bởin−2

Chúng ta tiếp tục với hai dòng ngang khác, thay phần tử dịng ngang làn−1và n Khi đó, ma trận tạo thành ma trận bạch kim

Thật vậy, ta có n−4>4 dịng ngang ma trận bạch kim Cn−3 Do tồn dòng ngang chưa sử dụng Kết hợp dòng ngang với dịng ngang C3 ta nhận dịng ngang ma trận cỡn×n Như vậy, ma trận xác định bạch kim

Ta minh họa cách xây dựng mà trận bạch kim với n = Ma trận cuối bạch kim với phần tử ô vng tạo thành dịng ngang ma trận

           

1 3 5 2 4

6 8 7

            −→            

1

3

5

2

6

5

4

8 7

            −→ −→            

1

3

5

7

6

5

4

2

6 8 7

            −→             

1

3

8

7

6

5

1

2

3

4

4

2

5

6

8

7

(139)

Cuối ta xét ma trận sau

   

1 4 2

   

       

1 6 4 6 3

       

Suy ma trận bạch kim với n= n =

Bài 148 Trong câu lạc nọ, có số cặp thành viên bạn bè Giả sửk >3, ta nói câu lạc làk-tốt nhóm k thành viên ngồi xung quanh bàn tròn cho hai người ngồi kề bạn bè Chứng minh câu lạc 6-tốt 7-tốt

Lời giải

Xét câu lạc 6-tốt kí hiệu A, , G thành viên Để thỏa mãn yêu cầu ta cần A, , G ngồi xung quanh bàn Chỉ xét tình bạn số A, , G Đầu tiên, ta thấy thành viên có nhất3người bạn Không tính tổng quát, xét thành viên G, từ giả thiết, B, , G ngồi thỏa mãn yêu cầu Do đó, G có hai người bạn Khơng tính tổng quát, F người bạn G Từ giả thiết, A, , E, G (bỏ qua F) ngồi xung quanh bàn trịn theo u cầu, G có hai người bạn ngồi F tổng số ba người bạn

Từ thành viên có nhất3người bạn, tồn thành viên có nhất4người bạn(nói cách khác số cặp bạn bạn thân

2 ·7·3,mà điều rõ ràng khơng thể) Khơng tính

tổng qt, giả sử G có 4người bạn

Từ giả thiết,A, , F ngồi thỏa mãn yêu cầu Trong số chỗ ngồi vậy, vài cặp bạn bè hai số bốn người bạn củaG ngồi cạnh để Gngồi họ Chú ý Mệnh đề " Nếu câu lạc bộk-tốt (k+ 1)-tốt k ∈ {3,4,5,6,7,8,10,11,13,16} Các phản ví dụ đồ thịhypo-Hamilton Vớik = 9, ví dụ đồ thị Petersen (hình 1)

Hình

Bài 149 Có 300 người chơi thi Trong đó, người chơi quen biết không quen biết nhau; người có người khơng quen Xác định giá trị lớn n mà thỏa mãn điều kiện:

(140)

Với số nguyên dương m thỏa mãn 16m 6n, tồn nhất 1người chơi quen biết với m người chơi khác

Lời giải

Trước hết, ta chứng minh n6 200 Giả sử n >200 thỏa mãn điều kiện đề Khi có người chơiX quen biết với 201 người chơi khác Gọi S tập hợp gồm 201 người chơi Luôn tồn tại1

người biết 1,2, ,200 người chơi khác

Ta nói người chơi có bậc m người biết m người chơi khác

Ta có X quen biết với tất người chơi tập S Mà người bất kì, ln tồn

2người không quen biết nên 2người chơi tập S khơng biết Do phần tử củaS có bậc tối đa 300−201 = 99

Người chơiX nhận1 bậc Những người chơi thuộc tậpS nhận tối đa 99bậc phân biệt(từ bậc1 đến bậc 99) Do có 98người khác X, khơng thuộc tậpS nên 98người nhận tối đa 98bậc phân biệt Suy có tối đa + 99 + 98 = 198<201 bậc phân biệt Do khơng thể có người chơi biết 1,2, ,201 người chơi khác Suy ran 6200

Bây ta chứng minhn = 200 thỏa mãn u cầu tốn

Kí hiệu 100 người chơi A1, A2, , A100 gọi họ A−người chơi Kí hiệu 200 người lại B1, B2, , B200 gọi họ B−người chơi Xét A−người chơi B−người chơi thỏa mãn:

Với mỗi i∈ {1,2, ,100}, Ai quen biếtBj với mọij >i. (1) 2người chơi trong A−người chơi không quen nhau, 2người chơi trong B−người

chơi khơng quen (2)

Khi đó:

Từ (2) ta có: Trong ba người chơi bất kỳ, có nhất 2người khơng quen Mỗi người chơi quen biết tối đa 200 người chơi khác

Từ(1) ta có: Với mỗii∈ {1,2, ,100},Ai quen biết đúng201−ingười chơi khác làBi,Bi+1, , B200 Do có người chơi biết 200,199, ,101 người chơi khác

Với j ∈ {1,2, ,100}, Bj quen biết j người chơi khác A1, , Aj−1, Aj Do

có người chơi biết đúng1, 2, ,100 người chơi khác

Vậy n= 200 giá trị lớn thỏa mãn đề

Bài 150 Hai người chơi thay phiên viết lần chữ số, từ trái sang phải Một người coi thua sau lượt người đó, có dãy chữ số a1, a2, , an cho tồn

số nguyênk đểakak+1 an bội của11 Hỏi người thắng cách chơi anh

như nào? Lời giải

Ta chứng minh người chơi thứ hai thắng người chơi thứ viết Tất nhiên không hai người chơi viết số0 vào lượt

Chú ý rằng10r ≡(−1)r (mod 11), đó

akak+1 an≡an−an−1+an−2− · · ·+ (−1)k−1ak (mod11)

GọiNk phần dư chia akak+1· · ·ancho11, với k= 1, , n Ta có sau n lượt, sốN1, , Nn

là số phân biệt, lượt ta có số

an+1−N1, , an+1−Nn, an+1(mod11)

(141)

kết luận có n + chữ số, người chơi viết chúng dẫn anh đến việc thua Giả sử trò chơi kết thúc sau lượt chơi Người chơi thứ hai thua sau lượt, số

{N1, N2, , N9} {1,2, ,9} Người chơi thứ hai thắng 10 số N1, N2, , N9 Nếu sau lượt chơi, {N1, , N8} thiếu hai số không liên tiếp nằm giữa1 và10, người chơi chọn lượt thứ 9, số N1, , N9 phải 10 Thật vậy, người chơi thứ chọn X, tồn k thỏa Nk =X+ (trong lượt thứ 9) Do đó,

sau lượt thứ9 ta có

Nk≡X−(X−1)≡ −1≡10(mod11)

Ta kết luận người chơi thứ hai chắn sau8lượt chơi, số N1, , N8 khơng có hai số liên tiếp Và người chơi thứ hai đảm bảo trị chơi kéo dài 7lượt chơi để

N1, , N7 = 1,2, ,10\ {X, Y, Z}

Nếu số X, Y, Z khơng có hai số liên tiếp, người chơi thứ hai chọn chúng

NếuY =X+ 1, người chơi thứ hai viết số X X+

Vì vậy, sau lượt thứ 8, có hai số liên tiếp cịn thiếu từ N1, , N8 Thật vậy, Z =X−1, người chơi thứ hai viếtX, Z 6=X−1, người chơi thứ hai viết X+

Bài 151 Xét n−giác với tâm Hai người chơi trò chơi sau: họ chọn đỉnh đa giác nối với hai đỉnh kề nối với tâm hình đa giác đoạn thẳng Người thắng người chơi mà sau lượt chơi đỉnh đa giác di chuyển đến đỉnh lại đoạn thẳng nối Với n>3, xác định xem người có chiến lược thắng

Lời giải

Đáp án: Với n lẻ người thứ trước thắng, với n chẵn người thứ hai thắng Với n chẵn, người thứ trước

• Trường hợp 1: Người thứ nối đỉnh với tâm Người thứ hai nối tâm với đỉnh khác Như vậy, sau người thứ hai thực ln có số lẻ đỉnh kết nối Do đó, người thứ khơng thể nối hai điểm cuối Khi đó, người thứ hai chiến thắng

• Trường hợp 2: Người thứ nối hai đỉnh Người thứ hai nối đoạn thẳng đối xứng với đoạn thẳng người thứ vẽ qua tâm Sau bước có số điểm nối lại số lẻ Người thứ hai tiếp tục thực bước đến người thứ di chuyển Giả sử có lúc người thứ chiến thắng người thứ hai trước Gọi x đoạn cuối nối người thứ bước thứ x x∗ đoạn đối xứng với nó, z đoạn mà người thứ chiến thắng người thứ hai vẽ x∗ Chú ý x đoạn qua tâm, tương tự trường hợp Xét điểm lại trước thực bước x Đoạn x nối điểm khác A B, đoạn x∗ nối hai điểm đối xứng tương ứng khác A∗ B∗ Xét O tâm đường tròn Giả sử O hai điểm A B, A∗ Điều có nghĩa A = A∗ đối xứng tâm chứa tâm đường tròn Bởi vậy, sau thực bước x∗ số điểm lẻ giống bước x Do đó, bướcz khơng chiến thắng Giả sử O khơng thuộc A B Khi có điểmC, có tâm đường trịn Để thực bướcz chiến thằng, phải nối C với điểm đẵ có từ bước x x∗ Sau thực bước x x∗, điểm nối lại không bao gồm tâm, B =A∗, A=B∗ bước x∗ không làm giảm số điểm Vậy người thứ hai chiến thằng việc thực bước z bao gồm x∗ Vớin lẻ, ta chứng minh người thứ chiến thắng nối đỉnh tùy ý với tâm

(142)

điều Xétn = 2k+ 1, đánh số đỉnh đa giác theo chiều kim đồng hồ: A0,A1, , A2k+1 Người thứ nối A0 với tâm O đường trịn bước Xét người thứ hai đầu tiên:

• Nối đỉnh khác A0 Giả sử người thứ hai nối Ai−1 với Ai 26i 6k+ Người

thứ nối Ai với Ai+1, trị chơi trở thành trường hợpn = 2k−1 Nghĩa là, đỉnh Ai−1, Ai, Ai+1, có đỉnh nối từ Ai−2 qua Ai−1, từAi+2 qua Ai+1, từ tâm qua Ai, ngồi có hai đường phụ nối từ tâm đường trịn khơng đóng

vai trị Người thứ tạo hai đường người thứ hai làm

• Nối A0 với đỉnh liền kề Giả sử người thứ hai nối A1 với A0 Người thứ nối A0 với A2n+1, trò chơi trở trường hợp n= 2k−1 Nghĩa là, đỉnhA2k+1,A0,A1 có đỉnh (nối từ O bước thứ người thứ nhất) tương tự trường hợp

• Nối O với đỉnh liền kề A0 Giả sử người thứ hai nối A1 với O Người thứ nối A2k+1 với O, trò chơi trở trường hợpn= 2k−1 Nghĩa là, đỉnhA2k+1,A0,A1, đỉnh tương tự trường hợp trước Điều khác biệt bao gồm hai điểm phụ nối tới O, điểm A1 A2k+1 có hai đường phụ nối tới A0

• NốiOvới đỉnh khơng liền kề vớiA0 Khơng tính tổng qt, giả sử người thứ hai nối A2i với O Người thứ nốiAi với O, sau trị chơi trở trường hợp n = 2k−2i+

Nghĩa là, xét đỉnh A0 A2, có đỉnh nối với O lần di chuyển người chơi Nếu người chơi thứ hai vẽ đoạn thẳng nằm trong miền sau OA0Ai OAiA2i, người chơi thứ vẽ đoạn thẳng đối xứng với OAi Có số chẵn bước phụ khơng đóng vai trị gì, trị chơi giảm tính

chính xác với số lượng

Trong trường hợp, trị chơi giảm xuống tương đương với số số lẻ đỉnh Vậy người thứ có chiến lược chiến thắng với số lẻ n>3

Bài 152 Một đa giác ô li A vẽ mặt phẳng lưới ô li Ta gọi ô (đơn vị) điểm A ô liền kề với thuộc A, cịn lại A mà điểm A gọi điểm biên A Biết tập tất điểm biên A thỏa mãn đồng thời hai điều kiện sau

1 điểm biên có hai cạnh kề với hai điểm biên khác;

2 tập tất điểm biên phân chia thành hình thang cân có diện tích

với đỉnh nằm nút lưới (và góc đáy hình thang cân bằng45◦)

Chứng minh diện tích đa thức li A đồng dư 1với modulo

Lời giải

Rõ ràng tập điểm biên đa thứcA chia thành hình thang, ta từ điểm đến điểm khác thơng qua cạnh hình thang Ta gọi đa thức thỏa mãn điều kiện đa thức tốt Ta chứng minh quy nạp tốn “nếu diện tích đa thức tốt làS thỏa mãn S 64k+ (với k>2) S ≡1 (mod 4)”

Với k = 2: dễ thấy đa thức ô li A có hình thang, có hình thang đa thức li A hình vng 3×3 Nếu có hình thang diện tích S >2·5>2·4 +

(143)

• Giả sử AB = đa thức liA khơng phải hình vng 3×3

Khi hai hình thang cân xếp lại cách tạo hình chữ nhật 3×4 từ đa thức li ta nhận đa thức tốt Rõ ràng với nhát cắt, phần dư phép chia S cho 4sẽ khơng đổi

• Giả sử đa thức tốt khơng có cạnh (nếu đa thức tốt có cạnh cạnh vng góc với nhau), cạnh bên có hai hình thang cân (nói cách khác cạnh hình thang 3)

Giả sử AB=` Nếu hình chữ nhật CABD (như hình vẽ bên) có kích cỡ `×4 khơng có điểm biên chấp nhận được, ta có điểm chung với đa thức đường CABD Khi cắt bỏ đa thức ta nhận đa thức tốt cách di chuyển hình thang cân tới cạnh AB 4ơ (giống trường hợp AB = 3)

Giả sử hình chữ nhật CABD chứa điểm biên khác Tô màu điểm mặt phẳng hai màu khác (như hình vẽ bên) Khi điểm biên có dạng

sẽ tô màu ô có dạng 2sẽ có màu 2(vì từ dạng ta xác định ô thuộc dạng 2)

Xét tất điểm hình chữ nhật CABD chọn điểm bên khơng có điểm chung với đa thức đường Khi vài điểm kề bên trái bên phải điểm biên ngược lại đường biên khơng phân chia thành hình thang (nếu điểm tơ màu phần hình thang theo chiều ngang, ngược lại điểm không nằm thấp khơng hình thang chứa điểm khơng có chiều dọc với lí tương tự)

Có hai điểm biên kề bị phân chia đường chéo hình thang Khơng tính tổng qt, ta giả sử điểm có dạng (được đánh dấu gạch chéo hình vẽ) Khi đó, điểm có màu1và xác định hai điểm nằm bên có màu kề với AB, nói cách khác khơng thể xác định bên CABD điểm CABD có màu Rõ ràng điểm nằm bên hình thang, xác định bên phải

Bây ta tạo nhát cắt (như hình vẽ), cắt đa thức làm

Bài toán giải

Bài 153 Giả sử có bàn cờ vua với số lượng ô vuông đủ lớn, quân mã dịch chuyển bàn cờ theo quy tắc: nước đi, dịch chuyển qua p liên hàng ngang qua tiếp q ô liên hàng dọc; dịch chuyển qua q ô liên hàng dọc quap ô liên hàng ngang Biết sau đúngn nước qn mã quay vng ban đầu, chứng minh n số chẵn Hình vẽ mô tả số cách cho trường hợp p= 3, q = 2:

Lời giải

Chọn hệ trục tọa độ để tâm ô vuông điểm (x;y) với x, y ∈ Z ban đầu quân mã đứng điểm (0; 0) Ta kí hiệu d= (p, q) Vì =

p

d, q d

nên có hai số p d,

(144)

lẻ Bây ta xây dựng hàmf xác định tập điểm nguyên gắn với nước quân mã cho ta biến động giá trị f số lẻ để suy luận số nước số chẵn

Nếu p d,

q

d số lẻ ta đặtf(x, y) = x

d Rõ ràng nước đi, quân mã di chuyển từ ô(x, y)sang ô (x0, y0)thì hàm f thay đổi lượng f(x0, y0)−f(x, y) = x

0−x

d , số lẻ nhận giá trị bốn số±p

d,± q d Nếu hai số p

d, q

d có số lẻ số chẵn ta đặt f(x, y) = x+y

d Ở nước đi, quân mã di chuyển từ ô(x, y) sang ô(x0, y0)thì hàm f thay đổi lượng

f(x0, y0)−f(x, y) = x

0+y0−x−y d

Do x0 = x±p y0 = y±q nên giá trị hiệu số ±p+q

d ,

±p−q

d , suy f(x

0, y0)−f(x, y) là số lẻ.

Trở lại toán Ban đầu quân mã ô (0; 0), sau nước thứ đến (x1;y1), sau nước thứ hai đến (x2;y2), sau nước thứ n đến (xn;yn)≡ (0; 0), tổng độ thay

đổi hàmf

0 =f(0; 0)−f(xn;yn) = (f(0; 0)−f(x1;y1))+(f(x1;y1)−f(x2;y2))+· · ·+(f(xn−1;yn−1)−f(xn;yn))

Như số 0được phân tích thành tổng n số lẻ, có n số chẵn Bài 154 Trong dịp hè2018, Viện nghiên cứu cao cấp Toán (VIASM) tổ chức6bài giảng dành cho sinh viên Biết giảng có 100 sinh viên tham dự khơng có hai sinh viên mà hợp lại tham dự đủ 6bài giảng Hỏi có sinh viên tham dự giảng VIASM mùa hè 2018

Lời giải

Ta thấy khơng thể có sinh viên tham dự5bài giảng, ngược lại, ta cần lấy1 sinh viên tham dự giảng số6là được2sinh viên mà hợp lại dự đủ cả6bài giảng Nếu có 1sinh viên tham dự4 giảng, giả sử 1, 2,3, khơng có sinh viên tham dự hai giảng 5, Suy 100 sinh viên tham gia giảng 100 sinh viên tham dự giảng khác Như trường hợp ta có 201 sinh viên Nếu sinh viên tham dự không giảng tổng số lượt tham dự là600 nên số sinh viên tham dự không nhỏ 600 : = 200 Ta trường hợp có đúng200 sinh viên tham dự giảng thỏa mãn yêu cầu toán Ta chia

200 sinh viên thành nhóm, nhóm 50sinh viên, nhóm tham dự giảng theo bảng sau

I II III IV

1 × ×

2 × ×

3 × ×

4 × ×

5 × ×

6 × ×

(145)

Bài 155 Bạn Thanh viết lên bảng số 1, 2, 3, , 2019 Mỗi bước Thanh xóa hai số a b bảng viết thêm số ab

a+b+ Chứng minh dù xóa

sau thực 2018 bước bảng ln cịn lại số

2019

Lời giải

Với tậpT ={a1;a2; .;an} số viết bảng đặt A(T) =

1

a1

+ 1

a2

+

· · ·

1

an +

⇒A({1; 2;· · · ; 2019}) = 2020

Ta thấy:

1

a +

1

b +

= (a+ 1) (b+ 1)

ab =

1

ab a+b+

+

Suy xóa hai số a vàb thay ab

a+b+ 1, tập T biến thành tập T

0 thì A(T) =A(T0).

Sau thực 2018 bước ta lại số bảng ta gọi số thực x, ta có

A({x}) =

x + = 2020⇒x=

1 2019

Vậy bảng ln cịn lại số

2019

Bài 156 Cho n nguyên dương, gọi S tập gồm 2n+ 1 phần tử, X là tập tất tập con

của S,Y ={0,1, ,2n−1−1} Xét ánh xạ f :X →Y thỏa mãn: Với mọix, y, z ∈S, trong 3 số f({x, y}), f({y, z}), f({z, x}), có số tổng hai số lại Chứng minh tồn a, b, c∈S cho f({a, b}) =f({b, c}) = f({c, a}) =

Lời giải

Với n= 1, S có 3phần tử S={a, b, c} Y ={0} nên đương nhiên f({a, b}) =f({b, c}) = f({c, a}) =

Vớin = 2,S ={a, b, c, d, e}vàY ={0,1}, tậpX có10phần tử, cố địnha∈S, quy ướcf({a, a}) = 0, xét tập U ={x ∈S, f({x, a}) = 0};V ={y ∈S, f({y, a}) = 1} Khi tồn

tập U V, có số phần tử lớn

Nếu tập làU thì tồn tại 3phần tử a, x, y thuộc U, rõ ràng

f({x, a}) = f({y, a}) = f({x, y}) = Nếu tập làV thì tồn tại x, y, z ∈V, đó

f({x, y}) = f({y, z}) = f({z, x}) =

Với n = 3, ta có S gồm phần tử, Y = {0,1,2,3} Cố định a ∈ S Chia tập S thành tập U ={x∈S, f({x, a})chẵn}và V ={y∈S, f({y, a})lẻ} Khi tồn tập có 5phần tử

Nếu tập làU U đóng vai trị S trường hợp trên, f({x, y})chẵn x, y ∈U, toán giải vớiY0 ={0,2} đóng vai trị Y

(146)

Từ trường hợp trên, ta có ý tưởng quy nạp sau Giả sử toán cho trường hợpn =k, xét tập S gồm 2k+1 phần tử, cố định a ∈ S Chia S thành tập U = {x ∈ S, f({x, a})chẵn} V ={y∈S, f({y, a})lẻ} Khi tồn tập có

2k+1+ 1

2

= 2k+ 1phần tử, giả sử làU Khi

đó với mọix, y ∈U thìf{x, y})chẵn vìf({x, y}) +f({x, a}) +f({y, a})chẵn f({x, a}) +f({y, a})

chẵn, từ f{x, y} ∈ {0,2, ,2k−2} Xét g({x, y}) =

2f({x, y}), ta chuyển toán trường

hợp trước Từ có điều phải chứng minh

Bài 157 (IMO Shortlisted 1987, problem 18) Với số nguyên r > 1, gọi h(r) số nguyên nhỏ thỏa mãn h(r) > phân hoạch tập {1,2, , h(r)} thành r lớp, tồn số nguyêna >0và số nguyên x, y : 16x6ysao cho a+x, a+y, a+x+y thuộc lớp Chứng minhh(r) = 2r

Lời giải

Xét trường hợp đặc biệt nhất, khih(r) = 2r, phân tập {1,2, ,2r} thành r lớp, lớp có phần tử có chuyệna+x, a+y, a+x+y (3 phần tử) thuộc lớp Phải đề sai? Khơng! Vấn đề làxvàycó thể nhau, đóa+x, a+y, a+x+ychỉ nhận giá trị nên khơng có vơ lý Tuy nhiên, điều câu hỏi lại sở giúp ta giải tốn nhờ phát chất Trước hết, ta chứng minh h(r) = 2r phân hoạch tập {1,2, ,2r} thành r lớp có lớp chứa sốa+x, a+y, a+x+ynào Thật vậy, xétr+ 1sốr, r+ 1, r+ 2, ,2r Vì có r lớp nên tồn lớp chứa phần tử Giả sử r+i, r+j thuộc lớp với i < j Khi đó, chọn x= y=j −i, a=r+ 2i−j a+x=r+i, a+y=r+i, a+x+y=r+j thuộc lớp Rõ ràng 6x 6 y a >0 Vậy h(r) = 2r thỏa mãn Nếu h(r) <2r xét tập {1,2, ,2r−t} Phân hoạch tập thành r tập

{1, r+ 1};{2, r+ 2}; .;{r−t,2r−t};{r−t+ 1};{r−t+ 2}, ,{r}

Khi đó, giả sửa+x, a+y, a+x+ythuộc tập Do a+x, a+y, a+x+y không nên chúng phải thuộc trongr−t tập Như vậyx=y vàa+x, a+ 2x thuộc tập, giả sử a+x = k, a+ 2x = r+k (k 6 r−t) a = r, a = k−r < Mâu thuẫn Vậy

h(r)>2r nên minh(r) = 2r

Nhận xét Điểm mấu chốt toán nảy sinh từ việc: a=y, a+x, a+x+y =a+ 2xthuộc lớp Màa+x < a+ 2x <2a+ 2xnên có cách phân hoạch lời giải

Bài 158 Cho số nguyên dương n1, n2, , n6 n(f) = n1nf(1)+n2nf(2)+· · ·+n6nf(6), f hoán vị {1,2, ,6} Đặt Ω =n(f) :f hoán {1,2, ,6} Tìm giá trị lớn số phần tử Ω

Lời giải

Bổ đề Nếu 2x1 + 2x2 +· · ·+ 2xn = 2y1 + 2y2 +· · ·+ 2yn với x

1, x2, , xn số nguyên dương

phân biệt y1, y2, , yn số nguyên dương phân biệt {x1, x2, , xn}={y1, y2, , yn}

(Một số nguyên dương có biểu diễn hệ số 2) Ta xétni = 22

i

Khi

n(f) =n(g)⇔

6

X

i=1

ninf(i) =

X

i=1

ning(i)

6

X

i=1

22i+2f(i) =

6

X

i=1

22i+2g(i)

(147)

Khi đó2i+ 2f(i) = 2i+ 2g(i) với inào {1,2, ,6} Tức {1, f(1)}={i, g(i)} Có trường hợp

i) g(i) = 1, f(1) =i suy f(1) =g−1(1)

ii) g(i) = f(1), i= suy f(1) =g(1)

Vậy {f(1), f−1(1)}={g(1), g−1(1)} Tương tự {f(i), f−1(i)}={g(i), g−1(i)}với i= 1,2, ,6 Bây giờ, xét hoán vị f bất kỳ, ta xem xét số hoán vịg chon(f) = n(g), từ đếm số giá trị mà n(f) nhận

Ví dụ.f = (3,4,1,6,5,2)thìf(1) = 3, f(3) = 1nên nếun(f) = n(g)thìg(1)∈ {f(1), f−1(1)}={3}

nên g(1) = 3, tương tự g−1(1) = 3nên g(3) = 1, g(5) =

Còn lạig−1(2), g(2)∈ {f(2) = 4, f−1(2) = 6}, tức làg(2)6= g−1(2)6= 2, nên

{g(2), g(4), g(6)}={2,4,6}

Từ đóg(2) = 4, g(4) = 2, g(6) = 6hoặc g(2) = 4, g(6) = 2, g(4) = 6hoặc g(2) = 6, g(4) = 2, g(6) =

hoặc g(2) = 6, g(6) = 2, g(4) = Nhưng g(6) = f(6) = (mâu thuẫn) g(4) = f(4) = 4(mâu thuẫn) Vậyg(2) = 4, g(4) = 6, g(6) = 4(g trùngf) hoặcg(2) = 6, g(6) = 4, g(4) = Như vậy, có thêm hoán vịg mà n(g) = n(f)nên hoán vị này, kể cảf cho giá trị n(f) Ta tổng qt hóa tốn từ ví dụ trên.Ta gọi C vịng xích f có độ dài k tồn tạia ∈ {1,2, ,6} cho

C ={f(a), f2(a), f3(a), , fk(a) = a}

và fi(a)6= fj(a),∀i 6=j,1 6i, j k Khi đó, hốn vịf ln phân tích thành

vịng xích Khi f có vịng xích độ dàix1, x2, , xm, ta nóif hốn vị loại x1+x2+· · ·+xm

Chẳng hạn,f = (3,4,1,6,5,2) f có vịng xích {f(1) = 3, f(3) =f2(1) = 1} có độ dài

{f(2) = 4, f2(2) =f(4) = 6, f3(2) =f(6) = 2}

có độ dài 3, {f(5) = 5} có độ dài Khi đó, ta viết f = (2,4,6)(1,3)(5), rõ ràng cách viết hồn tồn xác định hốn vị f Ví dụ f = (3,4,1,6,5,2) hốn vị loại + + Bằng cách tường minh ví dụ hốn vị loại + + 1ở với loại hoán vị khác, ta có kết thể bảng sau (chú ý hoán vị tập {1,2, ,6} thuộc loại sau: + + + 1; + + 1,3 + 3; + + 1; + 2; + 1; hoán vị khơng có vịng xích có độ dài lớn 2)

Loại hoán vị Số hoán vị f Số lần lặp giá trị n(f) Số giá trị n(f) + + +

6

.2! = 40 20

3 + +

6

.2!

3

= 120 60

3 +

6

.2! = 40 10

4 + +

6

.3! = 90 45

4 +

6

.3! = 90 45

5 +

6

.4! = 144 72

6 5! = 120 60

Còn lại 76 76 76

Vậy nhiều có388 giá trị có tậpΩkhin = 22i Còn lại n= 22i số lần lặp Điều dễ dàng với giá trị ni khác, ngồi việc lặp trên, cịn có thêm khả

(148)

Bài 159 Vớinlà số nguyên dương tùy ý,n >3, xétk =

1

6n(n+ 1)

và tậpXngồm

n(n+ 1)

phần tử, đókphần tử màu đỏ,kphần tử màu xanh, cịn lại màu trắng Chứng minh chia tập Xn thànhn tập rời A1, A2, An cho với sốm tùy ý, 16m6n

thì tập Am chứa m phần tử phần tử màu

Lời giải

Kiểm tra vớin = 4,5,6,7,8,9,10,11ta có bảng sau

n n(n2+1) k Các phần tử Các phần tử Các phần tử Phân bố màu

màu xanh màu đỏ màu trắng (X-Đ-T) 10 |A1|= 1,|A2|= |A3|= |A4|= - -

5 15 |A1|= 1,|A4|= |A2|= 2,|A3|= |A5|= 5 - -

6 21 |A1|= 1,|A6|= |A3|= 3,|A4|= |A2|= 2,|A5|= - -

7 28 |A4|= 4,|A5|= |A3|= 3,|A6|=

|A1|= 1,|A2|=

9 - - 10

|A7|=

8 36 12 |A5|= 5,|A7|= |A4|= 4,|A8|=

|A1|= 1,|A2|=

12 - 12 - 12

|A3|= 3,|A6|=

9 45 15 |A6|= 6,|A9|= |A7|= 7,|A8|=

|A1|= 1,|A2|=

15 - 15 - 15

|A3|= 3,|A4|= |A5|=

10 55 18 |A1|= 1,|A2|= |A3|= 3,|A6|= |A4|= 4,|A7|= 18 - 18 - 19

|A5|= 5,|A10|= 10 |A9|= |A8|=

11 66 22 |A1|= 1,|A4|= |A2|= 2,|A3|= |A5|= 5,|A8|= 22 - 22 - 22

|A6|= 6,|A11|= 11 |A7|= 7,|A10|= 10 |A9|=

Để giải toán trường hợp tổng quát ta cần cách chia tổng quát quy trường hợp n lớn trường hợp n nhỏ Qua việc nghiên cứu trường hợp nhỏ lẻ, thấy khó tìm quy luật tổng quát cho việc phân chia Bởi vậy, ta thử tìm cách đưa trường hợp sau trường hợp trước Các trường hợpn = 5,6,7,8,9cũng gần khó đưa trường hợp trước Nhưng vớin = 10ta đưa n= 4, số lượng chênh màu nhau, từ 3−3−4 lên 18−18−19, tức chênh lên 15 ta cần có phần, phần 15 phần tử màu, chia thành tập có nhiều phần tử (3−3−4 lên

9−9−10chênh tập lên phần tử, tập nhiều phần tử khơng thể làm vậy) Vậy ngun nhân đưa từ trường hợp n = 10 trường hợp n = vì: Mỗi tập chênh 15 phần tử, tập có số phần tử56x610nên chia 15 = + 10 = + = + Trường hợpn = 5(5−5−5)sang trường hợp (22−22−22) chênh lên 17 phần tử tập, số phần tử tập thuộc{6; 7; 8; 9; 10; 11} mà 17 = + 11 = + 10 = + Từ đây, ta có lời giải tổng quát quy nạp với việc xây dựng cách chia tập Xn dựa vào cách chia tập Xn−6 sau: Xét tập Xn

gồm n(n+ 1)

2 phần tử Ta xét tập Xn−6 gồm

(n−6)(n−5)

2 phần tử gồm

k1 =

1

6(n−6)(n−5)

phần tử màu xanh, k1 phần tử màu đỏ lại màu trắng Theo giả thiết quy nạp, tập Xn−6 chia thành n−6 tập rời nhauA1, A2, , An−6 thỏa mãn tốn Ta có

k−k1 =

n(n+ 1)

(n−6)(n−5)

Mặt khác[a]−[b]> a−1−[b]>a−1−b và[a]−[b]6a−[b]< a−(b−1) =a−b+ Nếua−b số nguyên [a]−[b] =a−b Mà

n(n+ 1)

6 −

(n−6)(n−5)

(149)

là số nguyên nên k−k1 = 2n−5 Vậy số phần tử màu xanh (đỏ) ngồi Xn−6 2n−5 Có

|Xn| − |Xn−6|= 6n−15nên số phần tử màu trắng Xn−6 là6n−15−2(2n−5) = 2n−5 Khi ta xây dựng tập An−5, An−4, An−3, An−2, An−1, An sau

TậpAn−5, An chứa toàn phần tử màu xanh. TậpAn−4, An chứa toàn phần tử màu đỏ. TậpAn−2, An−3 chứa toàn phần tử màu trắng.

Bài toán giải

Bài 160 (IMO Shortlisted 2012) Cho n > số nguyên, tìm số lớn tập rời có hai phần tử tập {1; 2; 3; .;n} cho

i) Tổng phần tử tập không vượt n

ii) Tổng phần tử tập khác số nguyên dương khác Lời giải

Giả sử k số lớn tập rời thỏa mãn đề Ví dụ

1 Với n = 3, có tập có phần tử mà tổng phần tử không tập

{1; 2} nên k = với tập {1; 2}

2 Với n= 4, có hai tập{1; 2},{1; 3}thỏa mãn tổng phần tử không hai tập không rời nênk = với tập {1; 2}

3 Với n = 5, có tập {1; 2},{1; 3},{1; 4},{2; 3} thỏa mãn tập đầu có chung phần tử nên chọn tập, đók 62 Nhưng vớik = chọn tập{2; 3},{1; 4}

mà hai tập có tổng nên cuối k = 1, với tập {1; 2}

4 Vớin= 6, tập{1; 2},{1; 3},{1; 4},{1; 5}chọn tập; tập{2; 3},{2; 4}

chọn tập Nên k = 2với tập {2; 4},{1; 3} {2; 3},{1; 4}nhưng cặp {2; 4},{1; 3} cho tổng khác Vậy k = với tập {2; 4},{1; 3}

5 Với n = 7, tập {1; 2},{1; 3},{1; 4},{1; 5},{1; 6} chọn tập Trong tập

{2; 3},{2; 4},{2; 5} chọn tập Với tập chọn với tập {3; 4} nên k 63, với tập {3; 4},{2; 5},{1; 6} Tuy nhiên chọn tập yêu cầu tổng phần tử khác nênk 62 Vớik = chọn tập {2; 4},{1; 3} Vậyk = chọn tập {2; 4},{1; 3}

6 Với n = 7, tập {1; 2},{1; 3},{1; 4},{1; 5},{1; 6},{1; 7} chọn tập Trong tập {2; 3},{2; 4},{2; 5},{2; 6} chọn tập Trong tập {3; 4},{3; 5} chọn tập Vậy vớik = với tập {3; 4},{2; 6},{1; 2}chẳng hạn

Tiếp tục ta có bảng sau

n 10 11 12 13 14 15 16 17 18 19 20

k 0 1 2 3 4 5 6 7

Theo bảng dự đốn quy luật lặp lại theo chu kỳ với n = 5m+ 1, n= 5m+ cho đáp số k = 2m Với n = 5m+ 3, n = 5m+ 4, n = 5m+ cho đáp số k = 2m+ Từ ta nghĩ cách tổng quát để phân hoạch tập{1; 2; 3; .;n}thành tập có phần tử thỏa mãn đề trường hợp với số k lớn số định khơng thể phân chia Ngồi ra, với hai trường hợp n= 5m+ n= 5m+ chọn chung tập Vậy chủ yếu làm việc với hai trường hợpn = 5m+ n= 5m+ Ta có lời giải sau

(150)

{1; 4m},{3; 4m−1},{5; 4m−2}, ,{2m+ 1; 3m} cóm tập.

{2; 2m} có tập

{4; 3m−1},{6; 3m−3},{8; 3m−5}, ,{2m−2; 2m+ 2} có(m−1)tập

Ngồi k>2m+ tổng S k cặp thỏa mãn:

S >(1 + 2) + (3 + 4) + .+ (4m+ + 4m+ 2) = (2m+ 1)(4m+ 3)

S 6(5m+ + 5m+ 1) + .+ (3m+ + 3m+ 2) = (2m+ 1)(4m+ 2)

Điều dẫn đến mâu thuẫn

Với n= 5m+ 3, n= 5m+ n= 5m+ ta chọn 2m+ tập gồm

{1; 4m+ 2},{3; 4m+ 3}, ,{2m+ 1; 3m+ 2} cóm+ tập

{2; 3m+ 1},{4; 3m}, ,{2m; 2m+ 2} cóm tập.

Ngồi k>2m+ tổng S k cặp thỏa mãn:

S >(1 + 2) + (3 + 4) + .+ (4m+ + 4m+ 4) = (2m+ 2)(4m+ 5)

S 6(5m+ + 5m+ 4) + .+ (3m+ + 3m+ 4) = (2m+ 2)(4m+ 4)

Điều dẫn đến mâu thuẫn Vậy đáp số cuối

2m−1

Bài 161 Tại đỉnh lục giác viết số ngun khơng âm có tổng 2013 Một người thực thay đổi sau: Chọn đỉnh, thay số đỉnh giá trị tuyệt đối hiệu 2số viết 2đỉnh kề với đỉnh chọn Chứng minh rằng, số lần cho số thu 6đỉnh

Lời giải

Đương nhiên muốn giảm số tổng số phải giảm dần Xét vài trường hợp cụ thể Dễ trường hợp có số đỉnh xen kẽ 0, cịn lại có tổng 2013 Khi cần tác động lần thao tác cho đỉnh khác 0, ta thu trạng thái0 Nếu có 2trong 3số đỉnh xen kẽ 0, ta dễ dàng thu đáp số, cụ thể

A B

0

D

0

F

A A

0

0

A

0 A

0

0

A

0

0

0

(151)

C−E A−C C C−E A−C

A−E

C−E A−C C C−E E

A−E

A A−C C C−E A−C

A−E

A A−C C C−E E

A−E

A A−C

|A+E−2C|

C−E E

A−E

Ta thấy có phép biến đổi

A B C D E

F

A A−C C C−E E

A−E

C−E A−C C C−E A−C

A−E

làm giảm tính lẻ tổngA+C+E khơng có số 0, phép biến đổi đáng lưu ý Từ đặt câu hỏi: Khi có số có đưa trường hợp số 0, khơng có số 0thì nào? Trả lời câu hỏi xem xét biến đổi cụ thể trên, ta có lời giải sau Trước hết, tồn 3đỉnh xen kẽ cho tổng số lẻ (bài toán sử dụng tính lẻ số 2013 nên tổng quát 2013 thành số n lẻ bất kỳ), giả sử đỉnh A, C, E ( điền số A, C, E luôn)

+A>C >E >0 Xét biến đổi sau

A B C D E

F

A A−C C C−E E

A−E

C−E A−C C C−E A−C

A−E

Ta thấy phép biến đổi làm giảm tổngA+C+E thành (C−E) +C+ (A−C) = A+C−E giữ nguyên tính lẻ tổng E khác 0, áp dụng liên tiếp thao tác này, đến trạng thái mà trong3 số A, C, E 2trong số bằng0

+ Nếu có2 3số A, C, E bằng0 Giả sử A > C =E = 0, xét phép biến đổi

A B D F A A 0 A A 0 A 0 0 0

Ta thu trạng thái toàn số +A>C > E = 0, A lẻ nênA > C, xét phép biến đổi

A B C D

0

F

A A−C C C

0

A

C A−C

|A−2C|

C

0

(152)

Ta thấy phép biến đổi làm giảm thực tổngA+C+ 0thànhC+|A−2C|< A+C ( doC >0

và C < A), giữ cho tổng lẻ, tức tiếp tục áp dụng biến đổi này, ta đến trạng thái mà có số 0, lúc áp dụng phép biến đổi cho trường hợp trên, ta thu kết

(153)

[1] Tuyển tập chuyên đề Tổ hợp - Diễn đàn MathScope, linkhttps://drive.google.com/file/ d/1MIcep1aiAUCun4NfRsYWYTPsu5iu85pp/view?usp=sharing

[2] Tạp chí Pi, tạp chí Tốn học tuổi trẻ

[3] Hội thảo chuyên đề bồi dưỡng học sinh giỏi

[4] Các chuyên đề bồi dưỡng học sinh giỏi nhóm tốn LATEX.

[5] Combinatorics - A Problem-Based Approach, Springer

https://www.facebook.com/OlympiadMathematical https://drive.google.com/file/d/1MIcep1aiAUCun4NfRsYWYTPsu5iu85pp/view?usp=sharing

Ngày đăng: 12/02/2021, 14:04

Từ khóa liên quan

Tài liệu cùng người dùng

  • Đang cập nhật ...

Tài liệu liên quan